sipoysatish@gmail · PDF file · 2013-08-07Explain the concept of Value Added Tax /...

164
SIPOY SATISH www.Cacwacs.wordpress.com [email protected] VALUE ADDED TAX 25 MARKS Including EXAMINATION QUESTIONS CA IPCC MAY-2013/ NOV-2013 F.Y. 2012-13 F. A. 2012 `100

Transcript of sipoysatish@gmail · PDF file · 2013-08-07Explain the concept of Value Added Tax /...

Page 1: sipoysatish@gmail · PDF file · 2013-08-07Explain the concept of Value Added Tax / Explain Taxonomy (process ... Explain accounting treatment of VAT as suggested by ICAI. ... Q26

SIPOY SATISH www.Cacwacs.wordpress.com

[email protected]

VALUE ADDED TAX

25 MARKS

Including

EXAMINATION QUESTIONS

CA

IPCC MAY-2013/

NOV-2013 F.Y. – 2012-13

F. A. – 2012

`100

Page 2: sipoysatish@gmail · PDF file · 2013-08-07Explain the concept of Value Added Tax / Explain Taxonomy (process ... Explain accounting treatment of VAT as suggested by ICAI. ... Q26

2

VALUE ADDED TAX INDEX

Q1 (V. Imp.): Explain the concept of Value Added Tax / Explain Taxonomy (process of making

groups) of VAT/ Different stages of VAT.

Q2 (V. Imp.): Explain concept of input tax credit.

Q3 (V. Imp.): Explain Central Sales Tax including rates under Central Sales Tax.

Q4 (V. Imp.): Write a note on registration under State VAT Act and Central Sales Tax Act.

Q5: Write a note on amendment in registration.

Q6: Write a note on cancellation of registration certificate.

Q7: Explain scope of input tax credit.

Q8 (V. Imp.): Explain purchases not eligible for input tax credit.

Q9 (V. Imp.): Explain provisions of Stock Transfer.

Q10: Explain accounting treatment of VAT as suggested by ICAI. (not covered in syllabus rather it is

only for self reading)

Q11 (V. Imp.): What are the Variants (different types) of VAT.

Q12: What are the rates under VAT.

Q13: Explain non- creditable/non-vatable goods.

Q14 (V. Imp.): Explain concept of excise duty (Central Value Added Tax).

Q15 (V. Imp): Write a note on tax credit in case of manufacturer.

Q16 (V. Imp): What is the common procedure for availing and adjusting cenvat credit for Excise

Duty, Service Tax as per Cenvat Credit Rules, 2004.

Q17 (V. Imp): What are methods for computation of Value Added Tax.

Q18 (V. Imp.): Explain VAT Invoice / Tax Invoice

Q19 (V. Imp.): Explain composition scheme for small traders.

Q20 (Imp.): Write a note on filing of return under state VAT.

Q21 (Imp.): Write a note on assessment under State VAT.

Q22: Write a note on System of Cross Checking.

Q23 (Imp.): Write a note on maintaining of books of accounts and records under State VAT.

Q24 (Imp.): Explain the provisions of audit under State VAT.

Q25 (V. Imp.): Explain merits of VAT.

Q26 (V. Imp): Explain demerits of VAT.

Q27: Explain role of ICAI in VAT.

Q28 (Imp.): Explain role of Chartered Accountant in VAT.

Q29: Explain White Paper.

Page 3: sipoysatish@gmail · PDF file · 2013-08-07Explain the concept of Value Added Tax / Explain Taxonomy (process ... Explain accounting treatment of VAT as suggested by ICAI. ... Q26

Value Added Tax 3

DIRECT TAXES: Income Tax / Wealth Tax

INDIRECT TAXES: Central Excise Duty/

Service Tax/Custom Duty/VAT

DIRECT TAX / INDIRECT TAX

If incidence of tax is borne by the person who is making payment of tax, such tax is called Direct Tax e.g.

Income Tax or Wealth Tax but if incidence is borne by one person and payment is made by some other

person, it is called Indirect Tax like Central Excise Duty, Service Tax, Custom Duty and Sales Tax.

Income Tax and Wealth Tax are levied and collected by Central Government and are monitored by Central

Board of Direct Taxes. Central Excise Duty, Custom Duty and Service Tax are levied and collected by Central Govt. and are

regulated by Central Board of Excise and Custom.

Sales Tax also called Value Added Tax is levied and collected by State Government and it is regulated by

Acts of individual States. E.g. in case of Delhi the relevant Act is Delhi Value Added Tax Act, 2004

Central Board of Direct Taxes (CBDT) and Central Board of Excise and Custom (CBEC) work under

Department of Revenue of Finance Ministry.

Value Added Tax system is applicable only in Indirect Taxes.

At IPCC level there will be one paper of Taxation and there will be 50 marks for Direct Tax and 50 Marks

for Indirect Tax and at CA-FINAL level, there will be 2 papers of Taxation -

Direct Tax 100 Marks (Income Tax – 90 Marks/Wealth Tax – 10 Marks)

Indirect Tax 100 Marks (Service Tax + VAT – 40 Marks / Excise – 40 Marks / Custom – 20 Marks)

Constitutional Validity

Parliament or State legislature can levy tax only if such authority is given in Indian Constitution and such

authority is given in Seventh Schedule of Indian Constitution as per article 246 and there are three list in

Seventh Schedule.

1. Union List – If any matter is mentioned in Union List, parliament can make law with regard to such

matter.

2. State List – If matter is mentioned in State List, State legislature, can make law with regard to such

matter.

3. Concurrent List – If matter is mentioned in Concurrent List, both of the government can make law with

regard to such matter.

Some of the important entries in the Union List are as given below:

82. Taxes on income other than agricultural income.

83. Duties of customs including export duties.

84. Duties of excise on tobacco and other goods manufactured or produced in India except—

(a) alcoholic liquors for human consumption;

(b) opium, Indian hemp and other narcotic drugs and narcotics, but including medicinal and toilet

preparations containing alcohol or any substance included in sub-paragraph (b) of this entry.

92A. Taxes on the sale or purchase of goods other than newspapers, where such sale or purchase takes place

in the course of inter-State trade or commerce.

92C. Taxes on services.

97. Any other matter not enumerated in List II or List III including any tax not mentioned in either of those

Lists.

Some of the important entries in State List are as given below:

54. Taxes on the sale or purchase of goods other than newspapers, subject to the provisions of entry 92A of

List I.

46. Taxes on agricultural income

Page 4: sipoysatish@gmail · PDF file · 2013-08-07Explain the concept of Value Added Tax / Explain Taxonomy (process ... Explain accounting treatment of VAT as suggested by ICAI. ... Q26

Value Added Tax 4

VALUE ADDED TAX Concept of Value Added Tax was introduced first of all in France in 1954.

It was introduced in India in Central Excise Duty w.e.f 1st March 1986 and at that time it was called

MODVAT (Modified Value Added Tax) but afterwards its name was changed as CENVAT (Central Value

Added Tax).

Central Excise Duty and Service Tax is charged by Central Govt. and its tax credit is called Cenvat and the

rules for Cenvat Credit are given in Cenvat Credit Rules, 2004 (Applicable w.e.f 10.09.2004).

Concept of tax credit was introduced in Sales Tax in the year 2005 and Sales Tax is being charged by the

State Govt. and accordingly every State Govt. has its own Value Added Tax Act (For example in Delhi

Delhi Value Added Tax Act, 2004)

Tax credit rules are the same for different tax but there is some difference because some of the tax are

charged by the Central Govt. and some of the tax are charged by State Govt.

Question 1 (V. Imp.): Explain the concept of Value Added Tax / Explain Taxonomy (process of

making groups) of VAT/ Different stages of VAT.

Answer:

Before VAT

If a Manufacturer has sold goods to the Wholesaler and has charged sales tax and Wholesaler has further

sold the same goods to the Retailer and has again charged sales tax, in this case there are two anomalies

(defect):

(i) Sales tax has been charged twice on the same item.

(ii) There is sales tax on sales tax (it is called cascading effect)

Example

If a Manufacturer has manufactured certain goods and cost price is `1,00,000 and his expenses plus profit is

`20,000 and sales tax is 10%, in this case goods shall be sold for (`1,00,000 plus 20,000) + (10% of

`1,20,000) = `1,32,000.

If the Wholesaler is purchasing goods from Manufacturer for `1,32,000 and his expenses plus profit is

`20,000 and is charging sales tax @ 10%, the goods shall be sold for (`1,32,000 plus 20,000) + (10% of

`1,52,000) = `1,67,200.

The Retailer has added his expenses plus profit of 20,000 and is charging sales tax @ 10%, the goods shall

be sold for (`1,67,200 plus 20,000) + (10% of `1,87,200) = `2,05,920.

In this case sales tax has been charged three times on the original amount of `1,20,000 (once by

manufacturer, once by the Wholesaler and once by retailer) accordingly a single item has suffered sales tax

three times.

Further the Wholesaler has charged sales tax on the amount of the sales tax paid by him i.e. there is sales tax

on sales tax which is called Cascading Effect.

After VAT

The above anomalies can be rectified with the help of VAT.

Under VAT, the Wholesaler is allowed to take tax credit of `12,000 being the sales tax paid by him and he

Page 5: sipoysatish@gmail · PDF file · 2013-08-07Explain the concept of Value Added Tax / Explain Taxonomy (process ... Explain accounting treatment of VAT as suggested by ICAI. ... Q26

Value Added Tax 5

will be selling the goods for `1,40,000 (`1,20,000 + `20,000), sales tax charged by him shall be `14,000.

Since he has taken credit of `12,000, net sales tax payable by him shall be `2,000 and it will be called value

added tax i.e. sales tax is only on the value addition of `20,000.

Similarly, the Retailer shall be allowed to take credit for the tax paid by him and he will pay sales tax only

on the value addition.

The tax paid by the Wholesaler to the Manufacturer shall be called his Input tax (`12,000) and tax charged

by him from the Retailer shall be called Output tax (`14,000) and tax payable by him shall be called Net tax

(`2,000).

Value Added Tax is a Multi Stage tax and is being charged at every stage of sale and these stages are called

stages of VAT and are as given below:

1. Manufacturer to Distributor

2. Distributor to Wholesaler

3. Wholesaler to Retailer

4. Retailer to Consumer

Question 2 (V. Imp.): Explain concept of input tax credit.

Answer:

If any Registered Dealer is purchasing goods within a particular State and has paid value added tax and

subsequently the goods were sold either in the same State or to some other State, in that case such

Registered Dealer shall be allowed to take credit for input tax, however, tax credit is allowed only to the

Registered Dealers and further registered dealer should purchase goods only from registered dealer i.e. if the

goods have been purchased from unregistered dealer, no VAT credit is allowed. Registration is required if

the sale turnover has exceeded the threshold limit (basic limit) of `10 lakh during the year. Voluntary

registration is allowed to every dealer at any time.

Example

Mr. D is a dealer registered in Delhi and he purchased goods of `20,00,000 plus Delhi VAT @ 12.5% and

sold the goods in Delhi for `30,00,000 plus Delhi VAT @ 12.5%, in this case, tax credit for input tax shall

be allowed and computation of output tax, input tax and net tax shall be as given below:

Sale 30,00,000

Output Tax @ 12.5% 3,75,000

Less: Input Tax credit (20,00,000 x 12.5%) 2,50,000

Net Delhi VAT Payable 1,25,000

If in the above case goods were sold to an unregistered dealer in UP and Central Sales Tax was charged @

12.5%, in that case also tax credit is allowed and tax treatment shall be as given below:

Sale 30,00,000

Output Tax – Central Sales Tax @ 12.5% 3,75,000

Less: Input Tax credit (20,00,000 x 12.5%) 2,50,000

Net CST Payable 1,25,000

If the goods have been sold to some Registered Dealer in U.P. and Central Sales Tax was charged @ 2%, in

that case also, tax credit is allowed and tax treatment shall be as given below:

Sale 30,00,000

Output Tax – Central Sales Tax @ 2% 60,000

Less: Input Tax credit (20,00,000 x 12.5%) 2,50,000

Page 6: sipoysatish@gmail · PDF file · 2013-08-07Explain the concept of Value Added Tax / Explain Taxonomy (process ... Explain accounting treatment of VAT as suggested by ICAI. ... Q26

Value Added Tax 6

Balance VAT Credit (to be carried forward or refund can be taken) 1,90,000

If the goods have been purchased from outside the State and has paid Central Sales Tax to that particular

State, tax credit for such INPUT TAX is not allowed. If in the above case goods were purchased from

Punjab and CST was paid @ 2% and the goods were sold either in Delhi or in some others States, tax credit

is not allowed and tax treatment shall be as given below:

Example

(a) Goods sold in Delhi

Sale 30,00,000

Output Tax @ 12.5% 3,75,000

Input tax credit is not allowed Nil

Net Delhi VAT Payable 3,75,000

(b) Goods sold to Unregistered Dealer in U.P.

Sale 30,00,000

Output Tax – Central Sales Tax @ 12.5% 3,75,000

Input tax credit is not allowed Nil

Net CST Payable 3,75,000

(c) Goods sold to Registered Dealer in U.P. against Form ‘C’

Sale 30,00,000

Output Tax – Central Sales Tax @ 2% 60,000

Input tax credit is not allowed Nil

Net CST Payable 60,000

Tax credit in case of goods exempt from output tax

If raw material or other goods have been purchased and have been utilized in manufacturing, processing or

packing of the product which is exempt from VAT, no VAT credit shall be allowed on such purchases.

Tax credit in case of Export or supply to Special Economic Zone

If the goods have been exported, in that case such export sales are exempt from excise duty or VAT etc but

even then as a special case tax credit is allowed and export goods are also called zero-rated goods. Similarly

if the goods have been sold in Special Economic Zone, there will be same procedure as in case of export.

Special Economic Zone means a specific area which has been declared to be SEZ by the Government and

the persons having their units in the SEZ shall be required to export the goods manufactured by them and

there will not be any output tax. The persons selling goods or providing services in the SEZ shall not charge

any tax from the buyers in the SEZ but the seller shall be allowed tax credit as a special case.

Example

Mr. X is a manufacturer and is registered in DVAT ACT, 2004 and he has purchased raw material for

`20,00,000 and paid DVAT @ 4% and manufactured certain goods which were sold in Delhi for `30,00,000

and the goods are exempt from DVAT, in this case, tax credit is not allowed and tax treatment shall be as

given below:

Sale 30,00,000

Output Tax Nil

Input tax credit is not allowed

Net Delhi VAT Payable Nil

If in the above case goods have been exported or goods have been sold in Special Economic Zone (SEZ), tax

treatment shall be as given below:

Sale 30,00,000

Page 7: sipoysatish@gmail · PDF file · 2013-08-07Explain the concept of Value Added Tax / Explain Taxonomy (process ... Explain accounting treatment of VAT as suggested by ICAI. ... Q26

Value Added Tax 7

Output Tax Nil

Input tax credit is allowed (20,00,000 x 4%) 80,000

Net Delhi VAT Payable Nil

Dealer can claim refund for such unutilized input tax credit.

Stock Transfer

If any dealer has purchased goods within the state and the goods were stock transferred to some other state,

in such cases VAT credit is allowed after retaining 2% of the input tax.

Example

ABC Ltd. has purchased raw material for `20,00,000 in the state of Delhi and paid DVAT @ 12.5% and

goods were stock transferred to there branch in U.P., in this case input tax credit allowed shall be

20,00,000 x 10.5% (12.5% - 2%) 2,10,000

Question 3 (V. Imp.) : Explain Central Sales Tax including rates under Central Sales Tax.

Answer:

Central Sales Tax was introduced in 1956 through Central Sales Tax Act 1956.

Prior to levy of CST, State Governments use to charge sales tax even in case of inter-state sales and there

was multiple taxation on the basis of nexus theory e.g. If seller was in Punjab and buyer was in U.P., both

of the States collected sales tax because of nexus i.e. connection of the sales transaction with each of the

state.

In order to check the problem of multiple taxation, Central Sales Tax Act was enacted (to pass a law) in

1956 and in case of Inter State sale, Central Sales Tax was levied and it was to be collected by the State

Government from where the sales has taken place. It is collected by the State Government on behalf of the

Central Government but as per provisions of Central Sales Tax Act, the tax so collected shall be retained by

the respective State Government. As the purpose of levying of CST was not to collect Central Sales Tax by

the Central Government rather the purpose was to check multiple taxation.

If Central Sales Tax has been paid in one particular State as input tax and output tax is to be paid in some

other State, tax credit shall not be allowed for such input tax e.g. A registered dealer of Delhi Mr. D has

purchased certain goods from Punjab for `1,00,000 and paid Central Sales Tax to the Punjab Government

`2,000 @ 2% and subsequently the same goods were sold in Delhi for `1,50,000 and has charged local sales

tax @ 12.5% amounting to `18,750, in this case VAT credit of `2,000 shall not be allowed and entire amount

of `18,750 shall be paid to the Delhi Government.

If the Dealer has purchased goods from Delhi and has sold the goods in U.P. and has collected Central Sales

Tax from the buyer in U.P., in this case such CST has to be paid to Delhi Government hence VAT credit for

input tax paid in Delhi shall be allowed.

Rates under Central Sales Tax

If the goods have been sold to a dealer registered under Central Sales Tax, rate of CST shall be equal to LST

but subject to highest rate of CST at 2%. If goods have been sold to any other person, rate of CST shall be

equal to LST and it can be shown as given below:

Rate of LST Sale to Dealer Registered under CST Sale to any Other Person

1% 1% 1%

2% 2% 2%

3% 2% 3%

10% 2% 10%

12% 2% 12%

Page 8: sipoysatish@gmail · PDF file · 2013-08-07Explain the concept of Value Added Tax / Explain Taxonomy (process ... Explain accounting treatment of VAT as suggested by ICAI. ... Q26

Value Added Tax 8

12.5% 2% 12.5%

The dealer selling the goods to the registered dealer shall obtain Form C from the purchasing dealer being

the proof of his being a registered dealer.

If the goods are sold to the Government department, it will not be considered to be registered dealer.

Question 4 (V. Imp.): Write a note on registration under State VAT Act and Central Sales Tax Act.

Answer:

Registration under State Value Added Tax Act

Compulsory Registration

A dealer must apply for registration in the following cases:

(i) If the turnover has exceeded `10 lakhs at any time during the year.

(ii) He is registered under Central Sales Tax Act, 1956.

(iii) He is purchasing goods from outside the state for sale within the state.

Example

Mr. X is an unregistered dealer in Delhi,. His sales turnover is `5,00,000 but he is purchasing some of the

goods from outside Delhi, in this case, he should apply for compulsory registration.

If in the above case, he is not purchasing goods from other States but he is selling some of the goods to other

States, in that case also registration is required. If he is not purchasing goods from other States and also not

selling goods to other states, registration is not required but if turnover is exceeding `10,00,000, registration

is required.

Voluntary Registration

Any dealer may apply for voluntary registration under State Value Added Tax Act at any time.

Only dealer registered under State Value Added Tax Act can charge VAT and can issue tax invoice and

VAT Credit is only allowed on the basis of tax invoice.

Registration under Central Sales Tax Act, 1956

Compulsory Registration

If any dealer has effected any Sale from one state to the other, registration is required under Central Sales

Tax Act.

Voluntary Registration

If a dealer is registered under State Value Added Tax Act, he may apply for registration under Central Sales

Tax Act at any time.

Registration under State Value Added Tax Act and Central Sales Tax Act shall be different.

TIN (Tax Payer’s Identification Number) is a code to identify a tax payer. It is the registration number of the

dealer. The taxpayer’s identification number will consist of 11 digit numerals throughout the country. First

two characters will represent the State code as used by the Union Ministry of Home Affairs. The set-off of

the next nine characters will be, however, different in different States. TIN will help cross-check information

Page 9: sipoysatish@gmail · PDF file · 2013-08-07Explain the concept of Value Added Tax / Explain Taxonomy (process ... Explain accounting treatment of VAT as suggested by ICAI. ... Q26

Value Added Tax 9

on tax payer compliance, for example, the selective cross-checking of sales and purchases among VAT

taxpayers.

Question 5: Write a note on amendment in registration.

Answer:

Amendment in Registration certificate

Registration certificate can be amended in the following situations:

1. If there is change in the name of business.

2. If there is change in the place of business.

3. If there is change in the nature of business i.e. Trading is converted into Manufacturing or Manufacturing

is converted into Trading.

4. If one or more new goods have been included or one or more goods have been deleted

5. If there is change in the Constitution e.g. Partnership Firm is converted into Proprietary or Proprietary is

converted into Partnership Firm.

6. If there is any other similar change.

Amendment can be made at the request of the dealer or the department can make amendment on its own.

Question 6: Write a note on cancellation of registration certificate.

Answer:

Cancellation of registration

VAT registration can be cancelled on:

(i) Discontinuance of business; or

(ii) Disposal of business; or

(iii) Death of the dealer

(iv) Annual turnover falling below the specified limit e.g. In case of Delhi VAT if turnover becomes less

than 10 lakhs

(v) Violation of the provisions of State VAT Act leading to cancellation of certificate

Illustration 1: Compute the invoice value to be charged and amount of tax payable under VAT by a

Registered Dealer who had purchased goods for `1,50,000 (exclusive of VAT) from the same State and after

adding for expenses of `12,000 and profit of `25,000 had sold in the same State. The rate of VAT on

purchase and sales is 12.5%.

Solution: ` Purchase price 1,50,000

Add: Expenses 12,000

Add: Profit 25,000

Page 10: sipoysatish@gmail · PDF file · 2013-08-07Explain the concept of Value Added Tax / Explain Taxonomy (process ... Explain accounting treatment of VAT as suggested by ICAI. ... Q26

Value Added Tax 10

Amount to be billed 1,87,000

Add: VAT @ 12.5% - Output Tax 23,375

Total invoice value 2,10,375

VAT Payable

VAT charged in the invoice - Output Tax 23,375

Less: VAT credit on input 12.5% of `1,50,000 - Input Tax (18,750)

Net VAT Payable 4,625

(b) Presume the goods were sold in some other States to Unregistered Dealer.

Solution:

Purchase price 1,50,000

Add: Expenses 12,000

Add: Profit 25,000

Amount to be billed 1,87,000

Add: CST @ 12.5% - Output Tax 23,375

Total invoice value 2,10,375

CST Payable

CST charged in the invoice - Output Tax 23,375

Less: VAT credit on input 12.5% of `1,50,000 - Input Tax (18,750)

Net CST Payable 4,625

(c) Presume the goods were sold in some other States to Registered Dealer against Form ‘C’.

Solution:

Purchase price 1,50,000

Add: Expenses 12,000

Add: Profit 25,000

Amount to be billed 1,87,000

Add: CST @ 2% - Output Tax 3,740

Total invoice value 1,90,740

CST Payable

CST charged in the invoice - Output Tax 3,740

Less: VAT credit on input 12.5% of `1,50,000 - Input Tax (18,750)

CST Payable Nil

Balance VAT Credit (to be carried forward or refund can be taken) 15,010

Illustration 2: Compute the VAT amount payable by Mr. A (Registered Dealer) who purchases goods from

a manufacturer on payment of `4,50,000 (including VAT) and earns 15% profit on purchase. The goods

have been sold to retailers and VAT rate on purchase and sale is 12.5%.

Solution: ` Purchase price 4,50,000.00

Less: Input tax (4,50,000 x 12.5% / 112.5%) (50,000.00)

4,00,000.00

Add: Profit (4,00,000 x 15%) 60,000.00

Amount to be billed 4,60,000.00

Add: VAT @ 12.5% - Output Tax 57,500.00

Page 11: sipoysatish@gmail · PDF file · 2013-08-07Explain the concept of Value Added Tax / Explain Taxonomy (process ... Explain accounting treatment of VAT as suggested by ICAI. ... Q26

Value Added Tax 11

Total invoice value 5,17,500.00

VAT Payable

Output tax 57,500.00

Less: Tax credit (12.5% of `4,00,000) - Input Tax (50,000.00)

Net VAT Payable 7,500.00

(b) Presume the goods were sold in some other States to Unregistered Dealer.

Solution: ` Purchase price 4,50,000.00

Less: Input tax (4,50,000 x 12.5% / 112.5%) (50,000.00)

4,00,000.00

Add: Profit (4,00,000 x 15%) 60,000.00

Amount to be billed 4,60,000.00

Add: CST @ 12.5% - Output Tax 57,500.00

Total invoice value 5,17,500.00

CST Payable

Output tax 57,500.00

Less: Tax credit (12.5% of `4,00,000) - Input Tax (50,000.00)

Net CST Payable 7,500.00

(c) Presume the goods were sold in some other States to Registered Dealer against Form ‘C’.

Solution: ` Purchase price 4,50,000.00

Less: Input tax (4,50,000 x 12.5% / 112.5%) (50,000.00)

4,00,000.00

Add: Profit (4,00,000 x 15%) 60,000.00

Amount to be billed 4,60,000.00

Add: CST @ 2% - Output Tax 9,200.00

Total invoice value 4,69,200.00

CST Payable

CST-Output tax 9,200.00

Less: Tax credit (12.5% of `4,00,000) - Input Tax (50,000.00)

CST Payable Nil

Balance VAT Credit (to be carried forward or refund can be taken) 40,800.00

Illustration 3: Mr. X (Registered Dealer) is a trader in Delhi and he has purchased certain goods from

Punjab for `4,00,000 and has paid central sales tax @ 2%.

He has sold all the goods in the state of Delhi for `6,00,000 plus VAT @ 12.5%.

He has purchased certain goods in Delhi for `5,00,000 and paid VAT @ 12.5% and all the goods were sold

by him under inter state sale to a registered dealer in U.P. for `7,00,000 plus central sales tax @ 2%.

Show VAT calculation.

Page 12: sipoysatish@gmail · PDF file · 2013-08-07Explain the concept of Value Added Tax / Explain Taxonomy (process ... Explain accounting treatment of VAT as suggested by ICAI. ... Q26

Value Added Tax 12

Solution :

Computation of VAT payable:

Purchase from Punjab and sold in Delhi ` Purchase price 4,00,000

Add: Central sales tax @ 2% 8,000

Purchase Price 4,08,000

Sale 6,00,000

Add: VAT @ 12.5% - Output tax 75,000

Selling Price 6,75,000

Purchase from Delhi and sold in U.P. ` Purchase price 5,00,000

Add: VAT @ 12.5% - VAT credit 62,500

Purchase Price 5,62,500

Sale 7,00,000

Add: Central sales tax @ 2% - Output tax 14,000

Selling Price 7,14,000

State VAT 75,000

Less: VAT credit 62,500

Net State VAT payable 12,500

CST Payable 14,000

Illustration 4: Mr. X (Registered Dealer in Delhi) is a manufacturer and he has purchased raw material R1

from Punjab for `2,00,000 plus central sales tax @ 2%.

He has purchased raw material R2 in Delhi for `3,30,000 inclusive of VAT @ 10%.

His processing charges is `1,00,000 and profit margin is `1,00,000.

Half of the goods were sold in Delhi and VAT payable is @ 10% and remaining half were sold to a person

in U.P. under inter state sale and has charged central sales tax @ 2%.

Show working for VAT.

Solution: ` Computation of VAT payable:

Raw material – R1

Purchase price 2,00,000

Add: Central sales tax @ 2% 4,000

Purchase Price 2,04,000

Raw material – R2

Purchase price 3,30,000

Less: Input tax (3,30,000 x 10 / 110) 30,000

Net purchase price exclusive of VAT 3,00,000

Cost of final product

Raw material – R1 2,04,000

Raw material – R2 3,00,000

Processing charges 1,00,000

Profit margin 1,00,000

Page 13: sipoysatish@gmail · PDF file · 2013-08-07Explain the concept of Value Added Tax / Explain Taxonomy (process ... Explain accounting treatment of VAT as suggested by ICAI. ... Q26

Value Added Tax 13

Total 7,04,000

Goods sold in Delhi

Assessable value 3,52,000

Add: VAT @ 10% - Output tax 35,200

Sales value 3,87,200

Goods sold in U.P.

Assessable value 3,52,000

Add: Central sales tax @ 2% - Output tax 7,040

Sales value 3,59,040

State VAT 35,200

Less: VAT credit 30,000

Net State VAT payable 5,200

CST Payable 7,040

Illustration 5: Mr. X (Registered Dealer) is a manufacturer sells goods to Mr. B (Registered Dealer), a

distributor for `2,000. Mr. B sells goods to Mr. K (Registered Dealer), a wholesaler for `2,400.

The wholesaler sells the goods to a Retailer (Registered Dealer) for `3,000. The retailer sold the goods to

consumers for `4,000.

All the above amounts are exclusive of VAT.

Compute input tax credit, output tax and net tax under invoice method for each of the person and VAT rate

is @ 12.5%.

(b) Presume all the above amounts are inclusive of VAT @ 12.5%.

Solution (a): ` Manufacturer (Mr. X)

Sale price 2,000

Add: VAT @ 12.5% 250

Total invoice value 2,250

VAT A/C:

Output tax 250

Less: Tax credit Nil

Net Tax Payable 250

Distributor (Mr. B)

Purchase price 2,000

Add: Profit 400

Amount to be billed 2,400

Add: VAT @ 12.5% 300

Total invoice value 2,700

VAT A/C:

Output tax 300

Less: Tax credit 250

Net Tax Payable 50

Page 14: sipoysatish@gmail · PDF file · 2013-08-07Explain the concept of Value Added Tax / Explain Taxonomy (process ... Explain accounting treatment of VAT as suggested by ICAI. ... Q26

Value Added Tax 14

Wholesaler (Mr. K)

Purchase price 2,400

Add: Profit 600

Amount to be billed 3,000

Add: VAT @ 12.5% 375

Total invoice value 3,375

VAT A/C:

Output tax 375

Less: Tax credit 300

Net Tax Payable 75

Retailer

Purchase price 3,000

Add: Profit 1,000

Amount to be billed 4,000

Add: VAT @ 12.5% 500

Total invoice value 4,500

VAT A/C:

Output tax 500

Less: Tax credit 375

Net Tax Payable 125

Solution (b): ` Manufacturer (Mr. X)

Sale price 2,000

Output tax (2,000 x 12.5 % / 112.5%) 222

VAT A/C:

Output tax 222

Less: Tax credit Nil

Net Tax Payable 222

Distributor (Mr. B)

Sale price 2,400

Output tax (2,400 x 12.5% / 112.5%) 267

VAT A/C:

Output tax 267

Less: Tax credit 222

Net Tax Payable 45

Wholesaler (Mr. K)

Sale price 3,000

Output tax (3,000 x 12.5% / 112.5%) 333

VAT A/C:

Output tax 333

Less: Tax credit 267

Net Tax Payable 66

Page 15: sipoysatish@gmail · PDF file · 2013-08-07Explain the concept of Value Added Tax / Explain Taxonomy (process ... Explain accounting treatment of VAT as suggested by ICAI. ... Q26

Value Added Tax 15

Retailer

Sale price 4,000

Output tax (4,000 x 12.5% / 112.5%) 444

VAT A/C:

Output tax 444

Less: Tax credit 333

Net Tax Payable 111

Question 7: Explain scope of input tax credit.

Answer:

Scope of input tax credit

For the purpose of claiming input tax credit, the taxable goods should be purchased for any one of the

following purposes:

1. for sale/resale within the State;

2. for sale to other parts of India in the course of inter-state trade or commerce;

3. to be used as containers or packing materials or raw materials or consumable stores, required for the

purpose of manufacture of taxable goods or in the packing of such manufactured goods intended for sale;

4. for being used in the execution of a works contract (rendering of services alongwith supply of goods e.g.

construction contract);

5. to be used as capital goods required for the purpose of manufacture or resale of taxable goods;

6. to be used as raw materials or capital goods or consumable stores and packing materials/containers for

manufacturing/packing goods to be sold in the course of export out of the territory of India.

Question 8 (V. Imp.): Explain purchases not eligible for input tax credit.

Answer:

Purchases not eligible for input tax credit

Input tax credit shall not be allowed in the following circumstances:

1. Purchases from unregistered dealers;

2. Purchases from registered dealer who opt for composition scheme;

3. Purchase of goods as may be notified by the State Government;

4. Purchase of goods, which are being utilized in the manufacture of exempted goods;

5. Goods in stock, which have suffered tax under an earlier Act but under VAT Act they are covered under

exempted items;

6. Goods imported from outside the territory of India;

7. Goods imported from other States.

Illustration 6:

Page 16: sipoysatish@gmail · PDF file · 2013-08-07Explain the concept of Value Added Tax / Explain Taxonomy (process ... Explain accounting treatment of VAT as suggested by ICAI. ... Q26

Value Added Tax 16

Mr. X is a Dealer Registered in Delhi Value Added Tax Act, 2004 and also under Central Sales Tax Act,

1956 and he has submitted the informations as given below:

(i) Purchased Goods A from Delhi `1,00,000 and paid VAT @ 4% and sold the goods in Delhi for `1,50,000

and charged VAT @ 4%.

(ii) Purchased goods B from U.P. for `2,00,000 and paid central sales tax @ 2% and sold goods in Delhi for

`2,50,000 and charged VAT @ 12.5%.

(iii) Purchased goods C from Delhi for `4,00,000 and paid VAT @ 12.5% and sold the goods to a registered

dealer in Orissa for `4,75,000 and charged central sales tax @ 2%

(iv) Purchased goods D for `5,00,000 in Delhi and paid VAT @ 12.5% and sold the goods for `5,50,000 to

an unregistered dealer in Punjab and charged central sales tax @ 12.5%.

(v) Purchased goods E from Madhya Pradesh for `3,00,000 and paid central sales tax @ 1% and sold goods

in Maharashtra for `3,50,000 and charged central sales tax @ 1%.

(vi) Purchased goods F from Delhi `7,00,000 and paid VAT @ 1% and the goods were sold to an

unregistered dealer in Maharashtra for `7,50,000 and charged central sales tax @ 1%.

(vii) Purchased goods G for `6,00,000 in Delhi and paid VAT @ 12.5% and goods were stock transferred to

some other state.

(viii) Purchased goods H for `8,00,000 in Delhi and paid VAT @ 4% and goods were exported for `8,50,000

and no VAT was charged (because as per section 6 Central Sales Tax Act, 1956, CST can not be charged in

case of export sale.)

(ix) Purchased goods I for `9,00,000 in Delhi and paid VAT @ 12.5% and sold the goods to a manufacturer

in SEZ for `10,50,000 and no VAT was charged.

Show the tax treatment for VAT and also compute his income tax liability for the assessment year 2013-14.

Solution: ` (i)

Purchased Goods A from Delhi 1,00,000

Add: VAT @ 4% 4,000

Purchase Price 1,04,000

Input tax credit 4,000

Goods sold in Delhi 1,50,000

Add: VAT @ 4% 6,000

Selling Price 1,56,000

(ii)

Purchased goods B from U.P. 2,00,000

Add: Central sales tax @ 2% 4,000

Purchase Price 2,04,000

Input tax credit Nil

Goods sold in Delhi 2,50,000

Page 17: sipoysatish@gmail · PDF file · 2013-08-07Explain the concept of Value Added Tax / Explain Taxonomy (process ... Explain accounting treatment of VAT as suggested by ICAI. ... Q26

Value Added Tax 17

Add: VAT @ 12.5% 31,250

Selling Price 2,81,250

(iii)

Purchased goods C from Delhi 4,00,000

Add: VAT @ 12.5% 50,000

Purchase Price 4,50,000

Input tax credit 50,000

Goods sold in Orissa 4,75,000

Add: Central sales tax @ 2% 9,500

Selling Price 4,84,500

(iv)

Purchased goods D from Delhi 5,00,000

Add: VAT @ 12.5% 62,500

Purchase Price 5,62,500

Input tax credit 62,500

Goods sold in Punjab to unregistered dealer 5,50,000

Add: Central sales tax @ 12.5% 68,750

Selling Price 6,18,750

(v)

Purchased goods E from Madhya Pradesh 3,00,000

Add: Central sales tax @ 1% 3,000

Purchase Price 3,03,000

Input tax credit Nil

Goods sold in Maharashtra 3,50,000

Add: Central sales tax @ 1% 3,500

Selling Price 3,53,500

(vi)

Purchased goods F from Delhi 7,00,000

Add: VAT @ 1% 7,000

Purchase Price 7,07,000

Input tax credit 7,000

Goods sold in Maharashtra to unregistered dealer 7,50,000

Add: Central sales tax @ 1% 7,500

Selling Price 7,57,500

(vii)

Purchased goods G from Delhi 6,00,000

Add: VAT @ 12.5% 75,000

Purchase Price 6,75,000

Goods Stock transferred 6,00,000

VAT credit allowed in stock transfer (6,00,000 x 10.5%) 63,000

(in case of stock transfer, VAT credit shall be allowed after retaining 2%)

(viii)

Purchased goods H from Delhi 8,00,000

Add: VAT @ 4% 32,000

Purchase Price 8,32,000

Input tax credit 32,000

Page 18: sipoysatish@gmail · PDF file · 2013-08-07Explain the concept of Value Added Tax / Explain Taxonomy (process ... Explain accounting treatment of VAT as suggested by ICAI. ... Q26

Value Added Tax 18

Goods exported 8,50,000

(ix)

Purchased goods I from Delhi 9,00,000

Add: VAT @ 12.5% 1,12,500

Purchase Price 10,12,500

Input tax credit 1,12,500

Goods sold to manufacturer in SEZ 10,50,000

VAT A/C

Particulars ` ` OUTPUT TAX VAT CST

Goods A 6,000 ---

Goods B 31,250 ---

Goods C --- 9,500

Goods D --- 68,750

Goods E --- 3,500

Goods F --- 7,500

Goods G (Stock transfer) Not applicable ---

Goods H (Export) Nil --

Goods I (Sale to SEZ) Nil --

37,250 89,250

LESS: INPUT TAX CREDIT

Goods A 4,000

Goods B Not allowed

Goods C 50,000

Goods D 62,500

Goods E Not allowed

Goods F 7,000

Goods G 63,000

Goods H 32,000

Goods I 1,12,500

3,31,000

After adjusting output VAT of `37,250 and CST of `89,250, there will be unutilised VAT credit of `2,04,500

and it can be set off from other output tax or it can be carried forward or refund can be claimed but

procedure differs from State to State. At the year end it should be shown on the assets side of the balance

sheet under the head CURRENT ASSETS, LOAN AND ADVANCES.

Computation of income tax liability

Particulars

Purchases

Amount

` Particulars

Sales

Amount

` Goods A 1,00,000 Goods A 1,50,000

Goods B 2,04,000 Goods B 2,50,000

Goods C 4,00,000 Goods C 4,75,000

Goods D 5,00,000 Goods D 5,50,000

Goods E 3,03,000 Goods E 3,50,000

Goods F 7,00,000 Goods F 7,50,000

Goods H 8,00,000 Goods H 8,50,000

Goods I 9,00,000 Goods I 10,50,000

Page 19: sipoysatish@gmail · PDF file · 2013-08-07Explain the concept of Value Added Tax / Explain Taxonomy (process ... Explain accounting treatment of VAT as suggested by ICAI. ... Q26

Value Added Tax 19

Net profit 5,18,000

44,25,000 44,25,000

Income under the head Business/Profession 5,18,000

Gross Total Income 5,18,000

Less: Deduction u/s 80C to 80U Nil

Total Income 5,18,000

Tax on `5,18,000 at slab rate 33,600

Add: Education cess @ 2% 672

Add: SHEC @ 1% 336

Tax Liability 34,608

Rounded off u/s 288B 34,610

Income shall be computed exclusive of VAT because any VAT collected shall be paid to the Government

and it will not be considered to be income. Similarly VAT paid by the dealer is collected from the customer

hence it will not be considered to be expense. If any balance is left in VAT credit, it will be shown as asset

in the Balance Sheet on the last day of the relevant financial year. Further, the stock transfer of goods G is

having a netural effect and thus ignored for calculation of business/profession income.

Illustration 7:

Mr. X is a Dealer Registered in Delhi Value Added Tax Act, 2004 and also under Central Sales Tax Act,

1956 and he has submitted the informations as given below:

(i) Purchased Goods A from Delhi `1,00,000 inclusive of VAT @ 4% and sold the goods in Delhi for

`1,50,000 inclusive of VAT @ 4%.

(ii) Purchased goods B from U.P. for `2,00,000 inclusive of central sales tax @ 2% and sold goods in Delhi

for `2,50,000 inclusive of VAT @ 12.5%.

(iii) Purchased goods C from Delhi for `4,00,000 inclusive of VAT @ 12.5% and sold the goods to a

registered dealer in Orissa for `4,75,000 inclusive of central sales tax @ 2%

(iv) Purchased goods D for `5,00,000 in Delhi inclusive of VAT @ 12.5% and sold the goods for `5,50,000

to an unregistered dealer in Punjab inclusive of central sales tax @ 12.5%.

(v) Purchased goods E from Madhya Pradesh for `3,00,000 inclusive of central sales tax @ 1% and sold

goods in Maharashtra for `3,50,000 inclusive of central sales tax @ 1%.

(vi) Purchased goods F from Delhi `7,00,000 inclusive of VAT @ 1% and the goods were sold to an

unregistered dealer in Maharashtra for `7,50,000 inclusive of central sales tax @ 1%.

(vii) Purchased goods G for `6,00,000 in Delhi inclusive of VAT @ 12.5% and goods were stock transferred

to some other state.

(viii) Purchased goods H for `8,00,000 in Delhi inclusive of VAT @ 4% and goods were exported for

`8,50,000 and no VAT was charged (because as per section 6 Central Sales Tax Act, 1956, CST cannot be

charged in case of export sale.)

Page 20: sipoysatish@gmail · PDF file · 2013-08-07Explain the concept of Value Added Tax / Explain Taxonomy (process ... Explain accounting treatment of VAT as suggested by ICAI. ... Q26

Value Added Tax 20

(ix) Purchased goods I for `9,00,000 in Delhi inclusive of VAT @ 12.5% and sold the goods to a

manufacturer in SEZ for `10,50,000 and no VAT was charged.

Show the tax treatment for VAT and also compute his income tax liability for the assessment year 2013-14.

Solution: ` (i)

Purchased Goods A from Delhi

Purchase Price 1,00,000

VAT (1,00,000 x 4% / 104%) 3,846

Purchase price net of Tax 96,154

Input tax credit 3,846

Goods sold in Delhi

Selling Price 1,50,000

VAT (1,50,000 x 4% / 104%) 5,769

Selling Price net of Tax 1,44,231

(ii)

Purchased goods B from U.P.

Purchase Price 2,00,000

Central sales tax (2,00,000 x 2% / 102%) 3,922

Input tax credit Nil

Goods sold in Delhi

Selling Price 2,50,000

VAT (2,50,000 x 12.5% / 112.5%) 27,778

Selling Price net of Tax 2,22,222

(iii)

Purchased goods C from Delhi

Purchase Price 4,00,000

VAT (4,00,000 x 12.5% / 112.5%) 44,444

Purchase price net of Tax 3,55,556

Input tax credit 44,444

Goods sold in Orissa

Selling Price 4,75,000

Central sales tax (4,75,000 x 2% / 102%) 9,314

Selling Price net of Tax 4,65,686

(iv)

Purchased goods D from Delhi

Purchase Price 5,00,000

VAT (5,00,000 x 12.5% / 112.5%) 55,556

Purchase price net of Tax 4,44,444

Input tax credit 55,556

Goods sold in Punjab to unregistered dealer

Selling Price 5,50,000

Central sales tax (5,50,000 x 12.5% / 112.5%) 61,111

Selling Price net of Tax 4,88,889

(v)

Purchased goods E from Madhya Pradesh

Page 21: sipoysatish@gmail · PDF file · 2013-08-07Explain the concept of Value Added Tax / Explain Taxonomy (process ... Explain accounting treatment of VAT as suggested by ICAI. ... Q26

Value Added Tax 21

Purchase Price 3,00,000

Central sales tax (3,00,000 x 1% / 101%) 2,970

Input tax credit Nil

Goods sold in Maharashtra

Selling Price 3,50,000

Central sales tax (3,50,000 x 1% / 101%) 3,465

Selling Price net of Tax 3,46,535

(vi)

Purchased goods F from Delhi

Purchase Price 7,00,000

VAT (7,00,000 x 1% / 101%) 6,931

Purchase price net of Tax 6,93,069

Input tax credit 6,931

Goods sold in Maharashtra to unregistered dealer

Selling Price 7,50,000

Central sales tax (7,50,000 x 1% / 101%) 7,426

Selling Price net of Tax 7,42,574

(vii)

Purchased goods G from Delhi

Purchase Price 6,00,000

VAT (6,00,000 x 12.5% / 112.5%) 66,667

Purchase price net of Tax 5,33,333

Goods Stock transferred 6,00,000

VAT credit allowed in stock transfer (6,00,000 x 10.5% / 112.5%) 56,000

(in case of stock transfer, VAT credit shall be allowed after retaining 2%)

(viii)

Purchased goods H from Delhi

Purchase Price 8,00,000

VAT (8,00,000 x 4% / 104%) 30,769

Purchase price net of Tax 7,69,231

Input tax credit 30,769

Goods exported

Selling Price 8,50,000

Output tax Nil

Selling Price net of Tax 8,50,000

(ix)

Purchased goods I from Delhi

Purchase Price 9,00,000

VAT (9,00,000 x 12.5% / 112.5%) 1,00,000

Purchase price net of Tax 8,00,000

Input tax credit 1,00,000

Goods sold to manufacturer in SEZ

Selling Price 10,50,000

Output tax Nil

Selling Price net of Tax 10,50,000

VAT A/C

Particulars ` `

Page 22: sipoysatish@gmail · PDF file · 2013-08-07Explain the concept of Value Added Tax / Explain Taxonomy (process ... Explain accounting treatment of VAT as suggested by ICAI. ... Q26

Value Added Tax 22

OUTPUT TAX VAT CST

Goods A 5,769 ---

Goods B 27,778 ---

Goods C --- 9,314

Goods D --- 61,111

Goods E --- 3,465

Goods F --- 7,426

Goods G (Stock transfer) Not applicable ---

Goods H (Export) Nil ---

Goods I (Sale to SEZ) Nil ---

33,547 81,316

LESS: INPUT TAX CREDIT

Goods A 3,846

Goods B Not allowed

Goods C 44,444

Goods D 55,556

Goods E Not allowed

Goods F 6,931

Goods G 56,000

Goods H 30,769

Goods I 1,00,000

2,97,546

After adjusting output VAT of `33,547 and CST of `81,316, there will be unutilised VAT credit of `1,82,683

and it can be set off from other output tax or it can be carried forward or refund can be claimed but

procedure differs from State to State. At the year end it should be shown on the assets side of the balance

sheet under the head CURRENT ASSETS, LOAN AND ADVANCES.

Computation of income tax liability

Particulars

Purchases

Amount

` Particulars

Sales

Amount

` Goods A 96,154 Goods A 1,44,231

Goods B 2,00,000 Goods B 2,22,222

Goods C 3,55,556 Goods C 4,65,686

Goods D 4,44,444 Goods D 4,88,889

Goods E 3,00,000 Goods E 3,46,535

Goods F 6,93,069 Goods F 7,42,574

Goods H 7,69,231 Goods H 8,50,000

Goods I 8,00,000 Goods I 10,50,000

Net profit 6,51,683

43,10,137 43,10,137

Income under the head Business/Profession 6,51,683.00

Gross Total Income 6,51,683.00

Less: Deduction u/s 80C to 80U Nil

Total Income 6,51,683.00

Rounded off u/s 288A 6,51,680.00

Tax on `6,51,680 at slab rate 60,336.00

Add: Education cess @ 2% 1,206.72

Add: SHEC @ 1% 603.36

Page 23: sipoysatish@gmail · PDF file · 2013-08-07Explain the concept of Value Added Tax / Explain Taxonomy (process ... Explain accounting treatment of VAT as suggested by ICAI. ... Q26

Value Added Tax 23

Tax Liability 62,146.08

Rounded off u/s 288B 62,150.00

Income shall be computed exclusive of VAT because any VAT collected shall be paid to the Government

and it will not be considered to be income. Similarly VAT paid by the dealer is collected from the customer

hence it will not be considered to be expense. If any balance is left in VAT credit, it will be shown as asset

in the Balance Sheet on the last day of the relevant financial year. Further, the stock transfer of goods G is

having a neutral effect and thus ignored for calculation of business/profession income.

Illustration 8:

Mr. X is a Dealer Registered in Delhi Value Added Tax Act, 2004 and also under Central Sales Tax Act,

1956 and he has submitted the informations as given below:

(i) Purchased Goods A from Delhi `1,00,000 and paid VAT @ 4% and sold the goods in Delhi at a profit of

50% on purchase price and charged VAT @ 4%.

(ii) Purchased goods B from U.P. for `2,00,000 and paid central sales tax @ 2% and sold goods in Delhi at a

profit of 50% on purchase price and charged VAT @ 12.5%.

(iii) Purchased goods C from Delhi for `4,00,000 and paid VAT @ 12.5% and sold the goods at a profit of

50% on purchase price to a registered dealer in Orissa and charged central sales tax @ 2%

(iv) Purchased goods D for `5,00,000 in Delhi and paid VAT @ 12.5% and sold the goods at a profit of 50%

on purchase price to an unregistered dealer in Punjab and charged central sales tax @ 12.5%.

(v) Purchased goods E from Madhya Pradesh for `3,00,000 and paid central sales tax @ 1% and sold goods

at a profit of 50% on purchase price in Maharashtra and charged central sales tax @ 1%.

(vi) Purchased goods F from Delhi `7,00,000 and paid VAT @ 1% and the goods were sold at a profit of

50% on purchase price to an unregistered dealer in Maharashtra and charged central sales tax @ 1%.

(vii) Purchased goods G for `6,00,000 in Delhi and paid VAT @ 12.5% and goods were stock transferred to

some other state.

(viii) Purchased goods H for `8,00,000 in Delhi and paid VAT @ 4% and goods were exported at a profit of

50% on purchase price and no VAT was charged (because as per section 6 Central Sales Tax Act, 1956,

CST can not be charged in case of export sale.)

(ix) Purchased goods I for `9,00,000 in Delhi and paid VAT @ 12.5% and sold the goods at a profit of 50%

on purchase price to a manufacturer in SEZ and no VAT was charged.

Show the tax treatment for VAT and also compute his income tax liability for the assessment year 2013-14.

Solution: ` (i)

Purchased Goods A from Delhi 1,00,000

Add: VAT @ 4% 4,000

Purchase Price 1,04,000

Input tax credit 4,000

Cost 1,00,000

Page 24: sipoysatish@gmail · PDF file · 2013-08-07Explain the concept of Value Added Tax / Explain Taxonomy (process ... Explain accounting treatment of VAT as suggested by ICAI. ... Q26

Value Added Tax 24

Add: Profit {1,04,000-4,000(as VAT credit is available)} x 50% 50,000

Sale Price before VAT 1,50,000

Goods sold in Delhi 1,50,000

Add: VAT @ 4% 6,000

Sale Price 1,56,000

(ii)

Purchased goods B from U.P. 2,00,000

Add: Central sales tax @ 2% 4,000

Purchase Price 2,04,000

Add: Profit (2,04,000 x 50%) 1,02,000

Sale Price before VAT 3,06,000

Input tax credit Nil

Goods sold in Delhi 3,06,000

Add: VAT @ 12.5% 38,250

Sale Price 3,44,250

(iii)

Purchased goods C from Delhi 4,00,000

Add: VAT @ 12.5% 50,000

Purchase Price 4,50,000

Cost 4,00,000

Add: Profit {4,50,000-50,000(as VAT credit is available)} x 50% 2,00,000

Sale Price before CST 6,00,000

Input tax credit 50,000

Goods sold in Orissa 6,00,000

Add: Central sales tax @ 2% 12,000

Sale Price 6,12,000

(iv)

Purchased goods D from Delhi 5,00,000

Add: VAT @ 12.5% 62,500

Purchase Price 5,62,500

Cost 5,00,000

Add: Profit {5,62,500-62,500(as VAT credit is available)} x 50% 2,50,000

Sale Price before CST 7,50,000

Input tax credit 62,500

Goods sold in Punjab to unregistered dealer 7,50,000

Add: Central sales tax @ 12.5% 93,750

Sale Price 8,43,750

(v)

Purchased goods E from Madhya Pradesh 3,00,000

Add: Central sales tax @ 1% 3,000

Purchase Price 3,03,000

Add: Profit (3,03,000 x 50%) 1,51,500

Sale Price 4,54,500

Input tax credit Nil

Goods sold in Maharashtra 4,54,500

Add: Central sales tax @ 1% 4,545

Sale Price 4,59,045

Page 25: sipoysatish@gmail · PDF file · 2013-08-07Explain the concept of Value Added Tax / Explain Taxonomy (process ... Explain accounting treatment of VAT as suggested by ICAI. ... Q26

Value Added Tax 25

(vi)

Purchased goods F from Delhi 7,00,000

Add: VAT @ 1% 7,000

Purchase Price 7,07,000

Cost 7,00,000

Add: Profit {7,07,000-7,000(as VAT credit is available)} x 50% 3,50,000

Sale Price 10,50,000

Input tax credit 7,000

Goods sold in Maharashtra to unregistered dealer 10,50,000

Add: Central sales tax @ 1% 10,500

Sale Price 10,60,500

(vii)

Purchased goods G from Delhi 6,00,000

Add: VAT @ 12.5% 75,000

Purchase Price 6,75,000

Goods Stock transferred 6,00,000

VAT credit allowed in stock transfer (6,00,000 x 10.5%) 63,000

(in case of stock transfer, VAT credit shall be allowed after retaining 2%)

(viii)

Purchased goods H from Delhi 8,00,000

Add: VAT @ 4% 32,000

Purchase Price 8,32,000

Cost 8,00,000

Add: Profit {8,32,000-32,000(as VAT credit is available)} x 50% 4,00,000

Sale Price 12,00,000

Input tax credit 32,000

Goods exported 12,00,000

(ix)

Purchased goods I from Delhi 9,00,000

Add: VAT @ 12.5% 1,12,500

Purchase Price 10,12,500

Cost 9,00,000

Add: Profit {10,12,500-1,12,500(as VAT credit is available)} x 50% 4,50,000

Sale Price 13,50,000

Input tax credit 1,12,500

Goods sold to manufacturer in SEZ 13,50,000

VAT A/C

Particulars ` ` OUTPUT TAX VAT CST

Goods A 6,000 ---

Goods B 38,250 ---

Goods C --- 12,000

Goods D --- 93,750

Goods E --- 4,545

Goods F --- 10,500

Goods G (Stock transfer) Not applicable ---

Goods H (Export) Nil ---

Page 26: sipoysatish@gmail · PDF file · 2013-08-07Explain the concept of Value Added Tax / Explain Taxonomy (process ... Explain accounting treatment of VAT as suggested by ICAI. ... Q26

Value Added Tax 26

Goods I (Sale to SEZ) Nil ---

44,250 1,20,795

LESS: INPUT TAX CREDIT

Goods A 4,000

Goods B Not allowed

Goods C 50,000

Goods D 62,500

Goods E Not allowed

Goods F 7,000

Goods G 63,000

Goods H 32,000

Goods I 1,12,500

3,31,000

After adjusting output VAT of `44,250 and CST of `1,20,795, there will be unutilised VAT credit of

`1,65,955 and it can be set off from other output tax or it can be carried forward or refund can be claimed but

procedure differs from State to State. At the year end it should be shown on the assets side of the balance

sheet under the head CURRENT ASSETS, LOAN AND ADVANCES.

Computation of income tax liability

Particulars

Purchases

Amount

` Particulars

Sales

Amount

` Goods A 1,00,000 Goods A 1,50,000

Goods B 2,04,000 Goods B 3,06,000

Goods C 4,00,000 Goods C 6,00,000

Goods D 5,00,000 Goods D 7,50,000

Goods E 3,03,000 Goods E 4,54,500

Goods F 7,00,000 Goods F 10,50,000

Goods H 8,00,000 Goods H 12,00,000

Goods I 9,00,000 Goods I 13,50,000

Net profit 19,53,500

58,60,500 58,60,500

Income under the head Business/Profession 19,53,500.00

Gross Total Income 19,53,500.00

Less: Deduction u/s 80C to 80U Nil

Total Income 19,53,500.00

Tax on `19,53,500 at slab rate 4,16,050.00

Add: Education cess @ 2% 8,321.00

Add: SHEC @ 1% 4,160.50

Tax Liability 4,28,531.50

Rounded off u/s 288B 4,28,530.00

Income shall be computed exclusive of VAT & CST because any VAT & CST collected shall be paid to the

Government and it will not be considered to be income. Similarly VAT paid by the dealer is collected from

the customer hence it will not be considered to be expense. Further, the stock transfer of goods G is having a

neutral effect and thus ignored for calculation of business/profession income.

Illustration 9:

Page 27: sipoysatish@gmail · PDF file · 2013-08-07Explain the concept of Value Added Tax / Explain Taxonomy (process ... Explain accounting treatment of VAT as suggested by ICAI. ... Q26

Value Added Tax 27

Mr. X is a Dealer Registered in Delhi Value Added Tax Act, 2004 and also under Central Sales Tax Act,

1956 and he has submitted the informations as given below:

(i) Purchased Goods A from Delhi `1,00,000 and paid VAT @ 4% and sold the goods at a profit of 50% on

sale price in Delhi and charged local sales tax @ 4%.

(ii) Purchased goods B from U.P. for `2,00,000 and paid central sales tax @ 2% and sold goods at a profit of

50% on sale price in Delhi and charged VAT @ 12.5%.

(iii) Purchased goods C from Delhi for `4,00,000 and paid VAT @ 12.5% and sold the goods at a profit of

50% on sale price to a registered dealer in Orissa and charged central sales tax @ 2%

(iv) Purchased goods D for `5,00,000 in Delhi and paid VAT @ 12.5% and sold the goods at a profit of 50%

on sale price to an unregistered dealer in Punjab and charged central sales tax @ 12.5%.

(v) Purchased goods E from Madhya Pradesh for `3,00,000 and paid central sales tax @ 1% and sold goods

at a profit of 50% on sale price in Maharashtra and charged central sales tax @ 1%.

(vi) Purchased goods F from Delhi `7,00,000 and paid VAT @ 1% and the goods were sold at a profit of

50% on sale price to an unregistered dealer in Maharashtra and charged central sales tax @ 1%.

(vii) Purchased goods G for `6,00,000 in Delhi and paid VAT @ 12.5% and goods were stock transferred to

some other state.

(viii) Purchased goods H for `8,00,000 in Delhi and paid VAT @ 4% and goods were exported at a profit of

50% on sale price and no VAT was charged (because as per section 6 Central Sales Tax Act, 1956, CST can

not be charged in case of export sale.)

(ix) Purchased goods I for `9,00,000 in Delhi and paid VAT @ 12.5% and sold the goods at a profit of 50%

on sale price to a manufacturer in SEZ and no VAT was charged.

Show the tax treatment for VAT and also compute his income tax liability for the assessment year 2013-14.

Solution:

(i) ` Purchased Goods A from Delhi 1,00,000

Add: VAT @ 4% 4,000

Purchase Price 1,04,000

Cost 1,00,000

Input tax credit 4,000

Since, Profit is 50% of sale price,

Cost of `1,00,000 is 50% of Sale Price

Hence, Sale Price shall be 1,00,000 x 100% / 50% = `2,00,000

Goods sold in Delhi 2,00,000

Add: VAT @ 4% 8,000

Selling Price 2,08,000

(ii)

Purchased goods B from U.P. 2,00,000

Add: Central sales tax @ 2% 4,000

Page 28: sipoysatish@gmail · PDF file · 2013-08-07Explain the concept of Value Added Tax / Explain Taxonomy (process ... Explain accounting treatment of VAT as suggested by ICAI. ... Q26

Value Added Tax 28

Purchase Price / Cost 2,04,000

Since, Profit is 50% of sale price,

Cost of `2,04,000 is 50% of sale price

Hence, Sale Price shall be 2,04,000 x 100% / 50% = `4,08,000

Input tax credit Nil

Goods sold in Delhi 4,08,000

Add: VAT @ 12.5% 51,000

Selling Price 4,59,000

(iii)

Purchased goods C from Delhi 4,00,000

Add: VAT @ 12.5% 50,000

Purchase Price 4,50,000

Input tax credit 50,000

Since, Profit is 50% of sale price, `4,00,000 is 50% of sale price

Hence, sale price shall be 4,00,000 x 100% / 50% = `8,00,000

Goods sold in Orissa 8,00,000

Add: Central sales tax @ 2% 16,000

Selling Price 8,16,000

(iv)

Purchased goods D from Delhi 5,00,000

Add: VAT @ 12.5% 62,500

Purchase Price 5,62,500

Input tax credit 62,500

Since, Profit is 50% of sale price, `5,00,000 is 50% of sale price

Hence, Sale Price shall be 5,00,000 x 100% / 50% = 10,00,000

Goods sold in Punjab to unregistered dealer 10,00,000

Add: Central sales tax @ 12.5% 1,25,000

Selling Price 11,25,000

(v)

Purchased goods E from Madhya Pradesh 3,00,000

Add: Central sales tax @ 1% 3,000

Purchase Price / Cost 3,03,000

Input tax credit Nil

Since, Profit is 50% of sale price, `3,03,000 is 50% of sale price

Hence, sale price shall be 3,03,000 x 100% / 50% = `6,06,000

Goods sold in Maharashtra 6,06,000

Add: Central sales tax @ 1% 6,060

Selling Price 6,12,060

(vi)

Purchased goods F from Delhi 7,00,000

Add: VAT @ 1% 7,000

Purchase Price 7,07,000

Input tax credit 7,000

Since, Profit is 50% of sale price, `7,00,000 is 50% of sale price

Hence, Sale Price shall be 7,00,000 x 100% / 50% = `14,00,000

Goods sold in Maharashtra to unregistered dealer 14,00,000

Page 29: sipoysatish@gmail · PDF file · 2013-08-07Explain the concept of Value Added Tax / Explain Taxonomy (process ... Explain accounting treatment of VAT as suggested by ICAI. ... Q26

Value Added Tax 29

Add: Central sales tax @ 1% 14,000

Selling Price 14,14,000

(vii)

Purchased goods G from Delhi 6,00,000

Add: VAT @ 12.5% 75,000

Purchase Price 6,75,000

Goods Stock transferred 6,00,000

VAT credit allowed in stock transfer (6,00,000 x 10.5%) 63,000

(in case of stock transfer, VAT credit shall be allowed after retaining 2%)

(viii)

Purchased goods H from Delhi 8,00,000

Add: VAT @ 4% 32,000

Purchase Price 8,32,000

Input tax credit 32,000

Since, Profit is 50% of Sale Price, `8,00,000 is 50% of sale price

Hence, Sale Price shall be 8,00,000 x 100% / 50% = `16,00,000

Goods exported 16,00,000

(ix)

Purchased goods I from Delhi 9,00,000

Add: VAT @ 12.5% 1,12,500

Purchase Price 10,12,500

Input tax credit 1,12,500

Since, Profit is 50% of sale price, `9,00,000 is 50% of sale price

Hence, Sale Price shall be 9,00,000 x 100% / 50% = `18,00,000

Goods sold to manufacturer in SEZ 18,00,000

VAT A/C

Particulars ` ` OUTPUT TAX VAT CST

Goods A 8,000 ---

Goods B 51,000 ---

Goods C --- 16,000

Goods D --- 1,25,000

Goods E --- 6,060

Goods F --- 14,000

Goods G (Stock transfer) Not applicable ---

Goods H (Export) Nil ---

Goods I (Sale to SEZ) Nil ---

59,000 1,61,060

LESS: INPUT TAX CREDIT

Goods A 4,000

Goods B Not allowed

Goods C 50,000

Goods D 62,500

Goods E Not allowed

Goods F 7,000

Goods G 63,000

Goods H 32,000

Page 30: sipoysatish@gmail · PDF file · 2013-08-07Explain the concept of Value Added Tax / Explain Taxonomy (process ... Explain accounting treatment of VAT as suggested by ICAI. ... Q26

Value Added Tax 30

Goods I 1,12,500

3,31,000

After adjusting output VAT of `59,000 and CST of `1,61,060, there will be unutilised VAT credit of

`1,10,940 and it can be set off from other output tax or it can be carried forward or refund can be claimed but

procedure differs from State to State. At the year end it should be shown on the assets side of the balance

sheet under the head CURRENT ASSETS, LOAN AND ADVANCES.

Computation of income tax liability

Particulars

Purchases

Amount

` Particulars

Sales

Amount

` Goods A 1,00,000 Goods A 2,00,000

Goods B 2,04,000 Goods B 4,08,000

Goods C 4,00,000 Goods C 8,00,000

Goods D 5,00,000 Goods D 10,00,000

Goods E 3,03,000 Goods E 6,06,000

Goods F 7,00,000 Goods F 14,00,000

Goods H 8,00,000 Goods H 16,00,000

Goods I 9,00,000 Goods I 18,00,000

Net profit 39,07,000

78,14,000 78,14,000

Income under the head Business/Profession 39,07,000.00

Gross Total Income 39,07,000.00

Less: Deduction u/s 80C to 80U Nil

Total Income 39,07,000.00

Tax on `39,07,000 at slab rate 10,02,100.00

Add: Education cess @ 2% 20,042.00

Add: SHEC @ 1% 10,021.00

Tax Liability 10,32,163.00

Rounded off u/s 288B 10,32,160.00

Income shall be computed exclusive of VAT because any VAT collected shall be paid to the Government

and it will not be considered to be income. Similarly VAT paid by the dealer is collected from the customer

hence it will not be considered to be expense. Further, the stock transfer of goods G is having a neutral effect

and thus ignored for calculation of business/profession income.

Question 9 (V. Imp.): Explain provisions of Stock Transfer.

Answer:

Inter-State branch transfers do not involve sale and, therefore they are not subjected to sales-tax. The same

position continues under VAT.

However, the tax paid on:

(i) inputs used in the manufacture of finished goods which are stock transferred: or

(ii) purchases of goods which are stock transferred

will be available as input tax credit after retention of 2% of such tax by the State Government.

Page 31: sipoysatish@gmail · PDF file · 2013-08-07Explain the concept of Value Added Tax / Explain Taxonomy (process ... Explain accounting treatment of VAT as suggested by ICAI. ... Q26

Value Added Tax 31

e.g. ABC Ltd. has purchased goods for `10 lakhs and has paid VAT @ 12.5% in Delhi and subsequently the

goods were stock transferred to a branch in some other State, in this case VAT credit allowed shall be

`1,05,000 (`10,00,000 x (12.5% - 2%)).

Illustration 10: A dealer purchased 16,500 kgs of inputs on which VAT paid @ 4% was `6,000. He

manufactured 15,000 Kgs of finished products from the inputs. 1,500 Kgs was the process loss. The final

product was sold at a price of `10 per Kg, as follows:

- Goods sold within State – 6,000 Kgs.

- Finished products sold in inter-state sale against C form – 3,750 Kgs.

- Goods sent on stock transfer to consignment agents outside the state – 3,000 Kgs.

- Goods sold to Government department outside the state – 2,250 Kgs.

There was no opening or closing stock of inputs, WIP or finished product.

The State VAT rate on the finished product of dealer is 12.5%.

Discuss tax treatment.

Solution:

CST against C form is 2%.

Sale to Government will be treated as sale to unregistered dealer and tax payable is 12.5%. Thus, the tax

payable would be as follows-

Output tax shall be as given below:

Description Quantity

sold

Kg

Value of

goods sold

`

CST

payable

`

State VAT

payable

` Sale within State @ 12.5% 6,000 60,000 7,500

Goods sent on stock transfer 3,000 30,000

Goods sold against C form, tax rate 2% 3,750 37,500 750

Goods sold to Government, tax rate 12.5% 2,250 22,500 2,813

Total 15,000 1,50,000 3,563 7,500

Input tax credit shall be = 6,000 – 600* = 5,400

` (i) State VAT 7,500

Less: VAT credit 5,400

Net State VAT payable 2,100

CST payable 3,563

Total tax payable 5,663

*In the given case total finished product is 15,000 kgs and stock transferred 3,000 kgs and proportionate

VAT credit is `1,200 (6,000 / 15,000 x 3,000) @ 4% but in case of stock transfer VAT credit shall be

Page 32: sipoysatish@gmail · PDF file · 2013-08-07Explain the concept of Value Added Tax / Explain Taxonomy (process ... Explain accounting treatment of VAT as suggested by ICAI. ... Q26

Value Added Tax 32

allowed after deducting 2% i.e. VAT credit shall be allowed for `600 instead of `1,200 i.e. VAT credit for

`600 shall not be allowed and total VAT credit allowed shall be (`6,000 – 600 = `5,400)

(if raw material is consumed in process, VAT credit is allowed even for such raw material i.e. VAT credit

even for process loss is allowed)

In aforesaid example, if 3,000 Kgs were exported (and not stock transferred), what would be the tax

liability and credit available.

If finished product is exported. There is no tax liability. Further, the credit of tax paid on raw material is

available.

` State VAT payable 7,500

Less: VAT credit 6,000

Net State VAT payable 1,500

CST payable 3,563

Total tax payable 5,063

Question 10: Explain accounting treatment of VAT as suggested by ICAI. (not covered in syllabus

rather it is only for self reading)

Answer:

VAT credit in case of inputs / supplies

1. A dealer purchases the following goods in a State during the month of March 2013:

Particulars Net Amount

(`) Input Tax Paid

(`) Total Amount

(`) 4% VAT Goods 10,00,000 40,000 10,40,000

12.5% VAT Goods 8,00,000 1,00,000 9,00,000

VAT Exempt Goods 2,00,000 - 2,00,000

Total 20,00,000 1,40,000 21,40,000

2. The input tax paid on purchase of goods is eligible for VAT credit.

3. Sales made by the dealer during the month are as below:

Particulars Net Sales

Consideration (`) Output Tax

Collected (`) Gross Amount

(`) 4% VAT Goods 11,00,000 44,000 11,44,000

12.5% VAT Goods 9,00,000 1,12,500 10,12,500

VAT Exempt Goods 2,50,000 - 2,50,000

Total 22,50,000 1,56,500 24,06,500

Suggested Accounting Treatment

1. The dealer passes the following entry to record the goods purchased and input tax paid thereon:

4% VAT Goods Purchase A/c Dr. `10,00,000

12.5% VAT Goods Purchase A/c Dr. ` 8,00,000

VAT Exempt Goods Purchase A/c Dr. ` 2,00,000

VAT Credit Receivable (Inputs) A/c Dr. ` 1,40,000

To Bank A/c ` 21,40,000

2. The dealer passes the following entry to record the goods sold and VAT collected thereon:

Bank A/c Dr. `24,06,500

Page 33: sipoysatish@gmail · PDF file · 2013-08-07Explain the concept of Value Added Tax / Explain Taxonomy (process ... Explain accounting treatment of VAT as suggested by ICAI. ... Q26

Value Added Tax 33

To 4% VAT Goods Sales A/c `11,00,000

To 12.5% VAT Goods Sales A/c ` 9,00,000

To VAT Exempt Goods Sales A/c ` 2,50,000

To VAT Payable A/c ` 1,56,500

3. The dealer passes the following entry to record the liability for VAT payable met by using the balance in

the VAT Credit Receivable (Inputs) Account and balance by bank:

VAT Payable A/c Dr. ` 1,56,500

To VAT Credit Receivable (Inputs) A/c ` 1,40,000

To Bank ` 16,500

VAT credit in case of capital goods

Example

On July 1, 2012 a dealer purchases one machine in a State for the total cost of `93,60,000 which includes

input tax of `3,60,000. As per the State VAT laws, input tax paid on purchases of machinery is adjustable as

VAT credit over 3 annual installments. Till the end of the year, the dealer has not utilized the VAT credit

available on the machine.

Suggested Accounting Treatment

1. The dealer passes the following entry to record the machinery purchased and input tax paid thereon:

Machinery A/c Dr. ` 90,00,000

VAT Credit Receivable (Capital Goods) A/c Dr. ` 1,20,000

VAT Credit Deferred (Capital Goods) A/c Dr. ` 2,40,000

To Bank A/c ` 93,60,000

In the subsequent year the dealer will transfer the amount from VAT Credit Deferred A/c to VAT Credit

Receivable A/c

2. The dealer charges depreciation on the cost of machinery excluding VAT credit (i.e. `93,60,000 –

`3,60,000 = `90,00,000).

3. Balances in VAT Credit Deferred (Capital Goods) A/c and VAT credit Receivable (Capital Goods) A/c

are disclosed in the balances sheet as on March 31, 2013 as below:

Extracts from the Balance Sheet

Assets Amounts (`) Current Assets

Loans and Advances

VAT Credit Deferred (Capital Goods) A/c 2,40,000

VAT Credit Receivable (Capital Goods) A/c 1,20,000

Question 11 (V. Imp.): What are the Variants (different types) of VAT.

Answer:

VAT has 3 variants:

(a) Gross Product Variant

(b) Income Variant

(c) Consumption Variant

(a) Gross Product Variant

Page 34: sipoysatish@gmail · PDF file · 2013-08-07Explain the concept of Value Added Tax / Explain Taxonomy (process ... Explain accounting treatment of VAT as suggested by ICAI. ... Q26

Value Added Tax 34

Gross Product Variant allows VAT credit on the raw materials, but tax credit is not allowed on capital goods

like plant and machinery etc.

(b) Income Variant of VAT The Income Variant of VAT allows VAT credit on raw materials etc and also on capital goods but VAT

credit on capital goods is allowed in instalments depending on the life of capital goods.

(c) Consumption Variant

It allows VAT credit on raw materials etc. and also on capital goods in the very first year.

Among the three variants of VAT, the consumption variant is widely used. Several countries of Europe and

other continents have adopted this variant, because there is no multiple taxation and also there is no

cascading effect and it can be shown as given below:

Mr. X manufactures product A out of raw material X. The cost of raw material X is `2 lakh. The labour and

other manufacturing costs are `8 lakh. The manufacturing process requires a machinery of `20 lakh (subject

to VAT @ 12.5%). The useful life of the plant is 4 years with no salvage and rate of depreciation is 25% on

straight line method. The expected output of product A is 1,000 units p.a. Mr. X fixes a profit margin of

`100 per unit.

Compute the selling price of product A and its cost to consumer if –

(a) No credit is allowed on the capital goods (Gross product variant);

(b) credit is allowed on the capital goods (Consumption variant).

The VAT rate on final product is 12.5%. There is no VAT on raw material.

Solution:

(a) No credit on capital goods - Gross Product Variant ` Raw material cost 2,00,000.00

Labour and other manufacturing costs 8,00,000.00

Depreciation on machinery:

(20 lakh + 12.5% VAT) 4 (as VAT credit is not available)

5,62,500.00

Total cost 15,62,500.00

Cost per unit 1,562.50

Profit per unit 100.00

Selling price per unit 1,662.50

Add: VAT @ 12.5% 207.81

Cost to consumer 1,870.31

(b) Credit available on capital goods- Consumption Variant

` Raw material cost 2,00,000.00

Labour and other manufacturing costs 8,00,000.00

Depreciation on machinery:

(20 lakh 4) (VAT credit available)

5,00,000.00

Total cost 15,00,000.00

Cost per unit 1,500.00

Profit per unit 100.00

Selling price per unit 1,600.00

Add: VAT @ 12.5% 200.00

Page 35: sipoysatish@gmail · PDF file · 2013-08-07Explain the concept of Value Added Tax / Explain Taxonomy (process ... Explain accounting treatment of VAT as suggested by ICAI. ... Q26

Value Added Tax 35

Cost to consumer 1,800.00

The availability of VAT credit on capital goods, reduces the cost and resultant selling price of the

goods and, therefore, eliminates cascading effect.

Illustration 11: A manufacturer has purchased raw material for `2,08,000 (inclusive of 4% VAT) and plant

and machinery of `4,50,000 (inclusive of 12.5% VAT).

The manufacturing and other expenses (excluding depreciation) are `6,00,000.

He sells the resultant products at 50% above cost (VAT on sales is 4%).

The plant and machinery is to be depreciated at 50% straight line.

(a) Compute the amount of VAT payable, as per the Gross Product Variant of VAT.

(b) Compute VAT payable as per Income Variant of VAT.

(c) Compute VAT payable as per Consumption Variant of VAT.

Solution (a):

Gross Product Variant ` Raw material net of VAT (2,08,000 x 100 ÷ 104) 2,00,000

Depreciation of plant and machinery (50% of `4,50,000 – VAT credit not allowed) 2,25,000

Manufacturing and other expenses 6,00,000

Total cost 10,25,000

Add: Profit 50% 5,12,500

Sale price 15,37,500

VAT on sales (4% of 15,37,500) 61,500

Less: Input tax credit on raw material (2,08,000 x 4 ÷ 104) 8,000

VAT payable 53,500

Solution (b):

Income Variant ` Raw material net of VAT (2,08,000 x 100 ÷ 104) 2,00,000

Depreciation on plant and machinery (50% of `4,00,000 VAT credit allowed) 2,00,000

Manufacturing and other expenses 6,00,000

Total cost 10,00,000

Add: Profit @ 50% 5,00,000

Sale price 15,00,000

VAT on sales (4% of 15,00,000) 60,000

Less: Input tax credit as follows:

Input tax credit on raw material (2,08,000 x 4 ÷ 104) 8,000

Input tax credit on plant (50% of 50,000) 25,000 33,000

VAT payable 27,000

Note: The VAT paid on plant and machinery has been allowed as credit only to the extent of depreciation

i.e. 50%. The balance VAT credit of `25,000 can be set-off in subsequent year.

Solution (c):

Consumption Variant `

Page 36: sipoysatish@gmail · PDF file · 2013-08-07Explain the concept of Value Added Tax / Explain Taxonomy (process ... Explain accounting treatment of VAT as suggested by ICAI. ... Q26

Value Added Tax 36

Raw material net of VAT (2,08,000 x 100 ÷ 104) 2,00,000

Depreciation on plant and machinery (50% of `4,00,000 VAT credit is allowed) 2,00,000

Manufacturing and other expenses 6,00,000

Total cost 10,00,000

Add: Profit 50% 5,00,000

Sale price 15,00,000

VAT on sales (4% of 15,00,000) 60,000

Less: Input tax credit as follows:

Input tax credit on raw material and components (2,08,000 x 4 ÷ 104) 8,000

Input tax credit on plant (100% of 50,000) 50,000 58,000

VAT payable 2,000

Question 12: What are the rates under VAT.

Answer:

Exempted Category

Under exempted category, there are about 50 commodities comprising of natural and unprocessed products

in unorganized sector, items which are legally barred from taxation and items which have social

implications. Included in this exempted category is a set of maximum of 10 commodities flexibly chosen by

individual States from a list of goods (finalized by the Empowered Committee) which are of local social

importance for the individual States without having any inter-State implication.

Example:

(i) Books, periodicals and journals including maps, charts and globes.;(ii) Curd, Lussi, butter milk and

separated milk.; (iii) Earthen pot.; (iv) Electricity energy; (v) Fresh plants, saplings and fresh flowers.; (vi)

Fresh vegetables and fruits.; (vii) All bangles except those made of precious metals.; (viii) Kumkum, bindi

alta and sindur.; (ix) Blood including blood components

1% Category

The special rate of 1% is meant for precious stones, bullion, gold and silver ornaments etc.

4% VAT Category

The goods declared as per section 14 of Central Sales Tax Act shall be taxable @ 4%

List of some of the goods is:

(i) Coal including coke in all its forms, but excluding charcoal, (ii) Cotton yarn, but not including cotton

yarn waste; (iii) Oil seeds; (iv) Pulses; (v) Sugar; (vi) Iron and steel; (vii) Liquefied petroleum gas for

domestic use

5% VAT Category

Under 5% VAT rate category, there are largest number of goods, common for all the States, comprising of

items of basic necessities such as medicines and drugs, all agricultural and industrial inputs, capital goods

and declared goods. The schedule of commodities are attached to the VAT Acts of the States.

List of some of the goods is:

1. Bicycles having MRP above `3,500, Tricycles, rickshaws and parts including tyres and tubes thereof; 2.

Drugs & medicines including vaccines, syringes and dressings, medicated ointments produced under a drugs

licence; 3. Coffee beans and seeds, green tea leaf and chicory.; 4. Cotton and cotton waste; 5. Edible oils and

oil cake.

20% Category

Page 37: sipoysatish@gmail · PDF file · 2013-08-07Explain the concept of Value Added Tax / Explain Taxonomy (process ... Explain accounting treatment of VAT as suggested by ICAI. ... Q26

Value Added Tax 37

Petrol, diesel, Aviation Turbine Fuel, other motor spirit, liquor and lottery tickets etc. will be subjected to

20% floor rate of tax.

12.5% Category

The remaining commodities, common for all the States, fall under the general VAT rate of 12.5%.

Question 13: Explain non- creditable/non-vatable goods.

Answer:

Description of Non- Creditable/Non-Vatable goods

No VAT credit is allowed in case of some of the commodities because these goods are either not meant for

resale or the goods are allowed to be sold only once and such goods are called non-creditable or non-vatable

goods and the list of such goods differs from State to State and some of such items are as given below:

(i) All automobiles including commercial vehicles, and two and three wheelers, and spare parts for repairs

and maintenance and tyres and tube thereof;

(ii) Fuels in the form of petrol, diesel, kerosene, LPG, CNG, PNG and coal;

(iii) Food for human consumption;

(iv) Beverages for human consumption;

(v) Air conditioners, air conditioning plants or units other than those used for manufacturing purposes; air

coolers, fans and air circulators;

(vi) Goods for personal consumption or for gifts

Question 14 (V. Imp.): Explain concept of excise duty (Central Value Added Tax).

Answer:

Central Excise Duty also called Central Value Added Tax is regulated through the following Acts/ Rules.

- Central Excise Act, 1944 : It is the principal Act governing Central Excise Duty

- Central Excise Rules, 2002: It contains procedure for collection of Central Excise Duty.

- Central Excise Tariff Act, 1985 : It contains the rates of Central Excise Duty

- Cenvat Credit Rules, 2004: It Contains the Rules for tax Credit including Service Tax and Custom

Duty.

Every manufacturer has to pay excise duty on the goods manufactured by him.

If excise duty is payable, it is computed on the total of cost plus profit and it is called assessable value as per

section 4 of Central Excise Act, 1944. EC and SHEC are also payable on excise duty.

VAT is charged on the total of cost plus profit plus excise duty plus EC and SHEC of excise duty.

Example

If cost is `5 lakhs, profit 1 lakh and excise duty rate is 10% and VAT rate is 12.5%, output tax shall be

computed in the manner given below: ` Cost 5,00,000.00

Add: Profit 1,00,000.00

Total (it is called assessable value as per section 4 of Central Excise Act, 1944) 6,00,000.00

Excise Duty @ 10% 60,000.00

Add: Education Cess @ 3% 1,800.00

Total 6,61,800.00

Add: VAT @ 12.5% 82,725.00

Page 38: sipoysatish@gmail · PDF file · 2013-08-07Explain the concept of Value Added Tax / Explain Taxonomy (process ... Explain accounting treatment of VAT as suggested by ICAI. ... Q26

Value Added Tax 38

Certain concessions have been given under Notification No. 8/2003 dated 01.03.2003 and accordingly the

unit having a turnover upto `400 lakhs in the immediately preceding year shall be exempt from excise duty

but only upto the turnover of `150 lakhs and excess over it shall be chargeable to excise duty.

If turnover in the immediately preceding year is exceeding `400 lakhs, the unit has to pay excise duty from

the beginning.

A unit with turnover upto `400 lakhs is called Small Scale Industry – SSI.

Example

ABC Ltd. has turnover in F.Y. 2011-12 `300 lakhs and turnover in F.Y. 2012-13 `350 lakhs and if the rate of

excise duty is 10%. Excise duty payable in F.Y. 2012-13 shall be

Lakhs

Total Turnover `350.00

Less: Exemption under Notification No. 8/2003 `150.00

Amount chargeable to Excise Duty `200.00

Excise duty @ 10% ` 20.00

Add: Education cess @ 3% ` 0.60

Excise Duty + EC ` 20.60

If turnover in F.Y. 2011-12 is `450 lakhs, excise duty shall be charged in F.Y. 2012-13 on entire amount of

`350 lakhs.

If a unit is availing exemption upto `150 lakhs under Notification No. 8/2003, Cenvat credit shall not be

allowed for the inputs used in the manufacturing of final product which is covered within the turnover of

`150 lakhs and tax credit shall be allowed for the inputs which will be used in manufacturing of final product

which will be covered in the turnover in excess of `150 lakhs.

As a special case, Cenvat credit shall be allowed for the capital goods even if capital goods have been used

in manufacturing of final products which is covered in the turnover of `150 lakhs.

Question 15 (V. Imp): Write a note on tax credit in case of manufacturer.

Answer:

A Manufacturer shall be allowed tax credit for the following tax paid by him.

1. Excise duty on inputs (raw material etc) or capital goods (plant and machinery, furniture and fixtures

etc) used in or in connection with the goods to be manufactured by him.

2. Service Tax paid on the input services taken by him in connection with the goods to be manufactured by

him and such services may be insurance services, banking services, warehousing, goods transport,

renting of immovable property or other similar services.

3. Education cess and SHEC paid on Excise Duty or Service Tax shall also be eligible for tax credit.

4. VAT paid by him on raw materials or capital goods used in or in connection with the goods to be

manufactured by him.

The manufacturer can utilize the tax credit in the manner given below:

1. Tax credit for Excise Duty or Service Tax can be utilized against output excise duty.

Page 39: sipoysatish@gmail · PDF file · 2013-08-07Explain the concept of Value Added Tax / Explain Taxonomy (process ... Explain accounting treatment of VAT as suggested by ICAI. ... Q26

Value Added Tax 39

2. Tax credit for Education cess of 2% can be utilized for payment of education cess of 2% on output

excise duty.

3. Tax credit for SHEC of 1% can be utilized for payment of SHEC of 1% on output excise duty.

4. Tax credit for VAT (Sales Tax) can be utilized for payment of output VAT on sale of manufactured

goods.

Inter-adjustment of Tax credit of Excise Duty, Service Tax is allowed to the manufacturer because all these

taxes are collected by Central Government but inter-adjustment with VAT is not allowed because VAT is

collected by State Government.

Tax credit for Excise Duty, Service Tax is called Cenvat Credit and is regulated through Cenvat Credit

Rules, 2004.

Question 16 (V. Imp): What is the common procedure for availing and adjusting cenvat credit for

Excise Duty, Service Tax as per Cenvat Credit Rules, 2004.

Answer:

Tax credit for Excise Duty, Service Tax is called Cenvat Credit and is regulated through Cenvat Credit

Rules, 2004.

Inter-adjustment of Tax credit of Excise Duty, Service Tax is allowed to the manufacturer because all these

taxes are collected by Central Government but inter-adjustment with VAT is not allowed because VAT is

collected by State Government.

The common rules are as given below:

1. The CENVAT credit in respect of inputs may be taken immediately on receipt of the inputs in the factory

of the manufacturer or in the premises of the provider of output service:

2. The CENVAT credit in respect of capital goods received in a factory or in the premises of the provider of

output service, at any point of time in a given financial year shall be taken only for an amount not exceeding

fifty per cent of the duty paid on such capital goods in the same financial year:

The balance of CENVAT credit may be taken in any financial year subsequent to the financial year in which

the capital goods were received in the factory of the manufacturer, or in the premises of the provider of

output service, if the capital goods, are in the possession of the manufacturer of final products, or provider of

output service in such subsequent years.

Illustration.—A manufacturer received machinery on the 16th day of April, 2012 in his factory. CENVAT of

two lakh rupees is paid on this machinery. The manufacturer can take credit upto a maximum of one lakh

rupees in the financial year 2012-13, and the balance in subsequent years.

3. In case of SSI unit availing exemption upto `150 lakhs, CENVAT credit for the capital goods shall be

allowed in the same Financial Year.

4. While paying duty of excise or service tax, as the case may be, the CENVAT credit shall be utilized only

to the extent such credit is available on the last day of the month or quarter, as the case may be, for payment

of duty or tax relating to that month or the quarter, as the case may be:

Page 40: sipoysatish@gmail · PDF file · 2013-08-07Explain the concept of Value Added Tax / Explain Taxonomy (process ... Explain accounting treatment of VAT as suggested by ICAI. ... Q26

Value Added Tax 40

Illustration 12: Mr. X is a manufacturer and he is registered under Central Excise, Central Sales Tax and

also under Delhi VAT.

He has purchased raw material R1 from Delhi and R2 from Punjab and R3 has been imported from outside

India and purchase price is `1,50,000, `3,80,000 and `7,20,000 respectively and excise duty rate is 10% for

raw material R1 and R2 and custom duty for R3 is 10% plus EC plus SHEC. Delhi VAT paid on raw

material R1 is 12.5% and Central Sales Tax paid on raw material R2 is 2%.

He has purchased plant and machinery for `18,00,000 from Delhi and paid excise duty @ 10% and VAT @

12.5% and life of plant is 10 years and rate of depreciation is 10% on straight line method.

Half of the raw material R1 was stock transferred to U.P. branch.

Tax credit on plant and machinery for excise duty and VAT is allowed in first year for 100%. (consumption

variant)

Amount paid for the services taken in connection with manufacturing of goods is `4,00,000 plus service tax

@ 12% plus EC @ 3%.

Other processing expenditure is `8,50,000 and profit margin is `3,10,000 and goods were sold in the state of

Delhi and excise duty is charged @ 10% plus EC plus SHEC and Delhi VAT charged is @ 12.5%.

Show the tax treatment.

(b) Tax credit for excise duty and DVAT for plant machinery is allowed in 5 installments. (income variant)

(c) Presume in the above question no tax credit is allowed for plant and machinery. (gross product variant)

(d) CENVAT Credit on plant and machinery is allowed in 2 yearly installments and Delhi VAT is allowed

in 3 yearly installments.

Solution(a):

Raw material – R1 Used in Delhi Stock transferred

` ` Assessable value 75,000 75,000

Excise duty @ 10% 7,500 7,500

EC @ 2% 150 150

SHEC@ 1% 75 75

Total 82,725 82,725

Delhi VAT @ 12.5% 10,341 10,341

Since half of the stock has been transferred to some other state, VAT credit allowed for such stock transfer

shall be 10.5% (12.5% - 2%) i.e. 82,725 x 10.5% = 8,686

` Raw material – R2

Assessable value 3,80,000

Excise duty @ 10% 38,000

EC @ 2% 760

SHEC@ 1% 380

Total 4,19,140

Central Sales tax @ 2% 8,383

Raw material – R3

Page 41: sipoysatish@gmail · PDF file · 2013-08-07Explain the concept of Value Added Tax / Explain Taxonomy (process ... Explain accounting treatment of VAT as suggested by ICAI. ... Q26

Value Added Tax 41

Assessable value 7,20,000

Custom duty @ 10% 72,000

EC @ 2% 1,440

SHEC @ 1% 720

7,94,160

Plant and machinery

Purchase price 18,00,000

Add: Excise duty @ 10% 1,80,000

EC @ 2% 3,600

SHEC@ 1% 1,800

19,85,400

Delhi VAT @ 12.5% 2,48,175

22,33,575

Cost of Final Product

Raw material - R1 75,000

Raw material - R2 3,88,383

Raw material - R3 7,94,160

Plant & Machinery (10% of `18,00,000) 1,80,000

Amount paid for services 4,00,000

Processing charges 8,50,000

Profit 3,10,000

Assessable value (as per section 4 of Central Excise Act, 1944) 29,97,543

Excise duty @ 10% 2,99,754

EC @ 2% 5,995

SHEC@ 1% 2,998

Total 33,06,290

Delhi VAT @ 12.5% - Output Tax 4,13,286

Selling Price 37,19,576

Computation of Net Tax Payable

Excise Duty / Service Tax /

` EC @ 2%

` SHEC @

1%

`

Delhi VAT

`

Output tax 2,99,754 5,995 2,998 4,13,286

Less: VAT / CENVAT

Credit

Raw material – R1 (7,500)

(7,500)

(150)

(150)

(75)

(75)

(10,341)

(8,686)

Raw material – R2 (38,000) (760) (380) -

Raw material – R3 - - - -

Plant and machinery (1,80,000) (3,600) (1,800) (2,48,175)

Service tax (48,000) (960) (480) -

Net tax payable 18,754 375 188 1,46,084

Solution(b):

Raw material – R1 Used in Delhi Stock transferred

Assessable value 75,000 75,000

Excise duty @ 10% 7,500 7,500

EC @ 2% 150 150

SHEC@ 1% 75 75

Page 42: sipoysatish@gmail · PDF file · 2013-08-07Explain the concept of Value Added Tax / Explain Taxonomy (process ... Explain accounting treatment of VAT as suggested by ICAI. ... Q26

Value Added Tax 42

Total 82,725 82,725

Delhi VAT @ 12.5% 10,341 10,341

Since half of the stock has been transferred to some other state, VAT credit allowed for such stock transfer

shall be 10.5% (12.5% - 2%) i.e. 82,725 x 10.5% = 8,686

` Raw material – R2

Assessable value 3,80,000

Excise duty @ 10% 38,000

EC @ 2% 760

SHEC @ 1% 380

Total 4,19,140

CST @ 2% 8,383

Raw material – R3

Assessable value 7,20,000

Custom duty @ 10% 72,000

EC @ 2% 1,440

SHEC @ 1% 720

7,94,160

Plant and machinery

Purchase price 18,00,000

Add: Excise duty @ 10% 1,80,000

EC @ 2% 3,600

SHEC@ 1% 1,800

19,85,400

Delhi VAT @ 12.5% 2,48,175

22,33,575

Cost of Final Product

Raw material - R1 75,000

Raw material - R2 3,88,383

Raw material - R3 7,94,160

Plant & Machinery (10% of `18,00,000) 1,80,000

Amount paid for services 4,00,000

Processing charges 8,50,000

Profit 3,10,000

Assessable value (as per section 4 of Central Excise Act, 1944) 29,97,543

Excise duty @ 10% 2,99,754

EC @ 2% 5,995

SHEC@ 1% 2,998

Total 33,06,290

Delhi VAT @ 12.5% - Output Tax 4,13,286

Selling Price 37,19,576

Computation of Net Tax Payable

Excise Duty / Service Tax /

`

EC @ 2%

` SHEC @

1%

`

Delhi VAT

`

Output tax 2,99,754 5,995 2,998 4,13,286

Page 43: sipoysatish@gmail · PDF file · 2013-08-07Explain the concept of Value Added Tax / Explain Taxonomy (process ... Explain accounting treatment of VAT as suggested by ICAI. ... Q26

Value Added Tax 43

Less: VAT / CENVAT

Credit

Raw material – R1 (7,500)

(7,500)

(150)

(150)

(75)

(75)

(10,341)

(8,686)

Raw material – R2 (38,000) (760) (380) -

Raw material – R3 - - - -

Plant and machinery (36,000) (720) (360) (49,635)

Service tax (48,000) (960) (480) -

Net tax payable 1,62,754 3,255 1,628 3,44,624

Solution(c):

Raw material – R1 Used in Delhi Stock transferred

Assessable value 75,000 75,000

Excise duty @ 10% 7,500 7,500

EC @ 2% 150 150

SHEC@ 1% 75 75

Total 82,725 82,725

Delhi VAT @ 12.5% 10,341 10,341

Since half of the stock has been transferred to some other state, VAT credit allowed for such stock transfer

shall be 10.5% (12.5% - 2%) i.e. 82,725 x 10.5% = 8,686

` Raw material – R2

Assessable value 3,80,000

Excise duty @ 10% 38,000

EC @ 2% 760

SHEC@ 1% 380

Total 4,19,140

CST @ 2% 8,383

Raw material – R3

Assessable value 7,20,000

Custom duty @ 10% 72,000

EC @ 2% 1,440

SHEC @ 1% 720

7,94,160

Plant and machinery

Purchase price 18,00,000

Add: Excise duty @ 10% 1,80,000

EC @ 2% 3,600

SHEC@ 1% 1,800

19,85,400

Delhi VAT @ 12.5% 2,48,175

22,33,575

Cost of Final Product

Raw material - R1 75,000

Raw material - R2 3,88,383

Raw material - R3 7,94,160

Plant & Machinery (10% of 22,33,575) 2,23,358

Amount paid for services 4,00,000

Processing charges 8,50,000

Profit 3,10,000

Page 44: sipoysatish@gmail · PDF file · 2013-08-07Explain the concept of Value Added Tax / Explain Taxonomy (process ... Explain accounting treatment of VAT as suggested by ICAI. ... Q26

Value Added Tax 44

Assessable value (as per section 4 of Central Excise Act, 1944) 30,40,901

Excise duty @ 10% 3,04,090

EC @ 2% 6,082

SHEC@ 1% 3,041

Total 33,54,114

Delhi VAT @ 12.5% - Output Tax 4,19,264

Selling Price 37,73,378

Computation of Net Tax Payable

Excise Duty / Service Tax /

`

EC @ 2%

` SHEC @

1%

`

Delhi VAT

`

Output tax 3,04,090 6,082 3,041 4,19,264

Less: VAT / CENVAT

Credit

Raw material – R1 (7,500)

(7,500)

(150)

(150)

(75)

(75)

(10,341)

(8,686)

Raw material – R2 (38,000) (760) (380) -

Raw material – R3 - - - -

Plant and machinery - - - -

Service tax (48,000) (960) (480) -

Net tax payable 2,03,090 4,062 2,031 4,00,237

Solution (d):

Raw material – R1 Used in Delhi Stock transferred

Assessable value 75,000 75,000

Excise duty @ 10% 7,500 7,500

EC @ 2% 150 150

SHEC@ 1% 75 75

Total 82,725 82,725

Delhi VAT @ 12.5% 10,341 10,341

Since half of the stock has been transferred to some other state, VAT credit allowed for such stock transfer

shall be 10.5% (12.5% - 2%) i.e. 82,725 x 10.5% = 8,686

` Raw material – R2

Assessable value 3,80,000

Excise duty @ 10% 38,000

EC @ 2% 760

SHEC@ 1% 380

Total 4,19,140

CST @ 2% 8,383

Raw material – R3

Assessable value 7,20,000

Custom duty @ 10% 72,000

EC @ 2% 1,440

SHEC @ 1% 720

7,94,160

Plant and machinery

Purchase price 18,00,000

Page 45: sipoysatish@gmail · PDF file · 2013-08-07Explain the concept of Value Added Tax / Explain Taxonomy (process ... Explain accounting treatment of VAT as suggested by ICAI. ... Q26

Value Added Tax 45

Add: Excise duty @ 10% 1,80,000

EC @ 2% 3,600

SHEC@ 1% 1,800

19,85,400

Delhi VAT @ 12.5% 2,48,175

22,33,575

Cost of Final Product

Raw material - R1 75,000

Raw material - R2 3,88,383

Raw material - R3 7,94,160

Plant & Machinery (10% of `18,00,000) 1,80,000

Amount paid for services 4,00,000

Processing charges 8,50,000

Profit 3,10,000

Assessable value (as per section 4 of Central Excise Act, 1944) 29,97,543

Excise duty @ 10% 2,99,754

EC @ 2% 5,995

SHEC@ 1% 2,998

Total 33,06,290

Delhi VAT @ 12.5% - Output Tax 4,13,286

Selling Price 37,19,576

Computation of Net Tax Payable

Excise Duty / Service Tax /

` EC @ 2%

` SHEC @

1%

`

Delhi VAT

`

Output tax 2,99,754 5,995 2,998 4,13,286

Less: VAT / CENVAT

Credit

Raw material – R1 (7,500)

(7,500)

(150)

(150)

(75)

(75)

(10,341)

(8,686)

Raw material – R2 (38,000) (760) (380) -

Raw material – R3 - - - -

Plant and machinery (90,000) (1,800) (900) (82,725)

Service tax (48,000) (960) (480) -

Net tax payable 1,08,754 2,175 1,088 3,11,534

Illustration 13: Mr. X is registered in Central Excise/Delhi VAT/CST and he is a manufacturer and he has

purchased raw material R1 for `2,50,000 and has paid excise duty @ 7% plus education cess and secondary

and higher education cess and Delhi VAT @ 10%.

He purchased raw material R2 for `3,20,000 and paid excise duty @ 5% plus education cess and secondary

and higher education cess and central sales tax @ 2% and raw material was purchased from other state.

He has purchased raw material R3 for `5,50,000 and has paid excise duty @ 7% plus education cess and

secondary and higher education cess and Delhi VAT @ 10%.

Processing charges `4,00,000 plus profit `70,000.

The manufacturer has taken input services in connection with manufacturing of the product and has paid

Page 46: sipoysatish@gmail · PDF file · 2013-08-07Explain the concept of Value Added Tax / Explain Taxonomy (process ... Explain accounting treatment of VAT as suggested by ICAI. ... Q26

Value Added Tax 46

`5,00,000 plus service tax of `60,000 plus education cess.

Final product was sold and excise duty is 18% plus education cess and Delhi VAT @ 10%.

Show the working for VAT/Cenvat credit and also show the working for payment of tax at the time of sale

of final product.

Solution : ` Raw material – R1

Assessable value 2,50,000

Excise duty @ 7% 17,500

EC @ 2% 350

SHEC @ 1% 175

Total 2,68,025

Delhi VAT @ 10% - Input Tax 26,803

Purchase Price 2,94,828

Raw material – R2

Assessable value 3,20,000

Excise duty @ 5% 16,000

EC @ 2% 320

SHEC @ 1% 160

Total 3,36,480

Central Sales tax @ 2% - Input Tax 6,730

Purchase Price 3,43,210

Raw material – R3

Assessable value 5,50,000

Excise duty @ 7% 38,500

EC @ 2% 770

SHEC @ 1% 385

Total 5,89,655

Delhi VAT @ 10% - Input Tax 58,966

Purchase Price 6,48,621

Cost of Final Product

Raw material - R1 2,50,000

Raw material - R2 (Purchase Price minus Excise Duty including EC & SHEC) 3,26,730

Raw material - R3 5,50,000

Processing charges 4,00,000

Payment for services 5,00,000

Profit 70,000

Assessable value (as per section 4 of Central Excise Act, 1944) 20,96,730

Excise duty @ 18% 3,77,411

EC @ 2% 7,548

SHEC @ 1% 3,774

Total 24,85,463

Delhi VAT @ 10% - Output Tax 2,48,546

Selling Price 27,34,009

CENVAT /VAT ACCOUNT

Excise Duty / Service Tax EC @ 2% SHEC @ 1% Delhi VAT

Page 47: sipoysatish@gmail · PDF file · 2013-08-07Explain the concept of Value Added Tax / Explain Taxonomy (process ... Explain accounting treatment of VAT as suggested by ICAI. ... Q26

Value Added Tax 47

` ` ` ` (Invoice 1) Raw material – R1 17,500.00 350.00 175.00 26,803.00

(Invoice 2) Raw material – R2 16,000.00 320.00 160.00 -

(Invoice 3) Raw material – R3 38,500.00 770.00 385.00 58,966.00

Service tax 60,000.00 1,200.00 600.00

Total 1,32,000.00 2,640.00 1,320.00 85,769.00

Final product

Output tax 3,77,411.00 7,548.00 3,774.00 2,48,546.00

Less: VAT/CENVAT Credit (1,32,000.00) (2,640.00) (1,320.00) (85,769.00)

Net tax payable 2,45,411.00 4,908.00 2,454.00 1,62,777.00

Illustration 14: ABC Limited is a manufacturing concern and the company has submitted the particulars as

given below:-

Purchased raw material, R1: `2,00,000.

(+) Excise Duty @10%

(+) Education cess @ 2%

(+) SHEC @1%

(+) DVAT @10%

Purchased raw material, R2: `3,00,000.

(+) Excise Duty @12%

(+) Education cess @ 2%

(+) SHEC @1%

(+) CST @2%

The company purchased plant and machinery for `10 Lakhs and paid excise duty @10% plus EC 2% plus

SHEC @1% plus DVAT @ 4%.

Life of the plant and machinery is 5 years and depreciation is allowed @ 20% on SLM.

The company has taken certain services in connection with manufacturing of goods and has paid `3,00,000

plus service tax @ 12% plus EC 2% plus SHEC 1%.

Other processing expenditure incurred by the company is `5,00,000 and profit is `3,00,000.

Final product was sold by the company and output excise duty is 12% plus EC 2% plus SHEC @ 1% and

output VAT is 10%.

Company is registered under Central Excise Act, DVAT Act and CST Act and the company is not eligible

for SSI exemption.

Compute Output Excise Duty, Output VAT / Net Excise Duty/ Net VAT under Consumption Variant.

(b) Presume the goods were sold in some other states to Registered Dealer against Form ‘C’

(c) Presume all the goods were exported by ABC Ltd.

Solution (a):

Page 48: sipoysatish@gmail · PDF file · 2013-08-07Explain the concept of Value Added Tax / Explain Taxonomy (process ... Explain accounting treatment of VAT as suggested by ICAI. ... Q26

Value Added Tax 48

Raw material –R1

Purchase price 2,00,000.00

Add: Excise duty @ 10% 20,000.00

Add: Education cess @ 2% 400.00

Add: SHEC @ 1% 200.00

2,20,600.00

Add: Delhi VAT @ 10% 22,060.00

2,42,660.00

Raw material –R2

Purchase price 3,00,000.00

Add: Excise duty @12% 36,000.00

Add: Education cess @ 2% 720.00

Add: SHEC @1% 360.00

3,37,080.00

Add: CST @ 2% 6,741.60

3,43,821.60

Capital goods

Purchase price 10,00,000.00

Add: Excise duty @10% 1,00,000.00

Add: Education cess @ 2% 2,000.00

Add: SHEC @1% 1,000.00

11,03,000.00

Add: Delhi VAT @ 4% 44,120.00

11,47,120.00

Services 3,00,000.00

Service Tax @ 12% 36,000.00

Add: Education cess @ 2% 720.00

Add: SHEC @1% 360.00

3,37,080.00

Cost of final product

Raw material –R1 2,00,000.00

Raw material –R2 3,06,741.60

Capital goods (10,00,000 @ 20%) 2,00,000.00

Services 3,00,000.00

Other processing charges 5,00,000.00

Profit 3,00,000.00

Assessable Value 18,06,741.60

Add: Excise duty @12% 2,16,809.00

Add: Education cess @ 2% 4,336.18

Add: SHEC @ 1% 2,168.09

20,30,054.87

Add: Delhi VAT @ 10% 2,03,005.49

CENVAT/VAT ACCOUNT

Excise Duty / Service Tax EC @ 2% SHEC @

1%

Delhi VAT

Raw material – R1 20,000 400 200 22060

Raw material – R2 36,000 720 360 -

Plant and machinery 1,00,000 2,000 1,000 44,120

Service tax 36,000 720 360 -

Page 49: sipoysatish@gmail · PDF file · 2013-08-07Explain the concept of Value Added Tax / Explain Taxonomy (process ... Explain accounting treatment of VAT as suggested by ICAI. ... Q26

Value Added Tax 49

Total 1,92,000 3,840 1,920 66,180

Final product

Output tax 2,16,809.00 4,336.18 2,168.09 2,03,005.49

Less: VAT/CENVAT

Credit

1,92,000.00 3,840.00 1,920.00 66,180.00

Net tax payable 24,809.00 496.18 248.09 1,36,825.49

Rounded Off 24,809.00 496.00 248.00 1,36,825.00

Solution (b):

In this case the manufacturer shall charge central sales tax on the sale instead of Delhi Value Added Tax

Act. Since CST shall also be paid to the Delhi Government, VAT credit shall be allowed in the normal

manner and it can be adjusted against output CST and tax treatment shall be as given below:

Raw material –R1

Purchase price 2,00,000.00

Add: Excise duty @10% 20,000.00

Add: Education cess @ 2% 400.00

Add: SHEC @1% 200.00

2,20,600.00

Add: Delhi VAT @ 10% 22,060.00

2,42,660.00

Raw material –R2

Purchase price 3,00,000.00

Add: Excise duty @12% 36,000.00

Add: Education cess @ 2% 720.00

Add: SHEC @1% 360.00

3,37,080.00

Add: CST @ 2% 6,741.60

3,43,821.60

Capital goods

Purchase price 10,00,000.00

Add: Excise duty @10% 1,00,000.00

Add: Education cess @ 2% 2,000.00

Add: SHEC @1% 1,000.00

11,03,000.00

Add: Delhi VAT @ 4% 44,120.00

11,47,120.00

Services 3,00,000.00

Service Tax @ 12% 36,000.00

Add: Education cess @ 2% 720.00

Add: SHEC @1% 360.00

3,37,080.00

Cost of final product

Raw material –R1 2,00,000.00

Raw material –R2 3,06,741.60

Capital goods (10,00,000 @ 20%) 2,00,000.00

Services 3,00,000.00

Other processing charges 5,00,000.00

Profit 3,00,000.00

Assessable Value 18,06,741.60

Page 50: sipoysatish@gmail · PDF file · 2013-08-07Explain the concept of Value Added Tax / Explain Taxonomy (process ... Explain accounting treatment of VAT as suggested by ICAI. ... Q26

Value Added Tax 50

Add: Excise duty @12% 2,16,809.00

Add: Education cess @ 2% 4,336.18

Add: SHEC @1% 2,168.09

20,30,054.87

Add: CST @ 2%- output tax 40,601.10

CENVAT/VAT ACCOUNT

Excise Duty /

Service Tax

EC @ 2% SHEC @ 1% Delhi VAT/CST

Raw material – R1 20,000 400 200 22,060

Raw material – R2 36,000 720 360 -

Plant and machinery 1,00,000 2,000 1,000 44,120

Service tax 36,000 720 360 -

Total 1,92,000 3,840 1,920 66,180

Final product

Output tax 2,16,809.00 4,336.18 2,168.09 40,601.10

Less: VAT/CENVAT Credit 1,92,000.00 3,840.00 1,920.00 66,180.00

Net tax payable 24,809.00 496.18 248.09 -

CENVAT/VAT credit balance - - - 25,578.90

Rounded Off 24,809 496 248 25,579

Solution (c):

Since the goods have been exported, there will not be any output tax and CENVAT credit/ VAT credit shall

be refunded.

CENVAT/VAT ACCOUNT

Excise Duty / Service Tax EC @ 2% SHEC @ 1% Delhi VAT/CST

Raw material – R1 20,000 400 200 22,060

Raw material – R2 36,000 720 360 -

Plant and machinery 1,00,000 2,000 1,000 44,120

Service tax 36,000 720 360 -

Total 1,92,000 3,840 1,920 66,180

Output tax Nil Nil Nil Nil

Question 17 (V. Imp): What are methods for computation of Value Added Tax.

Answer:

There are three methods to calculate the ‘Value Added Tax’.

1. Invoice Method

2. Addition Method

3. Subtraction Method

1. Invoice Method

Under this method, tax is imposed at each stage of sales on the entire sale value and tax paid at the earlier

stage is allowed as set-off. In other words, out of tax so calculated, tax paid at the earlier stage i.e., at the

stage of purchases, is allowed as set-off from the tax payable, and at every stage the differential tax is being

paid. The most important aspect of this method is that at each stage, tax credit is available for Tax paid on

the purchases made. This method is also called the ‘Tax Credit Method’ or ‘Voucher Method’ and can be

shown in the manner given below:

Product X

Sale Price `200

Output VAT `20

Page 51: sipoysatish@gmail · PDF file · 2013-08-07Explain the concept of Value Added Tax / Explain Taxonomy (process ... Explain accounting treatment of VAT as suggested by ICAI. ... Q26

Value Added Tax 51

Assumed Tax rate 10%.

In this case, manufacturer A shall be allowed tax credit for the input VAT of `20 + `20 hence his cost shall

be `400 and value addition is `600 and sale price shall be `1,000 and output VAT shall be `100 and net VAT

shall be `60

Under this method, tax credit cannot be claimed unless the tax invoice is produced. As a result, in chain, if at

any stage the transaction is kept out of the books, still there is no loss of revenue because the manufacturer

will not be allowed tax credit. The department will be in a position to recover the full tax at the next stage.

Thus, the possibility of tax evasion, if not entirely ruled out, will be reduced to a minimum.

2. Addition Method

This method aggregates all the factor payments like wages, rent, interest including profits to arrive at the

total value addition on which the rate is applied to calculate the tax. This is in line with the income method

of calculating National Income.

E.g. In the above case value addition of manufacturer A is `600 and net tax shall be `60 i.e. 10% of `600.

A drawback of this method is that is does not facilitate matching of invoices for detecting tax evasion.

3. Subtraction Method

Under this method, the tax is charged on the difference between sales and purchases.

In the above case, if tax credit is not allowed, purchase price for manufacture shall be `440 and his value

addition is `600 hence sale price shall be `1,040 and VAT shall be charged on the difference of purchase

price and sale price i.e. `1,040 – `440 = `600 and net VAT shall be `60 and it is called Direct Subtraction i.e.

difference is exclusive of VAT.

If the difference is inclusive of VAT, it will be called Intermediate Subtraction. E.g. In the above case

difference in sale price and purchase price shall be `1,100 – `440 = `660 and amount of net VAT shall be

`660 / 110% x 10% = `60

The drawback of this method is that is does not facilitate matching of invoices for detecting tax evasion.

Question 18 (V. Imp.): Explain VAT Invoice / Tax Invoice

Answer:

A dealer may issue two type of invoices:

1. VAT invoice or Tax invoice

2. Retail invoice

Manufacturer A

Value addition i.e. expenses + profit `600

Sale Price `1000

Output VAT `100

Net VAT `60

{`100 – (20+20)} Product Y

Sale Price `200

Output VAT `20

Net VAT `20

Page 52: sipoysatish@gmail · PDF file · 2013-08-07Explain the concept of Value Added Tax / Explain Taxonomy (process ... Explain accounting treatment of VAT as suggested by ICAI. ... Q26

Value Added Tax 52

1. Tax Invoice or VAT Invoice

Tax credit is allowed only on the basis of Tax Invoice or VAT Invoice. No VAT credit is allowed on the

basis of retail invoice. VAT invoice can be issued only by a registered dealer and also it can be issued only

to a registered dealer and it must contain TIN of buyer as well as seller.

Tax invoice can not be issued by a Composition Dealer.

Tax invoice can not be issued in case of sale/purchase in the course of inter state trade or commerce.

Tax invoice shall be issued in duplicate and original shall be given to the buyer and duplicate shall be

retained by the seller.

Tax invoice shall contain the following particulars:

a. the words Tax Invoice at a prominent place;

b. the name, address and registration number of the selling registered dealer;

c. the name and address of the purchaser and his registration number.

d. an individual pre-printed serialised number Invoice and the date on which the tax invoice is issued;

A dealer may maintain separate numerical series, with distinct codes either, as a prefix or suffix, for each

place of business in case the dealer has more than one place of business in State or for each product in

case he deals in more than one product or both;

e. description, quantity, volume and value of goods sold and services provided and the amount of tax

charged thereon indicated separately;

f. the signature of the selling dealer or his employee, manager or agent, duly authorized by him; and

g. the name and address of the printer and first and last serial number of tax invoices printed and supplied

by him to the dealer.

Issue of Duplicate Tax Invoice

If a purchaser claims to have lost the original tax invoice, the selling dealer may, subject to such

conditions and restrictions as may be prescribed, provide a copy clearly marked as a duplicate.

Where a purchasing dealer claims to have lost the original tax invoice, the selling dealer may, upon a request

made by the purchasing dealer accompanied by an undertaking cum indemnity (in Delhi Form DVAT-36),

provide a copy of such tax invoice clearly marked as a “duplicate” and shall furnish a copy of such

undertaking cum indemnity along with his return for the tax period in which such “duplicate” tax invoice

has been issued. (Similar provision in Rule 44 of DVAT Rules, 2005)

2. Retail invoice

No VAT credit is allowed on the basis of retail invoice. Retail invoice can be issued if VAT credit is not

allowed to the buyer.

Contents of retail invoice shall be as given below:

a. the words Retail Invoice or cash memorandum or bill at a prominent place;

Page 53: sipoysatish@gmail · PDF file · 2013-08-07Explain the concept of Value Added Tax / Explain Taxonomy (process ... Explain accounting treatment of VAT as suggested by ICAI. ... Q26

Value Added Tax 53

b. the name, address and registration number of the selling dealer, if registered;

c. in case the sale is in the course of inter-state trade or commerce, the name, registration number and

address of the purchasing dealer and type of statutory form, if any, against which the sale has been made;

d. an individual pre-printed serialized number invoice and the date on which the retail invoice is issued;

e. description, quantity, volume and value of goods sold and services provided, inclusive of amount of tax

charged thereon; and

f. the signature of the selling dealer or his employee, manager or agent, duly authorized by him.

Retail invoice shall be issued in duplicate, the original shall be issued to the purchaser and the copy shall be

retained by the selling dealer.

Question 19 (V. Imp.): Explain composition scheme for small traders.

Answer:

Composition scheme for small traders

(Similar provision in Section 16 of Delhi Value Added Tax Act, 2004)

If any dealer is having turnover UPTO ` 50 LAKHS, he may apply for Composition Scheme.

Composition Scheme is not allowed in the following cases:

(i) If any dealer is procuring goods from outside the State or is selling or supplying goods to any place

outside the State at any time during the year.

(ii) If he is registered under Central Sales Tax Act.

Salient features of the Scheme are asunder:

(i) A dealer covered under Composition Scheme is not allowed to take VAT Credit on his purchases but he

must retain all the tax invoices for the goods purchased by him.

(ii) A dealer covered under composition scheme is not allowed to issue tax invoice and also not allowed to

charge any tax from the buyer rather he himself has to pay tax on his sales turnover (@ 1% in Delhi).

(iii) The benefit of composition scheme is that dealer is exempt from maintaining lengthy records required

under VAT.

(iv) If any dealer is purchasing goods from a dealer covered under composition scheme, no VAT credit is

allowed to such a purchasing dealer.

(v) If any dealer is covered under composition scheme, he must purchase goods only from registered dealer.

(vi) If turnover has exceeded `50 lakhs, he has to shift immediately to the normal system.

(vii) If any dealer is covered under composition scheme, he may reject the scheme and may opt for normal

procedure but only from beginning of the year.

(viii) If any dealer is opting out of the composition scheme, he will be allowed VAT credit for the stock held

by him on the date of opting out.

A dealer has to apply in the prescribed form for opting the scheme and rejecting the scheme

A dealer opting for composition scheme has to apply in Form No. DVAT 01 under DVAT Act, 2004

Page 54: sipoysatish@gmail · PDF file · 2013-08-07Explain the concept of Value Added Tax / Explain Taxonomy (process ... Explain accounting treatment of VAT as suggested by ICAI. ... Q26

Value Added Tax 54

Illustration 15: Mr. A, a retailer, presents the following information for the year –

(1) Purchases of goods: `30 lakhs (VAT @ 4%)

(2) Sales (at fixed selling price inclusive of all taxes): `49 lakhs (VAT on sales @ 4%).

Expenses of keeping detailed statutory records required under the VAT-law will be `1,00,000 p.a. Other

expenses are `6,00,000 p.a.

(b) Presume he has opted for composition scheme and goods were sold for `49,00,000 and composition tax

@ 1% for the turnover was paid by the dealer. Expenditure on maintaining records was reduced to `40,000

Show the treatment for VAT in both of the options and also pass accounting entries for the composition

scheme.

Solution: Sales (inclusive of all taxes) 49,00,000

Less:

- Tax (VAT = 49 lakhs x 4 ÷ 104) 1,88,462

- Cost of goods sold 30,00,000

- Cost of maintaining records 1,00,000

- Normal Expenses 6,00,000

Profit of the dealer 10,11,538

Solution (b): Sales 49,00,000

Less:

Cost of goods sold 31,20,000

(No credit under composition scheme, hence, cost of goods sold will be higher)

Cost of maintaining records 40,000

Normal Expenses 6,00,000

Composition tax (49,00,000 x 1%) 49,000

Profit of the dealer 10,91,000

The profit of the dealer is higher if the dealer opts for composition scheme. Hence, composition

scheme should be opted.

The accounting entries under composition scheme are as under –

(Accounting entries are not part of the syllabus rather it is given for better understanding)

1. Purchases made:

Purchase A/c Dr.

To Cash /Bank A/c

` 31,20,000

`

31,20,000

2. Sales Made:

Cash/Bank A/c Dr.

To Sales A/c

49,00,000

49,00,000

3. Composition Tax paid @ 1%

Composition Tax A/c Dr.

To Cash/Bank A/c

49,000

49,000

Page 55: sipoysatish@gmail · PDF file · 2013-08-07Explain the concept of Value Added Tax / Explain Taxonomy (process ... Explain accounting treatment of VAT as suggested by ICAI. ... Q26

Value Added Tax 55

Illustration 16: A Registered Dealer Mr. A of Punjab extracted raw produce X and raw produce Y from

mines at `30,000 and `40,000 respectively and sold the same at 150% margin to Manufacturer B of Punjab

(VAT rate is 4% on produce X and 12.5% on produce Y).

Manufacturer B is a dealer operating under composition scheme who is liable to composition tax @ 0.4% of

turnover.

Manufacturer B used X and Y as raw material, added 100% of cost of raw material towards manufacturing

expenses and profits and sold the product to wholesaler C of Punjab.

Wholesaler C sold the same to Retailer D of Punjab at 25% above cost (VAT rate is 4%).

The retailer D sold the same to a consumer at 20% above cost (VAT rate is 4%).

Show the amount of VAT payable by each person.

Solution:

Manufacturer A ` Cost of raw produce X 30,000

Add: Profit margin @ 150% 45,000

Total 75,000

Add: VAT @ 4% 3,000

Total 78,000

Cost of raw produce Y 40,000

Add: Profit margin @ 150% 60,000

Total 1,00,000

Add: VAT @ 12.5% 12,500

Total Selling Price 1,12,500

VAT payable by Dealer Mr. A (3,000 + 12,500) 15,500

Manufacturer B

Cost of manufacturer B (78,000 + 1,12,500) 1,90,500

Add: Expenses + Profit - 100% 1,90,500

Total Selling Price 3,81,000

Composition tax @ 0.4% (to be paid by the seller) 1,524

Wholesaler C

Cost of wholesaler C 3,81,000

(No VAT credit is allowed because of purchase from a dealer opting under composition Scheme.)

Add: Profit Margin @ 25% 95,250

Total 4,76,250

Add: VAT @ 4% 19,050

Total Selling Price 4,95,300

VAT payable by Wholesaler Mr. C 19,050

Retailer D

Cost of Retailer D 4,76,250

(VAT credit is allowed for `19,050)

Add: Profit margin @ 20% 95,250

Total 5,71,500

Add: VAT @ 4% 22,860

Page 56: sipoysatish@gmail · PDF file · 2013-08-07Explain the concept of Value Added Tax / Explain Taxonomy (process ... Explain accounting treatment of VAT as suggested by ICAI. ... Q26

Value Added Tax 56

Total Selling Price 5,94,360

VAT Payable (22,860 – 19,050) 3,810

Question 20 (Imp.): Write a note on filing of return under state VAT.

Answer:

A dealer is required to file return on monthly / quarterly / half-yearly / annual basis depending upon the

turnover. In case of very high turnover, return has to be filed on monthly basis and in case of low turnover,

return has to be filed on yearly basis. In general return is filed in the manner given below:

(i) If the turnover is upto `10 lakhs, return should be filed on yearly basis.

(ii) If turnover is more than 10 lakhs but upto `50 lakhs, return should be filed on half-yearly basis.

(iii) If it is more than 50 lakhs but upto ` 5 crores, return should be filed on quarterly basis.

(iv) If it is exceeding `5 crores, return should be filed on monthly basis.

Such duration of filing the return is also called ‘Tax Period’.

Question 21 (Imp.): Write a note on assessment under State VAT.

Answer:

Assessment

The basic simplification of VAT is with reference to assessment. Under VAT system, there is no

compulsory assessment at the end of each year. The VAT liability is self-assessed by the dealer himself in

terms of submission of returns. The other procedures are also simple in all the States.

Deemed assessment concept is a major feature of the VAT. If no specific notice is issued proposing

departmental audit of the books of accounts of the dealer within the time limit specified in the respective

State VAT Acts, the dealer will be deemed to have been self-assessed on the basis of the returns submitted

by him.

VAT pre-supposes that all the dealers are honest. Scrutiny may be done in cases where a doubt arises of

under-reporting of transaction or evasion of tax. Honest dealers will be protected and fictitious or dishonest

would be penalized heavily.

Question 22: Write a note on System of Cross Checking.

Answer:

System of Cross Checking

In the VAT system more emphasis has been laid on self-assessment. Hence, a system of cross-checking is

essential. Dealers may be asked to submit the list of sales or purchases above a certain monetary value or to

give the dealer-wise list from whom or to whom the goods have been purchased/sold for values exceeding a

prescribed monetary ceiling.

A cross-checking computerized system is being worked out on the basis of coordination between the tax

authorities of the State Government and the authorities of Central Excise and Income-tax to compare

constantly the tax returns and set-off documents of VAT system of the States and those of Central Excise

and Income-tax. This comprehensive cross-checking system will help reduce tax evasion and also lead to

significant growth of tax revenue. At the same time, by protecting the interests of tax-complying dealers

against the unfair practices of tax-evaders, the system will also bring in more equal competition in the sphere

of trade and industry.

Page 57: sipoysatish@gmail · PDF file · 2013-08-07Explain the concept of Value Added Tax / Explain Taxonomy (process ... Explain accounting treatment of VAT as suggested by ICAI. ... Q26

Value Added Tax 57

Question 23 (Imp.): Write a note on maintaining of books of accounts and records under State VAT.

Answer:

Books of Accounts/ Records/ Documents

The following records should be maintained under VAT system:

1. Purchase records, showing details of purchases on which tax has been paid, purchases made without

payment of tax, purchases made from an exempted unit (Military Canteen) and purchases made from

outside State.

2. Sales records, showing separately sales made at different tax rates, zero-rated taxable sales and tax-free

sales.

3. VAT account - A monthly account specifying total output tax, total input tax and net tax payable or the

excess tax credit due for carry forward.

4. Details of input tax calculations where the dealer is making both taxable and tax free sales.

5. Stock records showing stock receipts and deliveries and manufacturing records.

6. Stock records showing separately the particulars of goods stored in cold storage, warehouse, godown or

any other place taken on hire.

7. Order records and delivery challans, wherever applicable.

8. Annual accounts including trading, profit and loss accounts and the balance sheet.

9. Bank records, including statements, cheque book counter foils and pay-in-slips.

10. Cash book, daybook and ledger.

The following documents should be retained:

1. Original tax invoices for purchases on which tax has been paid and invoices for purchases made without

payment of tax in numerical order.

2. Copies of tax invoices related to taxable sales and invoices related to exempt sales shall be retained date

wise and in numerical order.

Question 24 (Imp.): Explain the provisions of audit under State VAT.

Answer:

Audit

In the VAT system considerable weightage is placed on audit work in place of routine assessment work.

Page 58: sipoysatish@gmail · PDF file · 2013-08-07Explain the concept of Value Added Tax / Explain Taxonomy (process ... Explain accounting treatment of VAT as suggested by ICAI. ... Q26

Value Added Tax 58

Correctness of self assessment will be checked through a system of Departmental Audit. A certain

percentage of the dealers will be taken up for audit every year on a scientific basis. If, however, evasion is

detected in the course of audit, the previous records of the concerned dealer may be taken up for audit.

Authorized officers of the department will visit the business place of the dealer to conduct the audit. The

auditors will examine the correctness of the returns vis-a-vis the books of accounts of the dealer or any other

information available with them. They will be equipped with the information gathered from various agencies

such as suppliers, income tax department, excise and customs department, banks etc. Officers of the higher

rank will supervise to ensure that the audit work is done in a free, fearless and impartial manner.

Under the sales-tax laws, tax evasion is considered to be on a large scale. The sales-tax departments of

various States have not been able to effectively check the menace (harm) of tax avoidance and tax evasion.

Therefore, apart from the departmental audit many States have also incorporated the concept of audit of

accounts by Chartered Accountants. The State of Maharashtra has prescribed an elaborate list of particulars

to be furnished by the dealers. These particulars have to be verified by the VAT auditor.

However, auditing for all types of dealers may not be necessary. The selection of cases for auditing has to be

made in accordance with the criteria of the size of dealers. In Maharashtra and Rajasthan, the dealer whose

turnover exceeds `40 lakhs in any year is required to get his accounts audited in respect of such year.

Question 25 (V. Imp.): Explain merits of VAT.

Answer:

Merits

1. No Tax Evasion

Under VAT, unless proper records are kept in respect of duty or tax paid on various inputs, it is not possible

to claim credit. Hence, suppression of purchases or production will be very difficult.

2. Transparency

Under VAT system, the buyer knows, out of the total consideration paid for purchase of material, what is tax

component. Thus, the system ensures transparency, but prior to VAT the amount of tax actual paid may have

been different from the tax shown as paid. If a manufacturer purchases raw material for `10,00,000 plus

VAT @ 10% and his value addition is `5,00,000, if tax credit is not allowed, the product will be sold for

`16,00,000 and if output tax is 10%, the manufacturer will charge output tax of `1,60,000 but in reality tax

paid by the purchaser is `2,60,000 because input tax of `1,00,000 has also been recovered from the buyer and

in this sense prior to VAT the mechanism was not transparent. After VAT tax credit shall be allowed for

input tax of `1,00,000 and product will be sold for `15,00,000 and output tax shall be `1,50,000 and actually

paid is also `1,50,000 hence the system is transparent.

3. Neutrality (not affecting)

The greatest advantage of the tax credit system is that it does not interfere in the choice of decision for

purchases. This is because the system has anti-cascading effect. The system is neutral with regard to choice

of production technique, as well as business organisation. All other things remaining the same, the issue of

tax liability does not vary the decision about the source of purchase. In short, the allocation of resources is

left to be decided by the free play of market forces and competition. For example, a manufacturer may

purchase raw material after paying tax @ 5% or at higher rate of tax @ 8%. In both the cases the price of

final product will remain same, because he will get deduction of taxes on purchases while paying taxes on

final product.

4. Certainty

The VAT is a system based simply on transactions. Thus there is no need to go through complicated

definitions like sales, sales price, turnover of purchases and turnover of sales. The tax is also broad-based

Page 59: sipoysatish@gmail · PDF file · 2013-08-07Explain the concept of Value Added Tax / Explain Taxonomy (process ... Explain accounting treatment of VAT as suggested by ICAI. ... Q26

Value Added Tax 59

and applicable to all sales in business leaving little room for different interpretations. Thus, this system

brings certainty to a great extent. Also there are only a few rates under VAT in comparison to numerous

rates in earlier sales tax.

5. Better Revenue Collection and Stability

There will be a minimum possibility of revenue leakage, since the tax credit will be given only if the proof

of tax paid at an earlier stage is produced. This means that if the tax is evaded at one stage, full tax will be

recoverable from the person at the subsequent stage or from a person unable to produce proof of such tax

payment. Thus, in particular, an invoice of VAT will be self enforcing and will induce business to demand

invoices from the suppliers.

6. Better Accounting Systems

Since the tax paid on an earlier stage is to be received back, the system will promote better accounting

systems.

7. Effect on Retail Price/VAT is Inflationary

A persistent criticism of the VAT has been that since the tax is payable on the final sale price, the VAT

usually increases the prices of the goods. However, VAT does not have any inflationary impact as it merely

replaces the existing equal sales tax.

Question 26 (V. Imp): Explain demerits of VAT.

Answer:

Demerits

1. Central VAT/ State VAT

So long as Central VAT is not integrated with the State VAT, it will be difficult, to put the purchases from

other States at par with the State purchases.

2. Increase in Accounting Cost

For complying with the VAT provisions, the accounting cost will increase. The burden of this increase may

not be commensurate with the benefit to traders and small firms.

3. Increase in Working Capital

Since the tax is to be imposed or paid at various stages and not on last stage, it would increase the working

capital requirements and the interest burden on the same. In this way it is considered to be non-beneficial as

compared to the single stage-last point taxation system.

4. VAT is Regressive

VAT is a form of consumption tax. Since, the proportion of income spent on consumption is larger for the

poor than for the rich, VAT tends to be regressive. However, this weakness is inherent in all the forms of

consumption tax. While it may be possible to moderate the distribution impact of VAT by taxing necessities

at a lower rate, it is always advisable to moderate the distribution considerations through other programmes

rather than concessions or exemptions, which create complications for administration.

5. Increase in Administrative Cost

As a result of introduction of VAT, the administration cost of the State increase significantly as the number

of dealers to be administered will go up significantly.

Question 27: Explain role of ICAI in VAT.

Answer:

ICAI’s role in VAT

The ICAI has rendered pioneering service in evolving the necessary accounting guidelines both for

CENVAT as well as State Level VAT. It has brought out Guidance Notes for accounting for CENVAT as

Page 60: sipoysatish@gmail · PDF file · 2013-08-07Explain the concept of Value Added Tax / Explain Taxonomy (process ... Explain accounting treatment of VAT as suggested by ICAI. ... Q26

Value Added Tax 60

well as State-Level VAT. These Guidance Notes address all the accounting issues in regard to CENVAT

and State Level VAT. Further, the institute has brought out a comprehensive study on State Level VAT in

India. It contains an elaborate discussion of the various general principles of VAT and State Level VAT.

These general principles have been incorporated in the various State Level VAT legislation. However, there

are special provisions contained in the respective State level legislation to cater to the specific needs of the

States. Various State government have issued detailed clarification on different practical issues arising on

implementation of the State-Level VAT.

Question 28 (Imp.): Explain role of Chartered Accountant in VAT.

Answer:

Role of Chartered Accountant in VAT

Chartered Accountant have a key role to play in proper implementation of VAT.

(i) Record Keeping : VAT requires proper record keeping and accounting. Systematic records of input

credit and its proper utilisation is necessary for the success of VAT. Chartered Accountants are well

equipped to perform such tasks.

(ii) Tax Planning : In order to establish an efficient plan for purchases and sales, a careful study of VAT is

required. A Chartered Accountant is competent to analyze the impact of various alternatives and choose

the most optimum method of purchases and sales in order to minimize the tax impact.

(iii) Handling the audit departmental officers: There will be audit wing in department and certain

percentage of dealers will be taken up for audit every year on scientific basis. Chartered Accountant can

ensure proper record keeping so as to satisfy the departmental auditors. The professional expertise of a

Chartered Accountant will help him in effectively replying audit queries and sorting out audit objections.

(iv) External audit of VAT records: Under VAT system, trust has been reposed on tax payers as there will

be no regular assessment of all VAT returns but only few returns will be scrutinized. In other cases,

return filed by dealer will be accepted. Thus, a check on compliance becomes necessary. Chartered

Accountants can play a very vital role in ensuring tax compliance by audit of VAT accounts.

Question 29: Explain White Paper.

Answer:

An official study on any matter released by the Government is called White Paper. Government has realised

an official document on Value Added Tax and it is called White Paper on VAT.

This White Paper is a result of collective efforts of all the States in formulating the basic design of the State-

level Value Added Tax (VAT) through repeated and candid discussions in the Empowered Committee of

State Finance Ministers.

This White Paper on State-level Value Added Tax (VAT) is presented in three parts.

Part 1 contains the justification of VAT and its background. In the existing sales tax structure, there are

problems of double taxation of commodities and multiplicity of taxes, resulting in a cascading tax burden.

For instance, in the existing structure, before a commodity is produced, inputs are first taxed, and then after

the commodity is produced with input tax load, output is taxed again. This causes an unfair double taxation

with cascading effects. In the VAT, a set-off is given for input tax as well as tax paid on previous purchases.

In Part 2, the main design of VAT, as evolved on the basis of a consensus among the States through repeated

discussions in the Empowered Committee, has been elaborated. While doing so, it is recognized that this

VAT is a State subject and therefore the States will have freedom for appropriate variations consistent with

the basic design as agreed upon at the Empowered Committee.

Page 61: sipoysatish@gmail · PDF file · 2013-08-07Explain the concept of Value Added Tax / Explain Taxonomy (process ... Explain accounting treatment of VAT as suggested by ICAI. ... Q26

Value Added Tax 61

In Part 3, the other related issues have been discussed for effective implementation of VAT.

Goods and Services Tax (GST)

At present there are different Acts with regard to Goods and Services and also some tax are levied by

Central Government and some by State Government and in the implementation of Value Added Tax, there

are certain problems because of different tax structure e.g. no tax credit is allowed for Central Sales Tax and

also tax credit for sales tax cannot be set off from output excise duty or service tax and also vice versa is not

possible. In order to have one comprehensive law for goods and services and also for implementing smooth

functioning of Value Added Tax, it is proposed to have a new Act which will cover all the indirect taxes.

Goods and Service Tax is India’s most ambitious indirect tax reform aimed at attaining a comprehensive and

harmonized tax structure. The then Finance Minister, Mr. P. Chidambaram, in Union Budget 2006-07, had

proposed the roll out of GST. In a federal State like India, GST is aimed at accomplishing a common

domestic market, removing multiplicity of taxes, eliminating the cascading effect of tax, making the prices

of the Indian products competitive and, above all, benefiting the end consumers. In this regard, First

Discussion paper on Goods and Services Tax in India was proposed by the Empowered Committee of State

Finance Ministers on November 10, 2009. The Thirteenth Finance Commission gave its Task Force

Recommendations on GST on December 12, 2009. The GST model is expected to be dual so that both

Central and State Governments can collect taxes to raise resources to fulfill their sovereign

obligations/duties.

Illustration 17: The particulars regarding sale, purchase etc. of ABC Ltd. for the year 2012-13 are as under:

Purchases of raw material within the state –

(i) R1 for `60,00,000 + VAT @ 1%

(ii) R2 for `90,00,000 + VAT @ 4%

(iii) R3 for `15,00,000 + VAT @ 12.5%

(1) Sale of raw material R1 within the state – `66,00,000 + VAT @ 1%

(2) Sale of goods manufactured from raw material R2

(i) taxable sale within the state – `30,00,000 + VAT @ 4%

(ii) exempted sale within the state – `15,00,000

(iii) sale in the course of inter-state trade or commerce – `15,00,000 + CST @ 2%

(3) Goods manufactured from the raw material R3 `18,00,000 + VAT @ 12.5%

Show the tax treatment.

Solution:

Computation of Output Tax

Raw Material R1

Sale of Raw material within the State i.e. 66,00,000 x 1% = `66,000

Raw Material R2

(i) Sale of goods manufactured from raw material within the state = 30,00,000 x 4% = `1,20,000

Page 62: sipoysatish@gmail · PDF file · 2013-08-07Explain the concept of Value Added Tax / Explain Taxonomy (process ... Explain accounting treatment of VAT as suggested by ICAI. ... Q26

Value Added Tax 62

(ii) Since goods are exempt, no VAT shall be charged and also no tax credit shall be allowed.

(iii) Sale of goods manufactured from raw material in the course of interstate sale = 15,00,000 x 2% =

`30,000

Raw Material R3

Goods sold in the state 18,00,000 x 12.5% = `2,25,000

Output Tax

(i) State VAT = 66,000 + 1,20,000 + 2,25,000 = `4,11,000

(ii) CST = 30,000

Computation of Input Tax Credit

Raw material R1

60,00,000 x 1% = `60,000

Raw material R2

Raw material R2 was purchased for `90,00,000 but it was sold for `60,00,000 i.e. at a loss but in this case

VAT credit shall be allowed for full amount however proportionate VAT credit for the goods which are

exempt from VAT shall not be allowed hence VAT credit shall be allowed only for 75% of `90,00,000 i.e.

90,00,000 x 75% x 4% = `2,70,000

Raw material R3

15,00,000 x 12.5% = `1,87,500

Total Input Tax Credit = 60,000 + 2,70,000 + 1,87,500 = `5,17,500

Output Tax ` (i) State VAT 4,11,000

Less: VAT credit 4,11,000

VAT Payable Nil

(ii) CST 30,000

Less: VAT credit 30,000

CST payable Nil

Balance of unutilized VAT 76,500

Unutilized VAT shall be carry forward and in the balance sheet it will be shown as assets (advances)

Ultimately, the dealer can claim refund of VAT (no such refund is allowed in case of excise duty or service

tax except in case of export)

Illustration 18: Compute the net VAT liability of Mr. X from the information as given below:

(i) Raw material purchased from foreign market (including basic custom duty paid on imports @ 20% + EC

@ 3%) – `70,500

(ii) Raw material purchased from local market (including VAT charged on the material @ 1%) – `15,150

(iii) Raw material purchased from another state (excluding CST @ 2%) – `30,000

Page 63: sipoysatish@gmail · PDF file · 2013-08-07Explain the concept of Value Added Tax / Explain Taxonomy (process ... Explain accounting treatment of VAT as suggested by ICAI. ... Q26

Value Added Tax 63

(iv) Storage, transportation cost and insurance – `4,500

(v) Other manufacturing expenses incurred – `900.

Mr. X sold the goods to Mr. Y adding margin of profit @ 10% on the selling price.

VAT rate on sale of such goods is 10%.

Solution:

Total cost = 70,500 + 15,000 + 30,600 + 4,500 + 900 = `1,21,500.

Since profit is 10% of sale price, `1,21,500 is 90% of sale price hence sale price shall be

1,21,500 x 100% / 90% = `1,35,000 (it means the product was sold for 1,35,000 and there is a profit of 10%

i.e. 13,500)

VAT rate is 10% hence output tax shall be 1,35,000 x 10% = `13,500

Input tax credit shall be 15,150 x 1% / 101% = `150

Net tax = 13,500 – 150 = `13,350

Illustration 19: Calculate the VAT liability for the period Jan. 1, 2013 to Jan. 31, 2013 from the following

particulars:

Input worth `1,60,000 were purchased within the state plus VAT @ 12.5%

VAT paid on procurement of capital goods worth `1,50,000 plus VAT @ 12.5%.

`3,00,000 worth of finished goods were sold within the state plus VAT @ 4%

`1,50,000 worth of goods were sold in the course of inter-state trade plus CST @ 2%.

Show the total tax liability under the State VAT law and under the Central Sales Tax Act considering it to be

consumption variant.

Solution:

` ` VAT CST

Output Tax

Sale within State `3,00,000 x 4% 12,000

Inter state sale `1,50,000 x 2% 3,000

Less: VAT credit – On Inputs `1,60,000 x 12.5% = 20,000 12,000 3,000

– On capital goods `1,50,000 x 12.5% = 18,750

Tax Payable Nil Nil

Thus, out of VAT credit of `38,750, `15,000 can be utilized to pay tax and balance `23,750 will be carried

forward.

Page 64: sipoysatish@gmail · PDF file · 2013-08-07Explain the concept of Value Added Tax / Explain Taxonomy (process ... Explain accounting treatment of VAT as suggested by ICAI. ... Q26

Value Added Tax 64

Illustration 20: ABC Ltd. of Rajasthan purchased raw material A from Rajasthan for `20,800 (inclusive of

4% VAT), raw material B from Rajasthan for `45,000 (inclusive of 12.5% VAT), raw material C from China

for `67,980 (inclusive of 10% basic custom duty + EC @ 3%,) and raw material D from Maharashtra for

`600 (inclusive of 2% CST).

The plant and machinery required for manufacture was purchased for `4,16,000 (inclusive of 4% VAT).

Credit of VAT on plant and machinery is allowed in the year of purchase

The manufacturing and other expenses (excluding depreciation) were `1,51,420

The plant is to be depreciated at 100%. The manufacturer’s margin is 20% on cost.

The VAT rate on the manufactured product is 4%.

By way of necessary accounting entries, show the mode of operation of VAT system. Ignore the Central

Excise implications, assuming that there is no excise duty on the manufactured product.

Solution:

`

Raw material A (net of VAT `800) 20,000

Raw material B (net of VAT `5,000) 40,000

Raw material C (Basic custom duty + EC will form part of cost, as it is not available as

credit)

67,980

Raw material D (CST will form part of cost, as it is not available as credit) 600

Depreciation on plant and machinery (100% of 4,00,000) 4,00,000

Manufacturing and other expenses 1,51,420

Cost of the product 6,80,000

Add: 20% margin 1,36,000

Selling price 8,16,000

Add: VAT @ 4% of 8,16,000 32,640

VAT payable by the manufacturer = 32,640 – 800 – 5,000 – 16,000 = `10,840

Accounting Entries

(Accounting entries are not part of the syllabus rather it is given for better understanding)

Dr. ` Cr. ` 1. Purchase of Raw material A -

Raw material A A/c Dr. 20,000

Input Tax Credit A/c Dr. 800

To Bank 20,800

2. Purchase of Raw material B

Raw material B A/c Dr. 40,000

Input Tax Credit A/c Dr. 5,000

To Bank 45,000

3. Purchase of Raw material C

Raw material C A/c Dr. 67,980

To Bank 67,980

4. Purchase of Raw material D

Raw material D A/c Dr. 600

To Bank 600

5. Purchase of plant and machinery i.e. capital goods

Page 65: sipoysatish@gmail · PDF file · 2013-08-07Explain the concept of Value Added Tax / Explain Taxonomy (process ... Explain accounting treatment of VAT as suggested by ICAI. ... Q26

Value Added Tax 65

Plant and machinery A/c Dr. 4,00,000

Input Tax Credit A/c Dr. 16,000

To Bank 4,16,000

6. Manufacturing & other expenses

Manufacturing and other expenses A/c Dr. 1,51,420

To Bank 1,51,420

7. Depreciation

Depreciation A/c Dr. 4,00,000

To plant and machinery A/c 4,00,000

8. Sale of manufacturing product

Bank Dr. 8,48,640

To sales A/c 8,16,000

To VAT payable A/c 32,640

9. Payment of VAT

VAT payable A/c Dr. 32,640

To Input Tax credit A/c 21,800

To Bank 10,840

Illustration 21: A manufacturer has purchased raw material for `2,08,000 (inclusive of 4% VAT) and plant

and machinery of `4,00,000 (VAT nil).

The manufacturing and other expenses (including building, rent, wages etc.) are `6,00,000.

He sells the resultant products at 50% above cost (VAT on sales is 4%).

The plant and machinery is to be depreciated at 50% straight line.

Compute the amount of VAT payable as per the addition method.

Solution:

Computation of Value added and VAT payable

` Depreciation on plant and machinery @ 50% of 4,00,000 2,00,000

Manufacturing and other expenses 6,00,000

Total factor payments 8,00,000

Profit @ 50% of total cost i.e. Material cost + Factor payments i.e. 2,00,000 + 8,00,000 5,00,000

Value Added 13,00,000

VAT @ 4% on value added 52,000

Illustration 22: Manufacturer A of Punjab extracted raw produce X and raw produce Y from mines at

`30,000 and `40,000 respectively and sold the same at 150% margin to Manufacturer B of Delhi (CST rate is

2%).

Manufacturer B of Delhi used X and Y as raw material; added 100% of cost of raw material towards

manufacturing expenses and profits and sold the resultant product to wholesaler C of Delhi (VAT rate is

4%).

Wholesaler C of Delhi sold the same to Retailer D of Delhi at 25% above cost (VAT rate is 4%)

The retailer D sold the same to a consumer at 20% above cost (VAT rate is 4%).

Page 66: sipoysatish@gmail · PDF file · 2013-08-07Explain the concept of Value Added Tax / Explain Taxonomy (process ... Explain accounting treatment of VAT as suggested by ICAI. ... Q26

Value Added Tax 66

Show the amount of VAT payable by each person under the invoice method of computation of VAT.

Solution: `

Manufacturer A

Cost of raw material X and Y (30,000 + 40,000) 70,000

Add: Profit @ 150% of `70,000 1,05,000

Total 1,75,000

Add: CST @ 2% 3,500

Total Selling Price 1,78,500

CST of `3,500 shall be payable to the Government of Punjab

Manufacturer B

Cost of Manufacturer B 1,78,500

(VAT credit of `3,500 is not allowed because payment has been made in Punjab)

Add: Profit @ 100% of cost 1,78,500

Total 3,57,000

Add: VAT @ 4% 14,280

Total Selling Price 3,71,280

Wholesaler C

Cost of Wholesale C 3,57,000

(VAT credit shall be allowed for `14,280)

Add: Profit @ 25% 89,250

Total 4,46,250

Add: VAT @ 4% 17,850

Total Selling Price 4,64,100

Net tax payable shall be (17,850 – 14,280) 3,570

Retailer D

Cost of Retailer D 4,46,250

(VAT credit shall be allowed for `17,850)

Add: Profit @ 20% 89,250

Total 5,35,500

Add: VAT @ 4% 21,420

Total Selling Price 5,56,920

Net tax payable shall be (21,420 – 17,850) 3,570

Illustration 23: Manufacturer A of Jaipur extracted raw produce X and raw produce Y from mines at

`30,000 and `40,000 respectively and sold the same at 150% margin to Manufacturer B of Jaipur (VAT rate

is 4% on produce X and 12.5% on produce Y).

Manufacturer B of Jaipur used X and Y as raw material; added 100% of cost of raw material towards

manufacturing expenses and profits and sold the resultant product to wholesaler C of Delhi (CST rate is

2%).

Wholesaler C of Delhi sold the same to Retailer D of Delhi at 20% above cost (VAT rate is 4%).

The Retailer D sold the same to a consumer at 20% above cost (VAT rate is 4%).

Page 67: sipoysatish@gmail · PDF file · 2013-08-07Explain the concept of Value Added Tax / Explain Taxonomy (process ... Explain accounting treatment of VAT as suggested by ICAI. ... Q26

Value Added Tax 67

Show, by way of invoice method, the amount of VAT payable by each person.

Solution: `

Manufacturer A

Cost of raw material X 30,000

Add: Profit @ 150% of `30,000 45,000

Total 75,000

Add: VAT @ 4% 3,000

Total Selling Price 78,000

Cost of raw material Y 40,000

Add: Profit @ 150% of `40,000 60,000

Total 1,00,000

Add: VAT @ 12.5% 12,500

Total Selling Price 1,12,500

Manufacturer B

Cost of Manufacturer B (`75,000 + `1,00,000) 1,75,000

Add: Profit @ 100% of cost 1,75,000

Total 3,50,000

Add: CST @ 2% 7,000

Total Selling Price 3,57,000

Balance in the VAT receivable account (3,000 + 12,500) – 7,000 8,500

Wholesaler C

Cost of Wholesale C 3,57,000

Add: Profit @ 20% 71,400

Total 4,28,400

Add: VAT @ 4% 17,136

Total Selling Price 4,45,536

Retailer D

Cost of Retailer D 4,28,400

(VAT credit shall be allowed for `17,136)

Add: Profit @ 20% 85,680

Total 5,14,080

Add: VAT @ 4% 20,563

Total Selling Price 5,34,643

Net tax payable shall be (20,563 – 17,136) 3,427

Illustration 24: Mr. X of Rajasthan is registered with VAT authorities. He presents the following details for

the month of January, 2013–

Purchase from Rajasthan 20,00,000

Purchases from Delhi 16,00,000

Sales within Rajasthan out of purchases from Delhi 16,00,000

Sales to dealer of Maharashtra out of purchases within Rajasthan 2,00,000

Sales to dealer of Maharashtra out of purchases from Delhi 4,00,000

Page 68: sipoysatish@gmail · PDF file · 2013-08-07Explain the concept of Value Added Tax / Explain Taxonomy (process ... Explain accounting treatment of VAT as suggested by ICAI. ... Q26

Value Added Tax 68

Aforesaid amounts are exclusive of taxes. VAT rate is 4%. CST rate is 2%.

Compute tax payable by Mr. X.

Solution:

The tax payable by Mr. X for the month of January, 2013 is computed herein below-

Sales inside Rajasthan (VAT)` Total sales 16,00,000

Output VAT liability @ 4% 64,000

Input VAT credit available on purchases from Rajasthan (20,00,000 x 4%) 80,000

Net tax payable NIL

Excess VAT credit to be utilized against payment of CST 16,000

Sales outside Rajasthan (CST)

Total sales 6,00,000

Output CST liability @ 2% 12,000

Input VAT credit available on purchases from Rajasthan 16,000

Balance VAT credit available 4,000

Illustration 25: Mr. A , a registered dealer presents following details for March, 2013

1. Opening Balance of Input VAT credit as on 01.03.2013 ` 30,000.

2. Inputs purchased during the month of March: `30 lakh.

3. Within the state sales of manufactured goods: `40 lakh.

4. Inter –State Sales: `8 lakh.

CST rate is 2%. There was no inventory as on 01.03.2013 or 31.03.2013 The VAT laws governing Mr. A

provide for the refund of input VAT credit after the end of the first financial year itself.

The above amounts are exclusive of VAT/ CST.

VAT rate is 12.5% on inputs and 4% on sales. Compute the amount of refund available to Mr. A.

Solution:

Computation of refund available to Mr. A (Amounts in `) Opening balance of input VAT credit

30,000

Add: VAT credit availed on inputs purchased during March (12.5% of 30 lakh)

3,75,000

Less: VAT payable on sales (4% on `40 lakh)

1,60,000

Less: CST payable on inter-state sales (2% on 8 lakh)

16,000

Balance lying as VAT credit as on 31.03.2013 eligible for refund 2,29,000

Page 69: sipoysatish@gmail · PDF file · 2013-08-07Explain the concept of Value Added Tax / Explain Taxonomy (process ... Explain accounting treatment of VAT as suggested by ICAI. ... Q26

Value Added Tax 69

Illustration 26: Mr. X of Maharashtra has purchased a plant (capital goods) valuing `30 lakh (VAT thereon

@ 4%) on 01.01.2013. On 31.03.2013 he sold such plant for `28 lakh (net) to another trader engaged in

same business.

Pass the journal entries to record these transactions, assuming that there is no other transaction during the

intervening period. Ignore Depreciation.

(In Maharashtra, VAT credit on capital goods is allowed in one instalments but if capital goods have been

sold within 3 years, in that case proportionate amount of VAT credit shall be reversed.)

Solution:

Journal entries in the books of Mr. X

(Accounting entries are not part of the syllabus rather it is given for better understanding)

01/01/13 Plant A/c Dr.

Input VAT Credit A/c Dr.

To Bank

(Plant purchased and 100% credit of VAT paid thereon

allowed in the State of Maharashtra)

30,00,000

1,20,000

31,20,000

31/03/13 Plant A/c Dr.

To Input VAT credit A/c

(Since the plant has been sold after 3 months i.e. within

36 months, hence, proportionate input-credit of VAT to

be reversed on capital goods sold = 1,20,000 x 33 ÷ 36 =

`1,10,000. This amount will increase the cost of the

plant).

1,10,000

1,10,000

31/03/13 Bank A/c Dr.

Loss on sale of Plant A/c Dr.

To Plant A/c

(Plant sold and loss booked)

28,00,000

3,10,000

31,10,000

Illustration 27: Mr. M a manufacturer of taxable as well as tax-free goods, furnishes the following

information.

(a) Sales of Product A (tax-free goods): `100 lakhs;

(b) Sales of Product B (taxable goods): `200 lakhs (VAT @ 12.5%)

(c) Purchases of Input ‘X’ (used in manufacturing of Product A only): `60 lakhs (VAT @ 4%)

(d) Purchases of Input ‘Y’ (used in manufacturing of Product B only): `150 lakhs (VAT @ 4%)

(e) Purchases of Input ‘Z’ (used in manufacturing of Product A & B): `30 lakhs (VAT @ 20%)

Compute the amount of VAT payable in cash by Mr. M assuming that input ‘Z’ is used in product A and B

in the ratio of 1:2 Ignore implications under other laws.

Solution: ` Computation of VAT liability of Mr. M

Since product A is tax free, inputs used in manufacturing of product A shall not be eligible for VAT credit.

Inputs used in manufacturing of product B shall be eligible for VAT credit and is as given below:

Page 70: sipoysatish@gmail · PDF file · 2013-08-07Explain the concept of Value Added Tax / Explain Taxonomy (process ... Explain accounting treatment of VAT as suggested by ICAI. ... Q26

Value Added Tax 70

Input Y (150,00,000 x 4%) 6,00,000

Input Z (30,00,000 x 2/3 x 20%) 4,00,000

Total VAT credit 10,00,000

Sale of product B

Output Tax (200,00,000 x 12.5%) 25,00,000

Less: VAT credit 10,00,000

VAT payable 15,00,000

Illustration 28:

Mr. X is Registered under Central Sales Tax Act 1956 and Delhi VAT Act 2004 and he has purchased raw

material for `15,00,000 in Delhi and paid excise duty @ 10% plus education cess and secondary and higher

education cess @ 3% and Delhi VAT @ 4%.

(i) 1/6th

of raw material is stock transferred to some other state.

(ii) 1/6th

of raw material is used in manufacturing of final product which is exempt from VAT and

processing charges and profit is `1,50,000.

(iii) 1/6th

of raw material is used in manufacturing of final product which is exported from India.

(iv) 1/6th

of raw material is used in manufacturing of final product which is sold in some other state and CST

@ 2% and processing charges and profit is `1,50,000.

(v) 1/6th

of raw material is used in manufacturing of final product which is sold in the same state. Processing

charges and profit is `1,50,000.

(vi) 1/6th

of raw material is lying in the stock.

Output Delhi VAT @ 12.5%.

Compute Output VAT / Tax Credit and Net VAT.

Solution: ` Raw material 15,00,000.00

Excise duty @ 10% 1,50,000.00

EC @ 2% 3,000.00

SHEC @ 1% 1,500.00

Total 16,54,500.00

Delhi VAT @ 4% 66,180.00

17,20,680.00

(i)

Since 1/6th

of the stock has been transferred to some other state, VAT credit allowed for such stock transfer

shall be 2% (4% - 2%).

Tax paid (66,180 x 1/6) 11,030.00

VAT credit (11,030/4% x (4%-2%)) 5,515.00

(In case of stock transfer VAT credit is allowed after retaining 2%)

(ii)

Page 71: sipoysatish@gmail · PDF file · 2013-08-07Explain the concept of Value Added Tax / Explain Taxonomy (process ... Explain accounting treatment of VAT as suggested by ICAI. ... Q26

Value Added Tax 71

Raw material 2,75,750.00

VAT (`66,180 x 1/6) 11,030.00

Processing charges and profit 1,50,000.00

Sale Price 4,36,780.00

Output VAT Nil

(Since output VAT is exempt, hence VAT credit for input VAT is not allowed and it will be added in the

cost)

(iii) No tax is payable in case of export but VAT credit will be allowed.

(iv)

Raw material 2,75,750.00

Processing charges and profit 1,50,000.00

Sale Price 4,25,750.00

Central sales tax @ 2% 8,515.00

4,34,265.00

(v)

Raw material 2,75,750.00

Processing charges and profit 1,50,000.00

Sale Price 4,25,750.00

Delhi VAT @ 12.5% 53,218.75

4,78,968.75

VAT A/C

Particulars DVAT

` CST

`

OUTPUT TAX

1/6th

raw material (Stock transfer) - -

1/6th

Final Product (Exempt from VAT) -

1/6th

Final product Exported - -

1/6th

Final product sold in some other State 8,515.00

1/6th

Final product sold in same State 53,218.75 -

Total 53,218.75 8,515.00

Less: INPUT TAX CREDIT

1/6th

raw material (Stock transfer) 5,515.00 -

1/6th

Final Product (Exempt from VAT) - -

1/6th

Final product Exported 11,030.00 -

1/6th

Final product sold in some other State 11,030.00 -

1/6th

Final product sold in same State 11,030.00 -

1/6th

Raw material lying in the stock 11,030.00

Total 49,635.00 -

Tax payable 3,583.75 8,515.00

Rounded off 3,584.00 8,515.00

Page 72: sipoysatish@gmail · PDF file · 2013-08-07Explain the concept of Value Added Tax / Explain Taxonomy (process ... Explain accounting treatment of VAT as suggested by ICAI. ... Q26

Value Added Tax 72

PRACTICE PROBLEMS TOTAL PROBLEMS 17

Problem 1: Compute the invoice value to be charged and amount of tax payable under VAT by a Registered Dealer who

had purchased goods for `2,00,000 (exclusive of VAT) and after adding for expenses of `18,000 and profit

of `35,000 had sold out the same in the same state. The rate of VAT on purchase and sales is 12.5%.

(b) Goods were sold under inter-state sale to a Registered Dealer.

Problem 2: Compute the VAT amount payable by Mr. A (Registered Dealer) who purchases goods from a manufacturer

on payment of `5,62,500 (including VAT) and earns 20% profit on purchase. The goods have been sold to

retailers and VAT rate on purchase and sale is 12.5%.

(b) Goods were sold under inter-state sale to an Unregistered Dealer.

Problem 3:

Mr. X (Registered Dealer) is trader and he has purchased certain goods from Punjab for `4,50,000 and has

paid central sales tax @ 2%.

He has sold all the goods in the state of Delhi for `6,50,000 plus VAT @ 12.5%.

He has purchased certain goods in Delhi for `5,50,000 and paid VAT @ 12.5% and all the goods were sold

by him under inter state sale to some person in M.P. for `7,50,000 plus central sales tax @ 2%.

Show VAT calculation.

Page 73: sipoysatish@gmail · PDF file · 2013-08-07Explain the concept of Value Added Tax / Explain Taxonomy (process ... Explain accounting treatment of VAT as suggested by ICAI. ... Q26

Value Added Tax 73

Problem 4:

Mr. X (Registered Dealer in Delhi) is a manufacturer and he has purchased raw material R1 from Punjab for

`3,00,000 plus central sales tax @ 2%.

He has purchased raw material R2 in Delhi for `4,00,000 inclusive of VAT @ 10%.

His processing charges are `5,00,000 and profit margin is `5,00,000.

Half of the goods were sold in Delhi and VAT payable is @ 10% and remaining half were sold to a person

in M.P. under inter state sale and has charge central sales tax @ 2%. Show working of VAT/CST Payable.

Problem 5:

Mr. X is a manufacturer sells goods to Mr. B, a distributor for `6,70,000. Mr. B sells goods to Mr. K, a

wholesaler for `9,60,000.

The wholesaler sells the goods to a retailer for `11,40,000. The retailer sold the goods to consumers for

`13,00,000.

All the above amounts are exclusive of VAT.

All the above persons are registered under local VAT and also under Central Sales Tax Act 1956.

Compute input tax credit, output tax and net tax under invoice method for each of the person and VAT rate

is @ 12.5%.

(b) Presume all the above amounts are inclusive of VAT @ 12.5%.

(c) Presume manufacturer and distributor are in Punjab and wholesaler and retailer are in Delhi.

(i) VAT is exclusive

(ii) VAT is inclusive

(d) Presume manufacturer is in Punjab, distributor in Haryana, wholesaler in U.P. and retailer in M.P.

Amounts are exclusive of CST @ 2%.

Problem 6:

Mr. X is a Dealer Registered in Delhi Value Added Tax Act, 2004 and also under Central Sales Tax Act,

1956 and he has submitted the informations as given below:

(i) Purchased Goods A from Delhi for `2,00,000 and paid VAT @ 4% and sold the goods in Delhi at a profit

of 50% on purchase price and charged VAT @ 4%.

(ii) Purchased goods B from U.P. for `4,00,000 and paid central sales tax @ 2% and sold goods in Delhi at a

profit of 50% on purchase price and charged VAT @ 12.5%.

(iii) Purchased goods C from Delhi for `8,00,000 and paid VAT @ 12.5% and sold the goods at a profit of

50% on purchase price to a registered dealer in Orissa and charged Central Sales Tax @ 2%

Page 74: sipoysatish@gmail · PDF file · 2013-08-07Explain the concept of Value Added Tax / Explain Taxonomy (process ... Explain accounting treatment of VAT as suggested by ICAI. ... Q26

Value Added Tax 74

(iv) Purchased goods D for `10,00,000 in Delhi and paid VAT @ 12.5% and sold the goods at a profit of

50% on purchase price to an unregistered dealer in Punjab and charged Central Sales Tax @ 12.5%.

(v) Purchased goods E from Madhya Pradesh for `6,00,000 and paid Central Sales Tax @ 1% and sold

goods at a profit of 50% on purchase price in Maharashtra and charged central sales tax @ 1%.

(vi) Purchased goods F from Delhi `14,00,000 and paid VAT @ 1% and the goods were sold at a profit of

50% on purchase price to an unregistered dealer in Maharashtra and charged central sales tax @ 1%.

(vii) Purchased goods G for `12,00,000 in Delhi and paid VAT @ 12.5% and goods were stock transferred

to some other state.

(viii) Purchased goods H for `16,00,000 in Delhi and paid VAT @ 4% and goods were exported at a profit

of 50% on purchase price and no VAT was charged (because as per section 6 Central Sales Tax Act, 1956,

CST can not be charged in case of export sale.)

(ix) Purchased goods I for `18,00,000 in Delhi and paid VAT @ 12.5% and sold the goods at a profit of 50%

on purchase price to a manufacturer in SEZ and no VAT was charged.

Show the tax treatment for VAT and also compute his income tax liability for the assessment year 2013-14.

Problem 7:

Mr. X is registered in Central Excise/Delhi VAT/CST is a manufacturer and he has purchased raw material

R1 for `1,50,000 and has paid excise duty @ 8% plus education cess and secondary and higher education

cess and Delhi VAT @ 10%.

He purchased raw material R2 for `2,20,000 and paid excise duty @ 4% plus education cess and secondary

and higher education cess and central sales tax @ 2% and raw material was purchased from other state.

He has purchased raw material R3 for `4,50,000 and has paid excise duty @ 8% plus education cess and

secondary and higher education cess and Delhi VAT @ 10%.

Processing charges `6,00,000 plus profit `50,000.

The manufacturer has taken input services in connection with manufacturing of the product and has paid

`2,00,000 plus service tax of `24,000 plus education cess and secondary and higher education cess.

Final product was sold and excise duty is 16% plus education cess plus SHEC and Delhi VAT @ 10%.

Show the working for VAT credit and also show the working for payment of tax at the time of sale of final

product.

Problem 8:

ABC Limited is a manufacturing concern and the company has submitted the particulars as given below:-

Purchased raw material, R1: `3,00,000.

(+) Excise Duty @ 10%

(+) Education cess @ 2%

(+) SHEC @ 1%

(+) DVAT @ 10%

Page 75: sipoysatish@gmail · PDF file · 2013-08-07Explain the concept of Value Added Tax / Explain Taxonomy (process ... Explain accounting treatment of VAT as suggested by ICAI. ... Q26

Value Added Tax 75

Purchased raw material, R2: `4,00,000.

(+) Excise Duty @ 12%

(+) Education cess @ 2%

(+) SHEC @ 1%

(+) CST @ 2%

The company purchased plant and machinery for `12 Lakhs and paid excise duty @10% plus EC 2% plus

SHEC @1% plus DVAT @ 4%.

Life of the plant and machinery is 5 years and depreciation is allowed @ 20% on SLM.

The company has taken certain services in connection with manufacturing of goods and has paid `4,00,000

plus service tax @ 12% plus EC 2% plus SHEC 1%.

Other processing expenditure incurred by the company is `6,00,000 and profit is `4,00,000 and final product

was sold by the company and output excise duty is 12% plus EC 2% plus SHEC @ 1% and output VAT is

10%.

Company is registered under Central Excise Act, DVAT Act and CST Act and the company is not eligible

for SSI exemption.

Compute Output Excise Duty, Output VAT / Net Excise Duty/ Net VAT under Consumption Variant.

(b) Presume the goods were sold in some other states to Registered Dealer against Form ‘C’

(c) Presume all the goods were exported by ABC Ltd.

Problem 9:

Mr. X is Registered under Central Sales Tax Act 1956 and Delhi VAT Act 2004 and he has purchased raw

material for `12,00,000 in Delhi and paid excise duty @ 10% plus education cess and secondary and higher

education cess @ 3% and Delhi VAT @ 4%.

(i) 1/6th

of raw material is stock transferred to some other state.

(ii) 1/6th

of raw material is used in manufacturing of final product which is exempt from VAT and

processing charges and profit is `1,00,000.

(iii) 1/6th

of raw material is used in manufacturing of final product which is exported from India.

(iv) 1/6th

of raw material is used in manufacturing of final product which is sold in some other state and CST

@ 2% and processing charges and profit is `1,00,000.

(v) 1/6th

of raw material is used in manufacturing of final product which is sold in the same state. Processing

charges and profit is `1,00,000.

(vi) 1/6th

of raw material is lying in the stock.

Page 76: sipoysatish@gmail · PDF file · 2013-08-07Explain the concept of Value Added Tax / Explain Taxonomy (process ... Explain accounting treatment of VAT as suggested by ICAI. ... Q26

Value Added Tax 76

Output Delhi VAT @ 12.5%.

Compute output VAT / Tax Credit / Net VAT.

Problem 10:

The following particulars are provided by Mr. Karan of Calcutta, who has purchased Raw materials for

manufacturing product A and Product B from Mr. Piyush. The State VAT for Raw Materials and other

materials was 12.5%.

` 1. Cost of Raw materials purchased 3,00,000

2. VAT paid to Mr. Piyush 37,500

3. Cost of other materials

- Local Purchases 50,000

- Interstate Purchases 80,000

4. VAT paid on Local Materials Purchased-12.5% 6,250

5. CST Paid @ 2% 1,600

6. Manufacturing Expenses 49,200

7. Profit Margin (on Sale Value) 20%

Mr. Karan utilized and manufactured 75% of production as Product A and 25% of production as Product B.

While Product A are subject to 12.5% VAT, product B are exempt. All materials were used in production

and there was no closing stock of Raw materials and other materials.

What would be the invoice value of Sales charged by Mr. Karan if all the manufactured goods were sold

within the State? What would be his liability under VAT?

Problem 11:

Bhim, a registered dealer under DVAT /CST Act submits the following information for the month of

February, 2013.

Particulars Amount

`

Rate of

VAT

Details of purchase

Raw material purchased from another State (CST @ 2%).

Raw material X purchased within the State

Raw material Y imported from Singapore (includes custom duty paid @ 10%)

Raw material Z purchased within the State.

12,00,000

18,00,000

13,00,000

8,00,000

1%

12.5%

Details of sales

Sale of goods produced from raw material X.

Sale of goods produced from inter-State purchase and imported raw materials.

Sale of goods produced from raw material Z.

30,00,000

34,00,000

12,00,000

4%

1%

12.5%

Note: The purchase and sales figures given above do not include VAT/CST.

Assume that there was no opening or closing inventory. Compute the amount of Value Added Tax (VAT)

payable by Bhim for the month of February, 2013.

Page 77: sipoysatish@gmail · PDF file · 2013-08-07Explain the concept of Value Added Tax / Explain Taxonomy (process ... Explain accounting treatment of VAT as suggested by ICAI. ... Q26

Value Added Tax 77

Problem 12:

Vijay Co., furnishes you the following information:

Raw material purchased `9,00,000 plus VAT @ 4%.

Manufacturing expenses (revenue nature) `3,00,000.

Sale price `15,00,000 plus VAT @ 4%

Plant & machinery acquired `4,00,000 plus VAT @ 4%.

Compute VAT liability under (i) Gross Product Variant.

(ii) Consumption Variant.

State which variant is beneficial to the dealer?

Problem 13:

The following are details of purchases, sales, etc. effected by Kapil & Co., a registered dealer, for the year

ended 31.03.2013:

Particulars Amount

(`) Purchase of raw materials within State, 1000 units, inclusive of VAT levy at 6% 7,42,000

Inter-State purchase of raw materials, inclusive of CST at 2% 3,06,000

Import of raw materials, inclusive of basic customs duty plus education cess of `38,050 5,35,000

Capital goods purchased on 01.05.2012, inclusive of VAT levy at 10% 5,50,000

(input credit to be spread over 2 financial years)

Other manufacturing expenses 2,00,000

Sale of taxable goods within State, inclusive of VAT levy at 4% 10,40,000

Sale of goods within State, exempt from levy of VAT 2,00,000

(Goods were manufactured from the Inter-State purchase of raw materials)

Closing stock as on 31.03.2013 was 100 units of raw materials purchased within the State

Input credit is allowed only on raw material used in manufacture of the taxable goods.

Compute the VAT liability of the dealer for the year ended 31.03.2013.

Problem 14:

Compute net VAT liability of Rahul from the following information:

Particulars ` ` Raw materials from foreign market (Including Basic Custom duty @ 20%

plus EC)

- 1,36,500

Raw material purchased from local market

Page 78: sipoysatish@gmail · PDF file · 2013-08-07Explain the concept of Value Added Tax / Explain Taxonomy (process ... Explain accounting treatment of VAT as suggested by ICAI. ... Q26

Value Added Tax 78

Cost of raw material 3,50,000

Add: Excise duty @ 16% 56,000

4,06,000

Add: VAT @ 4%

16,240 4,22,240

Raw material purchased from neighbouring State (Includes CST @ 2%) 56,100

Storage and transportation cost 14,000

Manufacturing expenses 36,000

Rahul sold goods to Sohan and earned profit @ 12% on the cost of production. VAT rate on sale of such

goods is 4%.

Problem 15:

Compute the VAT liability of Mr. Pankaj Ahuja for the month of October, 2012, using the ‘Invoice method’

of computation of VAT.

Purchases from the local market

(Includes VAT 4%) `81,120

Storage cost incurred ` 2,800

Transportation cost ` 3,200

Goods sold at a margin of 5% on the cost of such goods

VAT rate on sales 12.5%.

Problem 16:

X Ltd of Delhi made a total purchases of input and capital goods of `100,00,000 during the month of March,

2013. The following further information is available.

(i) Goods worth `25,00,000 were purchased from UP on which CST @ 2% was paid.

(ii) Goods purchased from unregistered dealers amounting to `1,00,000.

(iii) It purchased capital goods (not eligible for input credit) worth `10,00,000 plus VAT @ 5% and

those eligible for input credit for `20,00,000 plus VAT @ 12.5%.

(iv) Remaining goods were purchased from Delhi and paid VAT @ 4%.

(v) Sales made in Delhi during the month of March, 2013 is `50,00,000 on which VAT at 12.5% is

payable.

Assuming that all purchases given are exclusive of tax.

Calculate:

(a) the amount of purchases eligible for input credit.

(b) the amount of input credit available for the month of March, 2013.

Page 79: sipoysatish@gmail · PDF file · 2013-08-07Explain the concept of Value Added Tax / Explain Taxonomy (process ... Explain accounting treatment of VAT as suggested by ICAI. ... Q26

Value Added Tax 79

(c) the VAT payable for the month of March, 2013.

The input VAT credit on eligible capital goods is available in 36 equal monthly instalments.

Problem 17:

Mr. X is registered under Delhi VAT Act and he submits the following information.

Compute the net VAT liability from the following information:

` Import of raw material (including 10% import duty) 5,00,000

Raw material purchased from Delhi (including excise duty @ 12%) 7,00,000

VAT @ 4% on the above purchase

Raw material purchased from UP (including CST @ 2%) 2,00,000

Transportation and manufacturing expenses 1,00,000

Mr. X sold entire stock at a profit of 20% on the sale. VAT rate on such sale is 12.5%.

SOLUTIONS TO

PRACTICE PROBLEMS

Solution 1: ` Purchase price 2,00,000

Add: Expenses 18,000

Add: Profit 35,000

Amount to be billed 2,53,000

Page 80: sipoysatish@gmail · PDF file · 2013-08-07Explain the concept of Value Added Tax / Explain Taxonomy (process ... Explain accounting treatment of VAT as suggested by ICAI. ... Q26

Value Added Tax 80

Add: VAT @ 12.5% - Output Tax 31,625

Total invoice value 2,84,625

VAT Payable

VAT charged in the invoice – Output Tax 31,625

Less: VAT credit on input 12.5% of `2,00,000 – Input Tax (25,000)

Net VAT Payable 6,625

Solution 1(b): ` Purchase price 2,00,000

Add: Expenses 18,000

Add: Profit 35,000

Amount to be billed 2,53,000

Add: CST @ 2% - Output Tax 5,060

Total invoice value 2,58,060

CST Payable

CST charged in the invoice – Output Tax 5,060

Less: VAT credit on input 12.5% of `2,00,000 – Input Tax (25,000)

Balance VAT Credit (to be carried forward or refund can be taken) 19,940

Solution 2: ` Purchase price 5,62,500

Less: Input tax (5,62,500 x 12.5 / 112.5) (62,500)

Purchase price net of tax 5,00,000

Add: Profit (5,00,000 x 20%) 1,00,000

Amount to be billed 6,00,000

Add: VAT @ 12.5% - Output Tax 75,000

Total invoice value 6,75,000

VAT Payable

Output tax 75,000

Less: Tax credit (12.5% of `5,00,000) – Input Tax (62,500)

Net VAT Payable 12,500

Solution 2(b): ` Purchase price 5,62,500

Less: Input tax (5,62,500 x 12.5 / 112.5) (62,500)

Purchase price net of tax 5,00,000

Add: Profit (5,00,000 x 20%) 1,00,000

Amount to be billed 6,00,000

Add: CST @ 12.5% - Output Tax 75,000

Total invoice value 6,75,000

CST Payable

Output tax 75,000

Less: Tax credit (12.5% of `5,00,000) – Input Tax (62,500)

Net CST Payable 12,500

Solution 3:

Page 81: sipoysatish@gmail · PDF file · 2013-08-07Explain the concept of Value Added Tax / Explain Taxonomy (process ... Explain accounting treatment of VAT as suggested by ICAI. ... Q26

Value Added Tax 81

Computation of VAT payable:

Purchase from Punjab and sold in Delhi ` Cost 4,50,000

Add: Central sales tax @ 2% 9,000

Purchase price 4,59,000

Sale 6,50,000

Add: VAT @ 12.5% - Output tax 81,250

Selling Price 7,31,250

Purchase from Delhi and sold in M.P. ` Cost 5,50,000

Add: VAT @ 12.5% - VAT credit 68,750

Purchase price 6,18,750

Sale 7,50,000

Add: Central sales tax @ 2% - Output tax 15,000

Selling Price 7,65,000

State VAT 81,250

Less: VAT credit 68,750

Net State VAT payable 12,500

CST Payable 15,000

Solution 4:

Computation of VAT payable: `

Raw material – R1

Cost 3,00,000

Add: Central sales tax @ 2% 6,000

Purchase price 3,06,000

Raw material – R2

Purchase price 4,00,000

Less: Input tax (4,00,000 x 10 / 110) 36,364

3,63,636

Cost of final product

Raw material – R1 3,06,000

Raw material – R2 3,63,636

Processing charges 5,00,000

Profit margin 5,00,000

Total 16,69,636

Goods sold in Delhi

Assessable value 8,34,818

Add: VAT @ 10% - Output tax 83,482

Sales value 9,18,300

Goods sold in M.P.

Assessable value 8,34,818

Add: Central sales tax @ 2% - Output tax 16,696

Sales value 8,51,514

State VAT 83,482

Page 82: sipoysatish@gmail · PDF file · 2013-08-07Explain the concept of Value Added Tax / Explain Taxonomy (process ... Explain accounting treatment of VAT as suggested by ICAI. ... Q26

Value Added Tax 82

Less: VAT credit 36,364

Net State VAT payable 47,118

CST Payable 16,696

Solution 5(a): `

Manufacturer (Mr. X)

Sale price 6,70,000

Add: VAT @ 12.5% 83,750

Total invoice value 7,53,750

VAT A/C:

Output tax 83,750

Less: Tax credit Nil

Net tax payable 83,750

Distributor (Mr. B)

Purchase price 6,70,000

Add: Profit 2,90,000

Amount to be billed 9,60,000

Add: VAT @ 12.5% 1,20,000

Total invoice value 10,80,000

VAT A/C:

Output tax 1,20,000

Less: Tax credit 83,750

Net tax payable 36,250

Wholesaler (Mr. K)

Purchase price 9,60,000

Add: Profit 1,80,000

Amount to be billed 11,40,000

Add: VAT @ 12.5% 1,42,500

Total invoice value 12,82,500

VAT A/C:

Output tax 1,42,500

Less: Tax credit 1,20,000

Net tax payable 22,500

Retailer

Purchase price 11,40,000

Add: Profit 1,60,000

Amount to be billed 13,00,000

Add: VAT @ 12.5% 1,62,500

Total invoice value 14,62,500

VAT A/C:

Output tax 1,62,500

Less: Tax credit 1,42,500

Net tax payable 20,000

Page 83: sipoysatish@gmail · PDF file · 2013-08-07Explain the concept of Value Added Tax / Explain Taxonomy (process ... Explain accounting treatment of VAT as suggested by ICAI. ... Q26

Value Added Tax 83

Solution 5(b): ` Manufacturer (Mr. X)

Sale price 6,70,000

Output tax (6,70,000 x 12.5 % / 112.5%) 74,444

VAT A/C:

Output tax 74,444

Less: Tax credit Nil

Net tax payable 74,444

Distributor (Mr. B)

Sale price 9,60,000

Output tax (9,60,000 x 12.5% / 112.5%) 1,06,667

VAT A/C:

Output tax 1,06,667

Less: Tax credit 74,444

Net tax payable 32,223

Wholesaler (Mr. K)

Sale price 11,40,000

Output tax (11,40,000 x 12.5% / 112.5%) 1,26,667

VAT A/C:

Output tax 1,26,667

Less: Tax credit 1,06,667

Net tax payable 20,000

Retailer

Sale price 13,00,000

Output tax (13,00,000 x 12.5% / 112.5%) 1,44,444

VAT A/C:

Output tax 1,44,444

Less: Tax credit 1,26,667

Net tax payable 17,777

Solution 5(c): (i) VAT is exclusive ` Manufacturer (Mr. X)

Sale price 6,70,000

Add: VAT @ 12.5% 83,750

Total invoice value 7,53,750

VAT A/C:

Output tax 83,750

Less: Tax credit Nil

Net tax payable 83,750

Distributor (Mr. B)

Purchase price 6,70,000

Add: Profit 2,90,000

Page 84: sipoysatish@gmail · PDF file · 2013-08-07Explain the concept of Value Added Tax / Explain Taxonomy (process ... Explain accounting treatment of VAT as suggested by ICAI. ... Q26

Value Added Tax 84

Amount to be billed 9,60,000

Add: CST @ 2% 19,200

Total invoice value 9,79,200

VAT A/C:

Output tax 19,200

Less: VAT credit to the extent of tax payable 19,200

Tax Payable NIL

Net VAT credit 64,550

Wholesaler (Mr. K)

Purchase price 9,79,200

Add: Profit 1,60,800

Amount to be billed 11,40,000

Add: VAT @ 12.5% 1,42,500

Total invoice value 12,82,500

VAT A/C:

Output tax 1,42,500

Less: Tax credit Nil

Net tax payable 1,42,500

Retailer

Purchase price 11,40,000

Add: Profit 1,60,000

Amount to be billed 13,00,000

Add: VAT @ 12.5% 1,62,500

Total invoice value 14,62,500

VAT A/C:

Output tax 1,62,500

Less: Tax credit 1,42,500

Net tax payable 20,000

Solution: (ii) VAT is inclusive ` Manufacturer (Mr. X)

Sale price 6,70,000

Output tax (6,70,000 x 12.5 % / 112.5%) 74,444

VAT A/C:

Output tax 74,444

Less: Tax credit Nil

Net tax payable 74,444

Distributor (Mr. B)

Sale price 9,60,000

Output tax (9,60,000 x 2% / 102%) 18,824

VAT A/C:

Page 85: sipoysatish@gmail · PDF file · 2013-08-07Explain the concept of Value Added Tax / Explain Taxonomy (process ... Explain accounting treatment of VAT as suggested by ICAI. ... Q26

Value Added Tax 85

Output tax 18,824

Less: VAT credit to the extent of tax payable 18,824

Tax Payable NIL

Net VAT credit 55,620

Wholesaler (Mr. K)

Sale price 11,40,000

Output tax (11,40,000 x 12.5% / 112.5%) 1,26,667

VAT A/C:

Output tax 1,26,667

Less: Tax credit Nil

Net tax payable 1,26,667

Retailer

Sale price 13,00,000

Output tax (13,00,000 x 12.5% / 112.5%) 1,44,444

VAT A/C:

Output tax 1,44,444

Less: Tax credit 1,26,667

Net tax payable 17,777

Solution 5(d): No VAT credit is allowed to manufacturer, distributor, wholesaler and retailer because goods

purchased from other states. `

Manufacturer (Mr. X)

Sale price 6,70,000

Add: CST @ 2% 13,400

Total invoice value 6,83,400

VAT A/C:

Output tax 13,400

Less: Tax credit Nil

Net tax payable 13,400

Distributor (Mr. B)

Purchase price 6,83,400

Add: Profit 2,76,600

Amount to be billed 9,60,000

Add: CST @ 2% 19,200

Total invoice value 9,79,200

VAT A/C:

Output tax 19,200

Less: Tax credit Nil

Net tax payable 19,200

Wholesaler (Mr. K)

Purchase price 9,79,200

Add: Profit 1,60,800

Amount to be billed 11,40,000

Add: CST @ 2% 22,800

Total invoice value 11,62,800

Page 86: sipoysatish@gmail · PDF file · 2013-08-07Explain the concept of Value Added Tax / Explain Taxonomy (process ... Explain accounting treatment of VAT as suggested by ICAI. ... Q26

Value Added Tax 86

VAT A/C:

Output tax 22,800

Less: Tax credit Nil

Net tax payable 22,800

Retailer

Purchase price 11,62,800

Add: Profit 1,37,200

Amount to be billed 13,00,000

Add: VAT @ 12.5% 1,62,500

Total invoice value 14,62,500

VAT A/C:

Output tax 1,62,500

Less: Tax credit Nil

Net tax payable 1,62,500

Solution 6: ` (i)

Purchased Goods A from Delhi 2,00,000

Add: VAT @ 4% 8,000

Purchase Price 2,08,000

Cost 2,00,000

Add: Profit {2,08,000 – 8,000(as VAT credit is available)} x 50% 1,00,000

Sale Price before VAT 3,00,000

Input tax credit 8,000

Goods sold in Delhi 3,00,000

Add: VAT @ 4% 12,000

Sale Price 3,12,000

(ii)

Purchased goods B from U.P. 4,00,000

Add: Central sales tax @ 2% 8,000

Purchase Price 4,08,000

Add: Profit (4,08,000 x 50%) 2,04,000

Sale Price before VAT 6,12,000

Input tax credit Nil

Goods sold in Delhi 6,12,000

Add: VAT @ 12.5% 76,500

Sale Price 6,88,500

(iii)

Purchased goods C from Delhi 8,00,000

Add: VAT @ 12.5% 1,00,000

Purchase Price 9,00,000

Cost 8,00,000

Add: Profit {9,00,000 – 1,00,000(as VAT credit is available)} x 50% 4,00,000

Sale Price before CST 12,00,000

Input tax credit 1,00,000

Goods sold in Orissa 12,00,000

Add: Central sales tax @ 2% 24,000

Page 87: sipoysatish@gmail · PDF file · 2013-08-07Explain the concept of Value Added Tax / Explain Taxonomy (process ... Explain accounting treatment of VAT as suggested by ICAI. ... Q26

Value Added Tax 87

Sale Price 12,24,000

(iv)

Purchased goods D from Delhi 10,00,000

Add: VAT @ 12.5% 1,25,000

Purchase Price 11,25,000

Cost 10,00,000

Add: Profit {11,25,000 – 1,25,000(as VAT credit is available)} x 50% 5,00,000

Sale Price before CST 15,00,000

Input tax credit 1,25,000

Goods sold in Punjab to unregistered dealer 15,00,000

Add: Central sales tax @ 12.5% 1,87,500

Sale Price 16,87,500

(v)

Purchased goods E from Madhya Pradesh 6,00,000

Add: Central sales tax @ 1% 6,000

Purchase Price 6,06,000

Add: Profit (6,06,000 x 50%) 3,03,000

Sale Price 9,09,000

Input tax credit Nil

Goods sold in Maharashtra 9,09,000

Add: Central sales tax @ 1% 9,090

Sale Price 9,18,090

(vi)

Purchased goods F from Delhi 14,00,000

Add: VAT @ 1% 14,000

Purchase Price 14,14,000

Cost 14,00,000

Add: Profit {14,14,000-14,000(as VAT credit is available)} x 50% 7,00,000

Sale Price 21,00,000

Input tax credit 14,000

Goods sold in Maharashtra to unregistered dealer 21,00,000

Add: Central sales tax @ 1% 21,000

Sale Price 21,21,000

(vii)

Purchased goods G from Delhi 12,00,000

Add: VAT @ 12.5% 1,50,000

Purchase Price 13,50,000

Goods Stock transferred 12,00,000

VAT credit allowed in stock transfer (12,00,000 x 10.5%) 1,26,000

(in case of stock transfer, VAT credit shall be allowed after retaining 2%)

(viii)

Purchased goods H from Delhi 16,00,000

Add: VAT @ 4% 64,000

Purchase Price 16,64,000

Cost 16,00,000

Add: Profit {16,64,000 – 64,000(as VAT credit is available)} x 50% 8,00,000

Page 88: sipoysatish@gmail · PDF file · 2013-08-07Explain the concept of Value Added Tax / Explain Taxonomy (process ... Explain accounting treatment of VAT as suggested by ICAI. ... Q26

Value Added Tax 88

Sale Price 24,00,000

Input tax credit 64,000

Goods exported 24,00,000

(ix)

Purchased goods I from Delhi 18,00,000

Add: VAT @ 12.5% 2,25,000

Purchase Price 20,25,000

Cost 18,00,000

Add: Profit {20,25,000-2,25,000(as VAT credit is available)} x 50% 9,00,000

Sale Price 27,00,000

Input tax credit 2,25,000

Goods sold to manufacturer in SEZ 27,00,000

VAT A/C

Particulars ` ` OUTPUT TAX VAT CST

Goods A 12,000 ---

Goods B 76,500 ---

Goods C --- 24,000

Goods D --- 1,87,500

Goods E --- 9,090

Goods F --- 21,000

Goods G (Stock transfer) Not applicable ---

Goods H (Export) Nil ---

Goods I (Sale to SEZ) Nil ---

88,500 2,41,590

LESS: INPUT TAX CREDIT

Goods A 8,000

Goods B Not allowed

Goods C 1,00,000

Goods D 1,25,000

Goods E Not allowed

Goods F 14,000

Goods G 1,26,000

Goods H 64,000

Goods I 2,25,000

6,62,000

After adjusting output VAT of `88,500 and CST of `2,41,590, there will be unutilised VAT credit of

`3,31,910 and it can be set off from other output tax or it can be carried forward or refund can be claimed but

procedure differs from State to State. At the year end it should be shown on the assets side of the balance

sheet under the head CURRENT ASSETS, LOAN AND ADVANCES.

Computation of Income Tax Liability

Particulars

Purchases

Amount

` Particulars

Sales

Amount

` Goods A 2,00,000 Goods A 3,00,000

Goods B 4,08,000 Goods B 6,12,000

Page 89: sipoysatish@gmail · PDF file · 2013-08-07Explain the concept of Value Added Tax / Explain Taxonomy (process ... Explain accounting treatment of VAT as suggested by ICAI. ... Q26

Value Added Tax 89

Goods C 8,00,000 Goods C 12,00,000

Goods D 10,00,000 Goods D 15,00,000

Goods E 6,06,000 Goods E 9,09,000

Goods F 14,00,000 Goods F 21,00,000

Goods H 16,00,000 Goods H 24,00,000

Goods I 18,00,000 Goods I 27,00,000

Net profit 39,07,000

1,17,21,000 1,17,21,000

Income under the head Business/Profession 39,07,000.00

Gross Total Income 39,07,000.00

Less: Deduction u/s 80C to 80U Nil

Total Income 39,07,000.00

Tax on `39,07,000 at slab rate 10,02,100.00

Add: Education cess @ 2% 20,042.00

Add: SHEC @ 1% 10,021.00

Tax Liability 10,32,163.00

Rounded off u/s 288B 10,32,160.00

Income shall be computed exclusive of VAT & CST because any VAT & CST collected shall be paid to the

Government and it will not be considered to be income. Similarly VAT paid by the dealer is collected from

the customer hence it will not be considered to be expense. Further, the stock transfer of goods G is having a

neutral effect and thus ignored for calculation of business/profession income.

Solution 7:

Raw material – R1

Assessable value 1,50,000

Excise duty @ 8% 12,000

EC @ 2% 240

SHEC@ 1% 120

Total 1,62,360

Delhi VAT @ 10% - Input Tax 16,236

Purchase Price 1,78,596

Raw material – R2

Assessable value 2,20,000

Excise duty @ 4% 8,800

EC @ 2% 176

SHEC@ 1% 88

Total 2,29,064

Central Sales tax @ 2% - Input Tax 4,581

Purchase Price 2,33,645

Raw material – R3

Assessable value 4,50,000

Excise duty @ 8% 36,000

EC @ 2% 720

SHEC@ 1% 360

Total 4,87,080

Delhi VAT @ 10% - Input Tax 48,708

Purchase Price 5,35,788

Page 90: sipoysatish@gmail · PDF file · 2013-08-07Explain the concept of Value Added Tax / Explain Taxonomy (process ... Explain accounting treatment of VAT as suggested by ICAI. ... Q26

Value Added Tax 90

Cost of Final Product

Raw material - R1 1,50,000

Raw material - R2 2,24,581

Raw material - R3 4,50,000

Processing charges 6,00,000

Payment for Services 2,00,000

Profit 50,000

Assessable value (as per section 4 of Central Excise Act, 1944) 16,74,581

Excise duty @ 16% 2,67,933

EC @ 2% 5,359

SHEC@ 1% 2,679

Total 19,50,552

Delhi VAT @ 10% - Output Tax 1,95,055

21,45,607

CENVAT/VAT ACCOUNT

Excise Duty / Service Tax

` EC @ 2%

` SHEC @ 1%

` Delhi VAT

` (Invoice 1) Raw material – R1 12,000.00 240.00 120.00 16,236.00 (Invoice 2) Raw material – R2 8,800.00 176.00 88.00 - (Invoice 3) Raw material – R3 36,000.00 720.00 360.00 48,708.00 Service tax 24,000.00 480.00 240.00 Total 80,800.00 1,616.00 808.00 64,944.00

Final product

Output tax 2,67,933.00 5,359.00 2,679.00 1,95,055.00

Less: VAT/CENVAT Credit (80,800.00) (1,616.00) (808.00) (64,944.00)

Net tax payable 1,87,133.00 3,743.00 1,871.00 1,30,111.00

Solution 8: ` Computation of VAT payable

Raw material –R1

Purchase price 3,00,000.00

Add: Excise duty @ 10% 30,000.00

Add: Education cess @ 2% 600.00

Add: SHEC @ 1% 300.00

3,30,900.00

Add: Delhi VAT @ 10% 33,090.00

3,63,990.00

Raw material –R2

Purchase price 4,00,000.00

Add: Excise duty @12% 48,000.00

Add: Education cess @ 2% 960.00

Add: SHEC @1% 480.00

4,49,440.00

Add: CST @ 2% 8,988.80

4,58,428.80

Capital goods

Purchase price 12,00,000.00

Add: Excise duty @10% 1,20,000.00

Add: Education cess @ 2% 2,400.00

Add: SHEC @1% 1,200.00

Page 91: sipoysatish@gmail · PDF file · 2013-08-07Explain the concept of Value Added Tax / Explain Taxonomy (process ... Explain accounting treatment of VAT as suggested by ICAI. ... Q26

Value Added Tax 91

13,23,600.00

Add: Delhi VAT @ 4% 52,944.00

13,76,544.00

Services 4,00,000.00

Service Tax @ 12% 48,000.00

Add: Education cess @ 2% 960.00

Add: SHEC @1% 480.00

4,49,440.00

Cost of final product

Raw material –R1 3,00,000.00

Raw material –R2 4,08,988.80

Capital goods (12,00,000 @ 20%) 2,40,000.00

Services 4,00,000.00

Other processing charges 6,00,000.00

Profit 4,00,000.00

Assessable Value 23,48,988.80

Add: Excise duty @12% 2,81,878.66

Add: Education cess @ 2% 5,637.57

Add: SHEC @ 1% 2,818.78

26,39,323.81

Add: Delhi VAT @ 10% 2,63,932.38

CENVAT/VAT ACCOUNT

Excise Duty / Service Tax EC @ 2% SHEC @

1%

Delhi VAT /

CST

Raw material – R1 30,000 600 300 33,090

Raw material – R2 48,000 960 480 -

Plant and machinery 1,20,000 2,400 1,200 52,944

Service tax 48,000 960 480 -

Total 2,46,000 4,920 2,460 86,034

Final product

Output tax 2,81,878.66 5,637.57 2,818.78 2,63,932.38

Less: VAT/CENVAT

Credit

2,46,000.00 4,920.00 2,460.00 86,034.00

Net tax payable 35,878.66 717.57 358.78 1,77,898.38

Rounded Off 35,879.00 718.00 359.00 1,77,898.00

Solution 8(b):

In this case the manufacturer shall charge central sales tax on the sale instead of Delhi Value Added Tax.

Since CST shall also be paid to the Delhi Government, VAT credit shall be allowed in the normal manner

and it can be adjusted against output CST and tax treatment shall be as given below:

Computation of VAT payable

Raw material –R1

Purchase price 3,00,000.00

Add: Excise duty @ 10% 30,000.00

Add: Education cess @ 2% 600.00

Add: SHEC @ 1% 300.00

3,30,900.00

Page 92: sipoysatish@gmail · PDF file · 2013-08-07Explain the concept of Value Added Tax / Explain Taxonomy (process ... Explain accounting treatment of VAT as suggested by ICAI. ... Q26

Value Added Tax 92

Add: Delhi VAT @ 10% 33,090.00

3,63,990.00

Raw material –R2

Purchase price 4,00,000.00

Add: Excise duty @12% 48,000.00

Add: Education cess @ 2% 960.00

Add: SHEC @1% 480.00

4,49,440.00

Add: CST @ 2% 8,988.80

4,58,428.80

Capital goods

Purchase price 12,00,000.00

Add: Excise duty @10% 1,20,000.00

Add: Education cess @ 2% 2,400.00

Add: SHEC @1% 1,200.00

13,23,600.00

Add: Delhi VAT @ 4% 52,944.00

13,76,544.00

Services 4,00,000.00

Service Tax @ 12% 48,000.00

Add: Education cess @ 2% 960.00

Add: SHEC @1% 480.00

4,49,440.00

Cost of final product

Raw material –R1 3,00,000.00

Raw material –R2 4,08,988.80

Capital goods (12,00,000 @ 20%) 2,40,000.00

Services 4,00,000.00

Other processing charges 6,00,000.00

Profit 4,00,000.00

Assessable Value 23,48,988.80

Add: Excise duty @12% 2,81,878.66

Add: Education cess @ 2% 5,637.57

Add: SHEC @ 1% 2,818.78

26,39,323.81

Add: CST @ 2%- output tax 52,786.47

CENVAT/VAT ACCOUNT

Excise Duty / Service Tax EC @ 2% SHEC @

1%

Delhi VAT

Raw material – R1 30,000 600 300 33,090

Raw material – R2 48,000 960 480 -

Plant and machinery 1,20,000 2,400 1,200 52,944

Service tax 48,000 960 480 -

Total 2,46,000 4,920 2,460 86,034

Final product

Output tax 2,81,878.66 5,637.57 2,818.78 52,786.47

Less: VAT/CENVAT

Credit

2,46,000.00 4,920.00 2,460.00 86,034.00

Page 93: sipoysatish@gmail · PDF file · 2013-08-07Explain the concept of Value Added Tax / Explain Taxonomy (process ... Explain accounting treatment of VAT as suggested by ICAI. ... Q26

Value Added Tax 93

Net tax payable 35,878.66 717.57 358.78 -

Net credit balance - - - 33,247.53

Rounded Off 35,879.00 718.00 359.00 33,248.00

Solution 8 (c):

Since the goods have been exported, there will not be any output tax and cenvat credit/ VAT credit shall be

refunded.

CENVAT/VAT ACCOUNT

Excise Duty / Service Tax EC @ 2% SHEC @

1%

Delhi VAT

Raw material – R1 30,000 600 300 33,090

Raw material – R2 48,000 960 480 -

Plant and machinery 1,20,000 2,400 1,200 52,944

Service tax 48,000 960 480 -

Total 2,46,000 4,920 2,460 86,034

Output tax Nil Nil Nil Nil

Solution 9: ` Raw material 12,00,000

Excise duty @ 10% 1,20,000

EC @ 2% 2,400

SHEC@ 1% 1,200

Total 13,23,600

Delhi VAT @ 4% 52,944

13,76,544

(i)

Since 1/6th

of the stock has been transferred to some other state, VAT credit allowed for such stock transfer

shall be 2% (4% - 2%).

Tax paid (52,944 x 1/6) 8,824

VAT credit (8,824 / 4% x (4%-2%)) 4,412

(In case of stock transfer VAT credit is allowed after retaining 2%)

(ii)

Raw material 2,20,600

VAT (`52,944 x 1/6) 8,824

Processing charges and profit 1,00,000

Sale Price 3,29,424

VAT Nil

(Since output VAT is exempt, hence VAT credit for input VAT is not allowed and it will be added in the

cost)

(iii) No tax is payable in case of export but Tax credit will be allowed.

(iv)

Raw material 2,20,600

Processing charges and profit 1,00,000

Sale Price 3,20,600

Central sales tax @ 2% 6,412

3,27,012

(v)

Raw material 2,20,600

Page 94: sipoysatish@gmail · PDF file · 2013-08-07Explain the concept of Value Added Tax / Explain Taxonomy (process ... Explain accounting treatment of VAT as suggested by ICAI. ... Q26

Value Added Tax 94

Processing charges and profit 1,00,000

Sale Price 3,20,600

Delhi VAT @ 12.5% 40,075

3,60,675

VAT A/C

Particulars DVAT

` CST

` OUTPUT TAX

1/6th

raw material (Stock transfer) - -

1/6th

Final Product (Exempt from VAT) -

1/6th

Final product Exported - -

1/6th

Final product sold in some other State 6,412

1/6th

Final product sold in same State 40,075 -

Total 40,075 6,412

Less: INPUT TAX CREDIT

1/6th

raw material (Stock transfer) 4,412 -

1/6th

Final Product (Exempt from VAT) - -

1/6th

Final product Exported 8,824 -

1/6th

Final product sold in some other State 8,824 -

1/6th

Final product sold in same State 8,824 -

1/6th

Raw material lying in stock 8,824

Total 39,708 -

Tax Payable 367 6,412

Solution 10:

Computation of Turnover and VAT

Taxable Exempt

75% 25%

Cost of Raw Material Purchased 2,25,000.00 75,000.00

VAT @ 12.5% --- 9,375.00

Other material – Local Purchases 37,500.00 12,500.00

VAT @ 12.5% --- 1,562.50

Other Material - Interstate Purchases 61,200.00 20,400.00

Manufacturing expenses 36,900.00 12,300.00

Cost of Product 3,60,600.00 1,31,137.50

Selling Price

(3,60,600 x 100% / 80%)/(1,31,137.50 x 100% / 80%) 4,50,750.00 1,63,921.88

VAT @ 12.5% 56,343.75 Nil

Invoice value of sale 5,07,093.75 1,63,921.88

Computation of VAT Payable

Output VAT 56,343.75

Less: Input VAT credit

Raw Material (37,500 x 75%) 28,125.00

Other raw Material (6,250 x 75%) 4,687.50

Net VAT Payable 23,531.25

Rounded off 23,531.00

Solution 11:

Page 95: sipoysatish@gmail · PDF file · 2013-08-07Explain the concept of Value Added Tax / Explain Taxonomy (process ... Explain accounting treatment of VAT as suggested by ICAI. ... Q26

Value Added Tax 95

` Computation of VAT payable by Bhim for the month of February’ 2013

Raw material purchased from another State

Purchase Price 12,00,000

Add: CST @ 2% 24,000

Total purchase price 12,24,000

Raw material X purchased within the State

Purchase Price 18,00,000

Add: VAT @ 1% 18,000

Raw material Y imported from Singapore

Purchase Price 13,00,000

Raw material Z purchased within the State

Purchase Price 8,00,000

Add: VAT @ 12.5% 1,00,000

Sale of goods produced from raw material X.

Sale Price 30,00,000

Add: VAT @ 4% 1,20,000

Sale of goods produced from inter-State purchase and imported raw materials.

Sale Price 34,00,000

Add: VAT @ 1% 34,000

Sale of goods produced from raw material Z.

Sale Price 12,00,000

Add: VAT @ 12.5% 1,50,000

Net Tax payable

Output tax (1,20,000 + 34,000 + 1,50,000) 3,04,000

Less: Tax credit (18,000 + 1,00,000) 1,18,000

Net tax payable 1,86,000

Solution 12:

` (i) Gross Product Variant

Raw material purchased 9,00,000

Add: VAT @ 4% 36,000

Sale price 15,00,000

Add: VAT @ 4% 60,000

Plant and machinery

Purchase price 4,00,000

Add: VAT @ 4% 16,000

Net tax payable

Page 96: sipoysatish@gmail · PDF file · 2013-08-07Explain the concept of Value Added Tax / Explain Taxonomy (process ... Explain accounting treatment of VAT as suggested by ICAI. ... Q26

Value Added Tax 96

Output tax 60,000

Less: Tax credit on raw material 36,000

Net tax payable 24,000

(ii) Consumption Variant

Net tax payable

Output tax 60,000

Less: Tax credit on raw material 36,000

Less: Tax credit on plant and machinery 16,000

Net tax payable 8,000

Consumption Variant is beneficial to the dealer

Solution 13:

Computation of VAT liability of Kapil & Co. for the year ended 31.03.2013:-

Particulars Amount

(`) Input tax credit:

Intra-State purchases of 1000 units of raw materials

106

6000,42,7 42,000

Inter-State purchases of raw materials --

Import of raw materials --

Purchase of Capital Goods

2110

10000,50,5 25,000

Other manufacturing expenses --

Total input tax credit available: 67,000

Output VAT payable:

Sale of taxable goods within State [(10,40,000 x 4)/104] 40,000

Sale of exempted goods within State --

VAT credit to be carried forward (40,000 – 67,000) (27000)

Notes:- 1. VAT paid on purchase of capital goods is eligible for input tax credit. However, the same has to be

spread over a period of two years.

2. VAT system allows credit in respect of purchases made during a period to be set-off against the

taxable sales during that period, irrespective of when the supplies/inputs purchased are utilized/sold.

Therefore, input tax credit in respect of closing stock of raw materials need not be reduced from total

input tax credit available.

Note: The statement in the question, “Input credit is allowed only on raw materials used in manufacture of

the taxable goods”, implies that the same is not allowable in respect of sale of goods within the State which

are exempt from levy of VAT.

Solution 14:

Computation of VAT liability of Rahul:-

Particulars ` `

Page 97: sipoysatish@gmail · PDF file · 2013-08-07Explain the concept of Value Added Tax / Explain Taxonomy (process ... Explain accounting treatment of VAT as suggested by ICAI. ... Q26

Value Added Tax 97

Raw materials purchased from foreign market (including basic custom duty 1,36,500

@ 20% plus EC)

Raw material purchased from local market:-

Cost of raw material 3,50,000

Add: Excise duty @ 16% 56,000 4,06,000

Raw material purchased from neighbouring State (including CST @ 2%) 56,100

Storage and transportation cost 14,000

Manufacturing expenses 36,000

Cost of production 6,48,600

Add: Profit @ 12% of cost of production 77,832

Sale Price 7,26,432

VAT @ 4% on `7,26,432 29,057

Net VAT liability of Rahul:- VAT on sale price 29,057

Less: Input tax credit

Basic custom duty paid on imports Nil

CST paid on inter-state purchases Nil

VAT paid on local purchases 16,240

Net VAT payable by Rahul 12,817

Solution 15:

Computation of VAT Liability of Mr. Pankaj Ahuja for the month of October 2012 using 'invoice

method' of computation of VAT:

Particulars ` Purchase price (including VAT @ 4%) 81,120

Less: VAT paid on purchases (81,120 × 4 / 104) 3,120

Add: Storage cost 2,800

Add: Transportation cost 3,200

Cost Price 84,000

Add: Profit @ 5% of cost price 4,200

Sale price before VAT 88,200

VAT @ 12.5% (` 88,200 × 12.5%) 11,025

Less: VAT paid on purchases 3,120

VAT Liability of Mr. Pankaj Ahuja 7,905

Solution 16:

` (a) Computation of Amount of Eligible Purchases for Input Tax Credit

Total Purchases 100,00,000.00

Less: Ineligible Purchases

Purchase From UP 25,00,000

Purchase From Unregistered Dealer 1,00,000

Capital goods not eligible 10,00,000 36,00,000.00

Eligible Purchases for Input Tax Credit 64,00,000.00

(b) Computation of Input Tax Credit for the month of March 2013

Capital Goods ( 20,00,000 x 12.50%/36) 6,944.44

Inputs (44,00,000 x 4%) 1,76,000.00

Page 98: sipoysatish@gmail · PDF file · 2013-08-07Explain the concept of Value Added Tax / Explain Taxonomy (process ... Explain accounting treatment of VAT as suggested by ICAI. ... Q26

Value Added Tax 98

Input Tax Credit 1,82,944.44

(c) Computation of VAT Payable for the month of March 2013

Output Tax (50,00,000 x 12.5%) 6,25,000.00

Less: Input Tax Credit 1,82,944.44

VAT Payable 4,42,055.56

Solution 17:

Computation of VAT Liability of Mr. X:-

Particulars ` Raw materials purchased from foreign market (including 10% import duty) 5,00,000

Raw material purchased from Delhi including excise duty 7,00,000

Raw material purchased from UP 2,00,000

Transportation and Manufacturing expenses 1,00,000

Cost of production 15,00,000

Add: Profit @ 20% on sale price 3,75,000

(15,00,000 / 80% x 100%) x 20%

Sale Price 18,75,000

VAT @ 12.50% on `18,75,000 2,34,375

Net VAT Liability of Mr. X:- VAT on sale price 2,34,375

Less: Input tax credit

Import duty paid on imports Nil

VAT paid on local purchases (7,00,000 X 4%) (28,000)

Net VAT Liability of Mr. X 2,06,375

EXAMINATION QUESTIONS

IPCC NOV – 2012 Question No. 1(c) (5 Marks)

The following are details of purchases, sales, etc. effected by Varadan & Co., a registered dealer under VAT

and CST Act, for the year ended 31.03.2013:

Particulars Amount

(`) Purchase of raw materials within State (500 units, inclusive of VAT levy at 12.5%) 11,25,000

Inter-State purchases of raw materials, inclusive of CST at 2%. 4,08,000

Page 99: sipoysatish@gmail · PDF file · 2013-08-07Explain the concept of Value Added Tax / Explain Taxonomy (process ... Explain accounting treatment of VAT as suggested by ICAI. ... Q26

Value Added Tax 99

Import of packing material, inclusive of customs duty of `10,000 2,10,000

Capital goods purchased on 01.04.2012 of VAT levy at 10% (inclusive)

(input credit to be spread over 2 financial years) 5,50,000

Sales of taxable goods within State inclusive of VAT levy at 4% 40,24,000

Sales of goods within State, exempt from levy of VAT (Goods were manufactured

from the Inter-State purchase of raw materials) 1,20,000

Compute the VAT liability of the dealer for the year ended 31.03.2013. (Modified)

Solution:

Computation of Tax Credit

Particular Amount(in `) Purchase of raw material within state (11,25,000 X 12.5/112.5) 1,25,000

Interstate Purchase (No credit on CST Paid goods) ------

Import of Packing Material ( No credit for import duty) ------

Capital goods credit (5,50,000 X 10/110)/2 25,000

Total input VAT credit 1,50,000

Computation of output VAT

Particular Amount (in `) Sale of goods within state (40,24,000 X 4/104) 1,54,769

Sale of exempted goods -----

Total output VAT 1,54,769

Computation of VAT Payable

Particulars Amount(in `) Output VAT 1,54,769

Less: Input VAT 1,50,000

VAT Payable 4,769

Question No. 2(c) (4 Marks)

How is VAT computed under the subtraction method?

Solution:

Tax credit method or invoice method is the most common method of VAT, however, another method to

determine VAT liability is the Subtraction Method and under this method, Tax is charged on the difference

between the sale price and purchase price. If the difference is exclusive of VAT, it is called Direct

Subtraction and if it is inclusive of VAT, it is called Intermediate Subtraction. Since the total value of goods

sold is not taken into account, Tax credit is not allowed and VAT is charged only on the difference between

sale price and purchase price. This method is normally applied where Tax is not charged separately.

Intermediate Subtraction can be shown as given below:

Page 100: sipoysatish@gmail · PDF file · 2013-08-07Explain the concept of Value Added Tax / Explain Taxonomy (process ... Explain accounting treatment of VAT as suggested by ICAI. ... Q26

Value Added Tax 100

Stage

No.

Particulars Turnover for tax

under VAT

(`)

Tax @ 12.50%

(`)

1. First Seller sells the goods to a Distributor at say,

` 1,125 inclusive of tax

1,125 125

(1,125 x 12.50/112.5)

2. Distributor sells the goods to a Whole-seller at

say, `1,350. Here taxable turnover will be ` 1,350

– ` 1,125

225 25

(225 x 12.50/112.5)

3. Wholesaler sells the goods to a Retailer at say ` 1,687.50. Here Taxable turnover will be

`1,687.50 – `1,350

337.50 37.50

(337.50 x 12.50/112.5)

4. Retailer selling the goods at say, ` 2,250.

Taxable turnover will be ` 2,250 – ` 1,687.50

562.50 62.50

(562.50 x 12.50/112.5)

In case of Direct Subtraction, VAT Shall be computed on the difference of sale and purchase exclusive of

VAT.

The demerit of this method is that there is no confirmation of the fact that the dealer has paid VAT at the

earlier stage, hence there may be Tax evasion.

Question No. 3(c) (4 Marks)

Enlist any six purchases eligible for availing input tax credit.

Solution:

Eligible purchase for input tax credit

For the purpose of claiming the input tax credit, taxable goods should be purchased for any of the following

purposes –

(i) for sale within State;

(ii) for sale in course of inter-State trade or commerce;

(iii) for being used in execution of a works contract;

(iv) to be used as capital goods required for the purpose of manufacture and resale of taxable goods;

(v) for making zero rated sale i.e. export sale;

(vi) to be used as containers or packing material, raw material, consumable stores, manufacture of taxable

goods or in the packing of such manufactured goods (intended for sale in the State or in the course of inter-

State trade or commerce).

Question No. 4(c) (4 Marks)

What are the conditions to be fulfilled by the dealer accepting the composition scheme under the value

added tax?

Solution:

Composition scheme for small traders

(Similar provision in Section 16 of Delhi Value Added Tax Act, 2004)

If any dealer is having turnover UPTO ` 50 LAKHS, he may apply for Composition Scheme.

Page 101: sipoysatish@gmail · PDF file · 2013-08-07Explain the concept of Value Added Tax / Explain Taxonomy (process ... Explain accounting treatment of VAT as suggested by ICAI. ... Q26

Value Added Tax 101

Composition Scheme is not allowed in the following cases:

(i) If any dealer is procuring goods from outside the State or is selling or supplying goods to any place

outside the State at any time during the year.

(ii) If he is registered under Central Sales Tax Act.

Salient features of the Scheme are asunder:

(i) A dealer covered under Composition Scheme is not allowed to take VAT Credit on his purchases but he

must retain all the tax invoices for the goods purchased by him.

(ii) A dealer covered under composition scheme is not allowed to issue tax invoice and also not allowed to

charge any tax from the buyer rather he himself has to pay tax on his sales turnover (@ 1% in Delhi).

(iii) The benefit of composition scheme is that dealer is exempt from maintaining lengthy records required

under VAT.

(iv) If any dealer is purchasing goods from a dealer covered under composition scheme, no VAT credit is

allowed to such a purchasing dealer.

(v) If any dealer is covered under composition scheme, he must purchase goods only from registered dealer.

(vi) If turnover has exceeded `50 lakhs, he has to shift immediately to the normal system.

(vii) If any dealer is covered under composition scheme, he may reject the scheme and may opt for normal

procedure but only from beginning of the year.

(viii) If any dealer is opting out of the composition scheme, he will be allowed VAT credit for the stock held

by him on the date of opting out.

A dealer has to apply in the prescribed form for opting the scheme and rejecting the scheme

A dealer opting for composition scheme has to apply in Form No. DVAT 01 under DVAT Act, 2004

Question No. 5(c) (4 Marks)

What is meant by input tax credit in the context of VAT provisions? How does input tax credit help in

achieving the essence of VAT?

Solution:

The tax paid by a registered dealer at the earlier point is called input tax. This amount is adjusted/rebated

against the tax payable by the purchasing dealer on his sales. This credit availability is called input tax credit

(ITC). It can also be referred to as tax credit on a sale within the State or in the course of inter-State trade or

commerce.

The essence of VAT is in providing set-off for the tax paid earlier, and this is given effect through the

concept of input tax credit/rebate. In this manner, there is no multiple taxation and also there is no cascading

effect and in this manner, input tax credit helps in achieving the essence of VAT.

Question No. 6(c) (4 Marks)

Discuss the compulsory and voluntary registration under VAT.

Page 102: sipoysatish@gmail · PDF file · 2013-08-07Explain the concept of Value Added Tax / Explain Taxonomy (process ... Explain accounting treatment of VAT as suggested by ICAI. ... Q26

Value Added Tax 102

Solution:

Compulsory Registration

A dealer must apply for registration in the following cases:

(i) If the turnover has exceeded `10 lakhs at any time during the year.

(ii) He is registered under Central Sales Tax Act, 1956.

(iii) He is purchasing goods from outside the state for sale within the state.

Example

Mr. X is an unregistered dealer in Delhi,. His sales turnover is `5,00,000 but he is purchasing some of the

goods from outside Delhi, in this case, he should apply for compulsory registration.

If in the above case, he is not purchasing goods from other States but he is selling some of the goods to other

States, in that case also registration is required. If he is not purchasing goods from other States and also not

selling goods to other states, registration is not required but if turnover is exceeding `10,00,000, registration

is required.

Voluntary Registration

Any dealer may apply for voluntary registration under State Value Added Tax Act at any time.

Only dealer registered under State Value Added Tax Act can charge sales tax and can issue tax invoice and

VAT Credit is only allowed on the basis of tax invoice.

Question No. 7(c) (4 Marks)

Since the VAT system emphasizes on self assessment the need for a system of cross-checking has arisen,

Elaborate.

Solution:

System of Cross Checking

In the VAT system more emphasis has been laid on self-assessment. Hence, a system of cross-checking is

essential. Dealers may be asked to submit the list of sales or purchases above a certain monetary value or to

give the dealer-wise list from whom or to whom the goods have been purchased/sold for values exceeding a

prescribed monetary ceiling.

A cross-checking computerized system is being worked out on the basis of coordination between the tax

authorities of the State Government and the authorities of Central Excise and Income-tax to compare

constantly the tax returns and set-off documents of VAT system of the States and those of Central Excise

and Income-tax. This comprehensive cross-checking system will help reduce tax evasion and also lead to

significant growth of tax revenue. At the same time, by protecting the interests of tax-complying dealers

against the unfair practices of tax-evaders, the system will also bring in more equal competition in the sphere

of trade and industry.

IPCC MAY – 2012 Question 1 (5 Marks)

R. Ltd of Mumbai made a total purchases of input and capital goods of `60,00,000 during the month of

February, 2013. The following further information is available.

Page 103: sipoysatish@gmail · PDF file · 2013-08-07Explain the concept of Value Added Tax / Explain Taxonomy (process ... Explain accounting treatment of VAT as suggested by ICAI. ... Q26

Value Added Tax 103

(i) Goods worth `15,00,000 were purchased from Assam on which C.S.T @ 2% was paid.

(ii) The purchases made in February, 2013 include goods purchased from unregistered dealers

amounting to `18,50,000.

(iii) It purchased capital goods (not eligible for input credit) worth `6,50,000 and those eligible for

input credit for `9,00,000.

(iv) Sales made in Mumbai during the month of February, 2013 is `10,00,000 on which VAT at

12.5% is payable.

Assuming that all purchases given are exclusive of tax and VAT @ 4% is paid on them.

Calculate:

(a) the amount of purchases eligible for input credit.

(b) the amount of input credit available for the month of February, 2013.

(c) the VAT payable for the month of February, 2013.

The input VAT credit on eligible capital goods is available in 36 equal monthly instalments. (Modified)

Answer:

` (a) Computation of Amount of Eligible Purchases for Input Tax Credit

Total Purchases 60,00,000

Less: Ineligible Purchases

Purchase From Assam 15,00,000

Purchase From Unregistered Dealer 18,50,000

Capital goods not eligible 6,50,000 40,00,000

Eligible Purchases for Input Tax Credit 20,00,000

(b) Computation of Input Tax Credit for the month of February 2013

Capital Goods (9,00,000 x 4%/36) 1,000

Inputs (11,00,000 x 4%) 44,000

Input Tax Credit 45,000

(c) Computation of VAT Payable for the month of February 2013

Output Tax ( 10,00,000 x 12.5%) 1,25,000

Less: Input Tax Credit 45,000

VAT Payable 80,000

Note:

1. Total purchases are `60,00,000 but details are given only for `49,00,000, hence remaining `11,00,000 are

presumed to be raw material eligible for tax credit.

Question 2(c) (4 Marks)

Explain the consumption variant of VAT. Mention the reasons for the preference of this variant of VAT.

Answer:

Page 104: sipoysatish@gmail · PDF file · 2013-08-07Explain the concept of Value Added Tax / Explain Taxonomy (process ... Explain accounting treatment of VAT as suggested by ICAI. ... Q26

Value Added Tax 104

It allows VAT credit on raw materials etc. and also on capital goods in the very first year.

Among the three variants of VAT, the consumption variant is widely used. Several countries of Europe and

other continents have adopted this variant, because there is no multiple taxation and also there is no

cascading effect.

Consumption variant of VAT allows for deduction on all business purchases including capital assets. Thus,

gross investment is deductible in calculating value added. It neither distinguishes between capital and

current expenditures nor specifies the life of assets or depreciation allowances for different assets.

Question 3 (8 x ½ = 4 Marks)

Test the veracity (truthfulness) of the following assertions with reference to the statutory provisions relating

to value added tax. Do not assign any reason for them.

(a) Input credit under VAT is available in respect of Central Sales Tax paid on purchases.

(b) VAT is leviable at the first stage of sale.

(c) Input credit is available in respect of customs duty paid on goods imported from a country outside

India.

(d) Input credit is available only if the purchaser has obtained proper tax invoice.

(e) No registration is required under any VAT regime.

(f) A trader can take credit of the inputs purchased by him only if he has obtained proper tax invoice

from the supplier.

(g) VAT is inflationary in nature.

(h) White paper on State level VAT provides a framework for drafting various State VAT legislations.

(Modified)

Answer:

(a) False

(b) False

(c) False

(d) True

(e) False

(f) True

(g) False

(h) True

Question 4 (4 Marks)

Determine the liability of VAT of X for the month of December 2012 using invoice method of computation

from the following data:

Purchase price of goods acquired from local market (including VAT) ` 52 lakhs

VAT rate on input 4%

Transportation, insurance, warehousing and handling cost incurred by X ` 20,000

Page 105: sipoysatish@gmail · PDF file · 2013-08-07Explain the concept of Value Added Tax / Explain Taxonomy (process ... Explain accounting treatment of VAT as suggested by ICAI. ... Q26

Value Added Tax 105

Goods sold at a profit margin on cost of production 14%

VAT rate on sales 12.50%

(Modified)

Answer:

Computation of Sales Value

` Local purchases of raw material (52,00,000/104 x 100) 50,00,000

Transportation, insurance, warehousing and handling cost 20,000

Cost of production 50,20,000

Add: Profit margin 14% 7,02,800

57,22,800

Add: VAT @ 12.5% 7,15,350

64,38,150

Computation of VAT Liability:- ` VAT on above sales price @ 12.5% 7,15,350

Less: Set off of VAT on purchases (52,00,000/104 x 4) 2,00,000

Net VAT Liability 5,15,350

Question 5 (2 Marks)

(i) What are the different variants of VAT and how is deduction available for tax paid on inputs including

capital inputs?

(2 Marks) (ii) What are the different stages of VAT? Can it be said that the entire burden falls on the final consumer?

Answer:

(i)

There are the different Variants of VAT:

(a) Gross Product Variant

(b) Income Variant

(c) Consumption Variant

(a) Gross Product Variant

Gross Product Variant allows VAT credit on the raw materials, but tax credit is not allowed on capital goods

like plant and machinery etc.

(b) Income Variant of VAT The Income Variant of VAT allows VAT credit on raw materials etc and also on capital goods but VAT

credit on capital goods is allowed in instalments depending on the life of capital goods.

(c) Consumption Variant

It allows VAT credit on raw materials etc. and also on capital goods in the very first year.

(ii) Value Added Tax is a Multi Stage tax and is being charged at every stage of sale and these stages are

called stages of VAT and are as given below:

1. Manufacturer to Distributor

2. Distributor to Wholesaler

3. Wholesaler to Retailer

4. Retailer to Consumer

Page 106: sipoysatish@gmail · PDF file · 2013-08-07Explain the concept of Value Added Tax / Explain Taxonomy (process ... Explain accounting treatment of VAT as suggested by ICAI. ... Q26

Value Added Tax 106

The Value Added Tax (VAT) is a multistage tax levied as a proportion of the value added (i.e. sale minus

purchase) which is equivalent to wages plus interest, other costs and profits. In an economy, apart from the

manufacturers and final consumers, there would be wholesalers and retailers also. VAT is collected at each

stage of production and distribution process, and in principle, its entire burden falls on the final consumer,

who does not get any tax credit. Thus, VAT is a broad based tax covering the value added to each

commodity by parties during the various stages of production and distribution.

Question 6 (4 Marks)

Mr. X is registered under local VAT of Kerala and he submits the following information. Compute the net

VAT liability from the following information:

` Import of raw material (including 10% import duty) 1,10,000

Raw material purchased from Kerala (including excise duty @ 12%) 2,24,000

VAT @ 4% on the above purchase

Raw material purchased from Karnataka 85,000

Transportation and manufacturing expenses 47,000

Mr. X sold entire stock to Nishu at a profit of 10% on the cost of production. VAT rate on such sale is 4%.

(Modified)

Answer:

Computation of VAT Liability of Mr. X:-

Particulars ` Raw materials purchased from foreign market (including 10% import duty ) 1,10,000

Raw material purchased from Kerala including excise duty 2,24,000

Raw material purchased from Karnataka 85,000

Transportation and Manufacturing expenses 47,000

Cost of production 4,66,000

Add: Profit @ 10% of cost of production 46,600

Sale Price 5,12,600

VAT @ 4% on `5,12,600 20,504

Net VAT Liability of Mr. X:- VAT on sale price 20,504

Less: Input tax credit

Import duty paid on imports Nil

VAT paid on local purchases (2,24,000 X 4%) (8,960)

Net VAT Liability of Mr. X 11,544

Question 7 (4 Marks)

Briefly explain the system of cross checking under VAT Act.

Answer:

In the VAT system more emphasis has been laid on self-assessment. Hence, a system of cross-checking is

essential. Dealers may be asked to submit the list of sales or purchases above a certain monetary value or to

give the dealer-wise list from whom or to whom the goods have been purchased/sold for values exceeding a

prescribed monetary ceiling.

Page 107: sipoysatish@gmail · PDF file · 2013-08-07Explain the concept of Value Added Tax / Explain Taxonomy (process ... Explain accounting treatment of VAT as suggested by ICAI. ... Q26

Value Added Tax 107

A cross-checking computerized system is being worked out on the basis of coordination between the tax

authorities of the State Government and the authorities of Central Excise and Income-tax to compare

constantly the tax returns and set-off documents of VAT system of the States and those of Central Excise

and Income-tax. This comprehensive cross-checking system will help reduce tax evasion and also lead to

significant growth of tax revenue. At the same time, by protecting the interests of tax-complying dealers

against the unfair practices of tax-evaders, the system will also bring in more equal competition in the sphere

of trade and industry.

PCC MAY – 2012 Question 1 (5 Marks)

ABC & Co purchased raw material ‘A’ for `30,00,000 plus VAT at 12.5%. out of such raw material 80%

was used for manufacture of taxable goods and the balance for the manufacture of exempted goods.

Another raw material ‘B’ was purchased for `20,00,000 on which VAT was paid @ 1%. Out of the raw

material ‘B’, 50% was used for manufacture of taxable goods and balance for the manufacture of exempt

goods.

The entire taxable goods were sold for `44,00,000 plus VAT at 12.5%. There was no opening or closing

inventory of taxable goods or raw materials.

Compute the VAT liability of ABC & Co. (Modified)

Answer:

` Computation of VAT Liability of ABC & CO.

Output Tax [44,00,000 x 12.5%] 5,50,000

Less: Input Tax Credit

Raw Material A [30,00,000 x 80% x12.5%] 3,00,000

Raw Material B [20,00,000 x 50% x1%] 10,000 3,10,000

VAT Liability 2,40,000

Note: Input tax credit on raw material used for manufacture of exempted goods is not available.

Question 3 (4 Marks)

Lee traders a registered dealer having stock of goods cost `30,000 purchased from outside the State, wishes

to opt for the Composition Scheme. Advise the dealer whether it is possible?

State the conditions to be satisfied by a dealer before opting for composition scheme.

Answer:

No it is not possible for dealer to opt for composition scheme because If any dealer is procuring goods from

outside the State or is selling or supplying goods to any place outside the State at any time during the year.

There are the following conditions to be satisfied by a dealer before opting for composition scheme:

Composition Scheme is not allowed in the following cases:

(i) If any dealer is procuring goods from outside the State or is selling or supplying goods to any place

outside the State at any time during the year.

(ii) If he is registered under Central Sales Tax Act.

Page 108: sipoysatish@gmail · PDF file · 2013-08-07Explain the concept of Value Added Tax / Explain Taxonomy (process ... Explain accounting treatment of VAT as suggested by ICAI. ... Q26

Value Added Tax 108

Question 5 (4 Marks)

What are the different rates under VAT system ?

Answer:

Exempted Category

Under exempted category, there are about 50 commodities comprising of natural and unprocessed products

in unorganized sector, items which are legally barred from taxation and items which have social

implications. Included in this exempted category is a set of maximum of 10 commodities flexibly chosen by

individual States from a list of goods (finalized by the Empowered Committee) which are of local social

importance for the individual States without having any inter-State implication.

Example:

(i) Books, periodicals and journals including maps, charts and globes.;(ii) Curd, Lussi, butter milk and

separated milk.; (iii) Earthen pot.; (iv) Electricity energy; (v) Fresh plants, saplings and fresh flowers.; (vi)

Fresh vegetables and fruits.; (vii) All bangles except those made of precious metals.; (viii) Kumkum, bindi

alta and sindur.; (ix) Blood including blood components

1% Category

The special rate of 1% is meant for precious stones, bullion, gold and silver ornaments etc.

4% VAT Category

The goods declared as per section 14 of Central Sales Tax Act shall be taxable @ 4%

List of some of the goods is:

(i) Coal including coke in all its forms, but excluding charcoal, (ii) Cotton yarn, but not including cotton

yarn waste; (iii) Oil seeds; (iv) Pulses; (v) Sugar; (vi) Iron and steel; (vii) Liquefied petroleum gas for

domestic use

5% VAT Category

Under 5% VAT rate category, there are largest number of goods, common for all the States, comprising of

items of basic necessities such as medicines and drugs, all agricultural and industrial inputs, capital goods

and declared goods. The schedule of commodities are attached to the VAT Acts of the States.

List of some of the goods is:

1. Bicycles having MRP above `3,500, Tricycles, rickshaws and parts including tyres and tubes thereof; 2.

Drugs & medicines including vaccines, syringes and dressings, medicated ointments produced under a drugs

licence; 3. Coffee beans and seeds, green tea leaf and chicory; 4. Cotton and cotton waste; 5. Edible oils and

oil cake.

20% Category

Petrol, diesel, Aviation Turbine Fuel, other motor spirit, liquor and lottery tickets etc. will be subjected to

20% floor rate of tax.

12.5% Category

The remaining commodities, common for all the States, fall under the general VAT rate of 12.5%.

Question 7 (4 Marks) What records should be maintained under VAT system by a registered dealer?

Answer:

The following records should be maintained under VAT system:

Page 109: sipoysatish@gmail · PDF file · 2013-08-07Explain the concept of Value Added Tax / Explain Taxonomy (process ... Explain accounting treatment of VAT as suggested by ICAI. ... Q26

Value Added Tax 109

1. Purchase records, showing details of purchases on which tax has been paid, purchases made without

payment of tax, purchases made from an exempted unit (Military Canteen) and purchases made from

outside State.

2. Sales records, showing separately sales made at different tax rates, zero-rated taxable sales and tax-free

sales.

3. VAT account - A monthly account specifying total output tax, total input tax and net tax payable or the

excess tax credit due for carry forward.

4. Details of input tax calculations where the dealer is making both taxable and tax free sales.

5. Stock records showing stock receipts and deliveries and manufacturing records.

6. Stock records showing separately the particulars of goods stored in cold storage, warehouse, godown or

any other place taken on hire.

7. Order records and delivery challan, wherever applicable.

8. Annual accounts including trading, profit and loss accounts and the balance sheet.

9. Bank records, including statements, cheque book counter foils and pay-in-slips.

10. Cash book, daybook and ledger.

IPCC NOV – 2011 Question 1 (5 Marks) Laxman, a registered dealer under DVAT /CST Act submits the following information for the month of

February, 2013.

Particulars Amount

` Rate of

VAT

Details of purchase

Raw material purchased from another State (CST @ 2%).

Raw material X purchased within the State

Raw material Y imported from Singapore (includes custom duty paid @ 10%)

Raw material Z purchased within the State.

10,00,000

15,00,000

11,00,000

6,00,000

1%

12.5%

Details of sales

Sale of goods produced from raw material X.

Sale of goods produced from inter-State purchase and imported raw materials.

Sale of goods produced from raw material Z.

27,00,000

32,00,000

8,00,000

4%

1%

12.5%

Note: The purchase and sales figures given above do not include VAT/CST.

Assume that there was no opening or closing inventory. Compute the amount of Value Added Tax (VAT)

payable by Laxman for the month of February, 2013. (Modified)

Answer: ` Computation of VAT payable by Laxman for the month of February’ 2013

Raw material purchased from another State

Purchase Price 10,00,000

Page 110: sipoysatish@gmail · PDF file · 2013-08-07Explain the concept of Value Added Tax / Explain Taxonomy (process ... Explain accounting treatment of VAT as suggested by ICAI. ... Q26

Value Added Tax 110

Add: CST @ 2% 20,000

Total purchase price 10,20,000

Raw material X purchased within the State

Purchase Price 15,00,000

Add: VAT @ 1% 15,000

Raw material Y imported from Singapore

Purchase Price 11,00,000

Raw material Z purchased within the State

Purchase Price 6,00,000

Add: VAT @ 12.5% 75,000

Sale of goods produced from raw material X.

Sale Price 27,00,000

Add: VAT @ 4% 1,08,000

Sale of goods produced from inter-State purchase and imported raw materials.

Sale Price 32,00,000

Add: VAT @ 1% 32,000

Sale of goods produced from raw material Z.

Sale Price 8,00,000

Add: VAT @ 12.5% 1,00,000

Net Tax payable

Output tax (1,08,000 + 32,000 + 1,00,000) 2,40,000

Less: Tax credit (15,000 + 75,000) 90,000

Net tax payable 1,50,000

Question 2 (4 Marks)

Explain the role of chartered Accountants in proper compliance of VAT. (Any 4 points).

Answer.

Under the VAT system, trust has been reposed on tax payers, as there will be no regular assessment of all

VAT returns, but only a few VAT returns will be taken up for scrutiny assessment. In other cases, the return

filed by the trader will be accepted. It will not be also seen whether proper records have been maintained by

the trader.

As a consequence, a check on compliance becomes essential. Chartered Accountants can ensure tax

compliance by:-

(i) helping the client in systematic record keeping;

(ii) helping the client in interpretation of the provisions of VAT law, and

(iii) performing audit of VAT accounts.

(iv) reporting the under-assessment, if any, made by the dealer requiring additional payment or

(v) reporting any excess payment of tax warranting refund to the tax payers.

Page 111: sipoysatish@gmail · PDF file · 2013-08-07Explain the concept of Value Added Tax / Explain Taxonomy (process ... Explain accounting treatment of VAT as suggested by ICAI. ... Q26

Value Added Tax 111

Question 3 (4 Marks) State any two benefits and two drawbacks for a dealer who opts for composition scheme under VAT as per

White Paper.

Answer:

Benefits Drawback

1) Dealer opting for the scheme is not required to

maintain lengthy records.

1) Dealer will not be able to take the credit of the vat

paid by him on purchases.

2) It reduces the administrative cost to the dealer. 2) Dealer cannot pass on the credit of VAT, which

will increase the cost of the product and adversely

effect the business.

Question 4 (4 Marks)

State with reasons whether the following are true or false in the context of VAT as per White Paper:

(i) No declaration form is prescribed under VAT system.

(ii) Taxpayer’s Identification Number (TIN) is a 10 digit alpha numeral.

(iii) Self assessment concept on deemed basis is one of the important features of VAT.

(iv) Set off of input tax credit on capital goods is available only to manufacturers and not to traders.

Answer:

(i) The statement is true. In view of the fact that a lot of time and energy is wasted by the dealer in getting

the declaration forms from the Department, most of earlier forms have been dispensed with. There is no

declaration form prescribed under VAT.

(ii) The statement is false. Taxpayer’s identification Number (TIN) is a 11 digit numeral. The first two

characters represent State Code as used by the Union Ministry of Home Affairs and the next nine characters

will be different in different states.

(iii) The statement is true. Under VAT, a dealer assesses the VAT liability on his own and submits the

return. The dealer is deemed to be self-assessed on the basis of the return filed by him. If he does not receive

any notice proposing departmental audit of his books of account within the time-limit specified in the VAT

Act of the respective State.

(iv) The statement is false. As per the White Paper on VAT, set off of input tax credit on capital goods is

available to both manufacturers and traders.

Question 5 (4 Marks)

Ashok, purchased raw material ‘A’ for `30,00,000 plus VAT @ 4%. Out of such raw material 60% was used

for manufacture of taxable goods and the remaining for manufacture of goods which are exempt from VAT.

Another raw material ‘B’ was purchased for `15,00,000 on which VAT was paid @ 1%. Entire raw material

‘B’ was used for manufacture of taxable goods only.

The entire taxable goods were sold for `50,00,000 plus VAT @ 12.5%.

Compute VAT liability of Ashok on the assumption that there was no opening or closing inventory.

Page 112: sipoysatish@gmail · PDF file · 2013-08-07Explain the concept of Value Added Tax / Explain Taxonomy (process ... Explain accounting treatment of VAT as suggested by ICAI. ... Q26

Value Added Tax 112

Note: Ashok is not a dealer who opted for Composition Scheme. (Modified)

Answer: ` Raw Material ‘A’

Purchase price 30,00,000

Add: VAT @ 4% 1,20,000

Raw Material ‘B’

Purchase price 15,00,000

Add: VAT @ 1% 15,000

Sale price 50,00,000

Add: VAT @ 12.5% 6,25,000

Net tax payable

Output Tax 6,25,000

Less: Input tax credit Raw Material ‘A’ (1,20,000 x 60%) 72,000

Less: Input tax credit Raw Material ‘B’ 15,000

Net Tax Payable 5,38,000

Question 6 (4 Marks)

Briefly explain the benefits of the system of cross-checking under VAT as per White Paper.

Answer:

System of Cross Checking

In the VAT system more emphasis has been laid on self-assessment. Hence, a system of cross-checking is

essential. Dealers may be asked to submit the list of sales or purchases above a certain monetary value or to

give the dealer-wise list from whom or to whom the goods have been purchased/sold for values exceeding a

prescribed monetary ceiling.

A cross-checking computerized system is being worked out on the basis of coordination between the tax

authorities of the State Government and the authorities of Central Excise and Income-tax to compare

constantly the tax returns and set-off documents of VAT system of the States and those of Central Excise

and Income-tax. This comprehensive cross-checking system will help reduce tax evasion and also lead to

significant growth of tax revenue. At the same time, by protecting the interests of tax-complying dealers

against the unfair practices of tax-evaders, the system will also bring in more equal competition in the sphere

of trade and industry.

Question 7 (4 Marks)

X Co., furnishes you the following information:

Raw material purchased `5,00,000 plus VAT @ 4%.

Manufacturing expenses (revenue nature) `2,00,000.

Sale price `8,00,000 plus VAT @ 4%

Plant & machinery acquired `2,50,000 plus VAT @ 4%.

Compute VAT liability under (i) Gross Product Variant.

(ii) Consumption Variant.

Page 113: sipoysatish@gmail · PDF file · 2013-08-07Explain the concept of Value Added Tax / Explain Taxonomy (process ... Explain accounting treatment of VAT as suggested by ICAI. ... Q26

Value Added Tax 113

State which variant is beneficial to the dealer?

Answer: ` (i) Gross Product Variant

Raw material purchased 5,00,000

Add: VAT @ 4% 20,000

Sale price 8,00,000

Add: VAT @ 4% 32,000

Plant and machinery

Purchase price 2,50,000

Add: VAT @ 4% 10,000

Net tax payable

Output tax 32,000

Less: Tax credit on raw material 20,000

Net tax payable 12,000

(ii) Consumption Variant

Net tax payable

Output tax 32,000

Less: Tax credit on raw material 20,000

Less: Tax credit on plant and machinery 10,000

Net tax payable 2,000

Consumption Variant is beneficial to the dealer

PCC NOV – 2011 Question 1 (5 Marks)

The following particulars are provided by Mr. Prohit of Calcutta, who has purchased Raw materials for

manufacturing PVC Cans and PVC Pipes from Mr. Arvind. The State VAT for Raw Materials and other

materials was 12.5%.

` 1. Cost of Raw materials purchased 1,00,000

2. VAT paid to Mr. Arvind 12,500

3. Cost of other materials

- Local Purchases 20,000

- Interstate Purchases 40,000

4. VAT paid on Local Materials Purchased-12.5% 2,500

5. CST Paid @ 2% 800

6. Manufacturing Expenses 39,200

7. Profit Margin (on Sale Value) 20%

Page 114: sipoysatish@gmail · PDF file · 2013-08-07Explain the concept of Value Added Tax / Explain Taxonomy (process ... Explain accounting treatment of VAT as suggested by ICAI. ... Q26

Value Added Tax 114

Mr. Prohit utilized and manufactured 75% of production as PVC Cans and 25% of production as PVC Pipes.

While PVC Cans are subject to 12.5% VAT, PVC Pipes are exempt. All materials were used in production

and there was no closing stock of Raw materials and other materials.

What would be the invoice value of Sales charged by Mr. Prohit if all the manufactured goods were sold

within the State? What would be his liability under VAT?

Answer:

Computation of Invoice Value of Sale and VAT

Taxable Exempt

75% 25%

Cost of Raw Material Purchased 75,000.00 25,000.00

VAT @ 12.5% --- 3,125.00

Other material – Local Purchases 15,000.00 5,000.00

VAT @ 12.5% --- 625.00

Other Material - Interstate Purchases 30,600.00 10,200.00

Manufacturing expenses 29,400.00 9,800.00

Cost of Product 1,50,000.00 53,750.00

Selling Price

(1,50,000 x 100% / 80%) / (53,750 x 100% / 80%) 1,87,500.00 67,187.50

VAT @ 12.5% 23,437.50 Nil

Invoice value of sale 2,10,937.50 67,187.50

Computation of VAT Payable

Output VAT 23,437.50

Less: Input VAT credit

Raw Material (12,500 x 75%) 9,375.00

Other raw Material (2,500 x 75%) 1,875.00

Net VAT Payable 12,187.50

Rounded off 12,188.00

Question 5 (4 Marks)

Briefly explain the consumption variant of VAT and reasons for its preference over other variants.

Answer. The three variants of VAT are:

(a) Gross product variant

(b) Income variant

(c) Consumption variant.

Consumption variant.

Consumption variant of VAT allows for deduction on all business purchases including capital assets. Thus,

gross investment is deductible in calculating value added. It neither distinguishes between capital and

current expenditures nor specifies the life of assets or depreciation allowances for different assets.

Among the three variants, the consumption variant is most widely used.

The reasons are-

1. It does not affect decisions regarding investment because the tax on capital goods is also set off against

the VAT liability. Hence, the system is tax neutral in respect of techniques of production.

Page 115: sipoysatish@gmail · PDF file · 2013-08-07Explain the concept of Value Added Tax / Explain Taxonomy (process ... Explain accounting treatment of VAT as suggested by ICAI. ... Q26

Value Added Tax 115

2. The consumption variant is convenient from the point of administrative expediency as it simplifies tax

administration by obviating the need to distinguish between capital goods on one hand and consumption

goods on the other hand.

IPCC MAY – 2011 Question 1 (5 Marks)

The following are details of purchases, sales, etc. effected by Vasudha & Co., a registered dealer, for the

year ended 31.03.2013:

Particulars Amount

(`) Purchase of raw materials within State, 1000 units, inclusive of VAT levy at 6% 5,30,000

Inter-State purchase of raw materials, inclusive of CST at 2% 2,04,000

Import of raw materials, inclusive of basic customs duty plus education cess of `36,050 4,35,000

Capital goods purchased on 01.05.2012, inclusive of VAT levy at 10% 3,30,000

(input credit to be spread over 2 financial years)

Other manufacturing expenses 1,50,000

Sale of taxable goods within State, inclusive of VAT levy at 4% 7,28,000

Sale of goods within State, exempt from levy of VAT 1,20,000

(Goods were manufactured from the Inter-State purchase of raw materials)

Closing stock as on 31.03.2013 was 100 units of raw materials purchased within the State

Input credit is allowed only on raw material used in manufacture of the taxable goods. Compute the VAT

liability of the dealer for the year ended 31.03.2013. (Modified)

Answer.

Computation of VAT liability of Vasudha & Co. for the year ended 31.03.2013:-

Particulars Amount

(`) Input tax credit:

Intra-State purchases of 1000 units of raw materials

106

6000,30,5 30,000

Inter-State purchases of raw materials --

Import of raw materials --

Purchase of Capital Goods

2110

10000,30,3 15,000

Other manufacturing expenses --

Total input tax credit available : 45,000

Output VAT payable:

Sale of taxable goods within State [(7,28,000 x 4)/104] 28,000

Sale of exempted goods within State [Refer Note 2] --

VAT credit to be carried forward (28,000 – 45,000) (17000)

Page 116: sipoysatish@gmail · PDF file · 2013-08-07Explain the concept of Value Added Tax / Explain Taxonomy (process ... Explain accounting treatment of VAT as suggested by ICAI. ... Q26

Value Added Tax 116

Notes:- 1. VAT paid on purchase of capital goods is eligible for input tax credit. However, the same has to be

spread over a period of two years.

2. VAT system allows credit in respect of purchases made during a period to be set-off against the

taxable sales during that period, irrespective of when the supplies/inputs purchased are utilized/sold.

Therefore, input tax credit in respect of closing stock of raw materials need not be reduced from total

input tax credit available.

Note: The statement in the question, “Input credit is allowed only on raw materials used in manufacture of

the taxable goods”, implies that the same is not allowable in respect of sale of goods within the State which

are exempt from levy of VAT.

Question 2 (4 Marks)

Which variant of VAT is most widely used in the world and why? Are some services also included in the

VAT net by such countries?

Answer.

Among the three variants of VAT, the consumption variant is widely used. Several countries of Europe

and other countries have adopted this variant. The reasons for preference of this variant are:

Firstly, it does not affect decisions regarding investment because the tax on capital goods is also set-off

against the VAT liability. Hence, the system is tax neutral in respect of techniques of production.

Secondly, the consumption variant is convenient from the point of administrative expediency as it simplifies

tax administration by obviating (to remove problem) the need to distinguish between capital goods on the

one hand and consumption goods on the other hand.

In practice, therefore, most countries use the consumption variant. Also, most VAT countries include many

services in the tax base. Since the business gets set-off for the tax on services, it does not cause any

cascading effect.

Question 3 (4 Marks)

M/s. Staruss & Co., a registered dealer under the local VAT law, having stock of goods purchased from

outside the State, wishes to opt for the Composition Scheme. Advise him whether the same is possible. Will

the VAT chain be broken if the dealer opts for the said scheme?

Answer:

As per the principles laid down in the White Paper, a dealer desirous of availing the benefits of VAT

Composition Scheme should not have stock of the goods purchased from outside the State. Therefore, if the

dealer wishes to avail the benefit of the scheme, he must ensure that he does not possess stock of such goods

as on the date of exercise of option. Advice is to be tendered on above lines.

The selling dealer will not be able to pass on the benefit of the input credit when he opts for the Composition

Scheme. A purchasing dealer buying goods from a dealer operating under the Composition Scheme will not

get any tax credit for the goods purchased. Hence, as soon as any dealer opts for the Composition Scheme,

the VAT chain is broken.

Question 4 (4 Marks)

What is meant by input tax credit in the context of VAT provisions? How does input tax credit help in

achieving the essence of VAT?

Page 117: sipoysatish@gmail · PDF file · 2013-08-07Explain the concept of Value Added Tax / Explain Taxonomy (process ... Explain accounting treatment of VAT as suggested by ICAI. ... Q26

Value Added Tax 117

Answer.

The tax paid by a registered dealer at the earlier point is called input tax. This amount is adjusted/rebated

against the tax payable by the purchasing dealer on his sales. This credit availability is called input tax credit

(ITC). It can also be referred to as tax credit on a sale within the State or in the course of inter-State trade or

commerce.

The essence of VAT is in providing set-off for the tax paid earlier, and this is given effect through the

concept of input tax credit/rebate. Thus, input tax credit in relation to any period can be set off by the

registered dealer against the amount of his output tax.

Question 5 (4 Marks)

What are the major deficiencies of VAT system in India?

Answer:

The major deficiencies of VAT system in India are as under:

(1) There is lack of uniformity in the rates of VAT in different States. Distortion occurs on account of

different rates of VAT, Composition scheme, Exemptions, difference in classification of goods, etc.

(2) Central Sales Tax is not integrated with the State VAT. Therefore, it is difficult to put the purchases

from other States at par with the purchases within the State. Consequently, the advantage of neutrality is

confined only for purchases within the State.

(3) For complying with the VAT provisions, the accounting cost has increased which may not be

commensurate with the benefits to traders and small firms.

(4) VAT is paid at various stages and not at last stage. This has increased the requirement of working

capital and the interest burden on the same.

(5) VAT, being a consumption tax, tends to be regressive since the proportion of income spent on

consumption is large for the poor than the rich.

(6) As a result of introduction of VAT, the administrative cost to the States has increased.

Question 6 (4 Marks)

How can a Chartered Accountant help a client in the handling of VAT audit called for by the Department

and in conducting external audit of VAT records?

Answer:

Handling audit by Departmental Auditors

There are audit wings in VAT Departments and certain percentage of dealers are taken up for audit every

year on scientific basis. Chartered Accountants can ensure proper record keeping to satisfy the Departmental

auditors. The professional expertise of a Chartered Accountant will help him in effectively replying audit

queries and sorting out audit objections.

External audit of VAT records

Under VAT system, trust has been reposed on tax payers as there will be no regular assessment of all VAT

returns, but only few returns will be scrutinized. In other cases, returns filed by dealers will be accepted.

Thus, a check on compliance becomes necessary. Chartered Accountants can play a very vital role in

ensuring tax compliance by audit of VAT accounts.

Question 7 (4 Marks)

Page 118: sipoysatish@gmail · PDF file · 2013-08-07Explain the concept of Value Added Tax / Explain Taxonomy (process ... Explain accounting treatment of VAT as suggested by ICAI. ... Q26

Value Added Tax 118

Briefly list out the contents of VAT invoice.

Answer:

VAT legislations of all the States provide for the tax invoice. Generally, the various legislations provide that

the tax invoice should have the following contents:

(i) the words tax invoice in a prominent place;

(ii) name and address of the selling dealer;

(iii) registration number of the selling dealer;

(iv) name and address of the purchasing dealer;

(v) registration number of the purchasing dealer (may not be required under all VAT legislations);

(vi) pre-printed or self-generated serial numbers;

(vii) date of issue;

(viii) description, quantity and value of goods sold;

(ix) rate and amount of tax charged in respect of taxable goods;

(x) signature of the selling dealer or his regular employee duly authorized by him for such purposes.

PCC MAY – 2011 Question 4 (4 Marks)

A Manufacturer (Registered Dealer) sold goods to Distributor (Registered Dealer) for `20,000. The

Distributor sold the goods to the Wholesaler (Registered Dealer) for `24,000. The Wholesaler sold the goods

to the Retailer (Registered Dealer) for `30,000. The Retailer sold the goods to the final consumer for

`40,000.

The VAT rate is 12.5% which is charged separately.

Compute VAT liability under Invoice method. State why this method is preferable?

Answer.

Computation of VAT liability under invoice method

Particulars Output Tax

` Tax Credit

` Net Tax

` Manufacturer sells the goods to distributor.

` 20,000 x 12.5%

2,500

--

2,500

Distributor sells the goods to wholesaler.

` 24,000 x 12.5%

3,000

2,500

500

Wholesaler sells the goods to a retailer.

` 30,000 x 12.5%

3,750

3,000

7,50

Retailer sold the goods to consumer.

` 40,000 x 12.5%

5,000

3,750

1,250

Total 14,250 9,250 5,000

Page 119: sipoysatish@gmail · PDF file · 2013-08-07Explain the concept of Value Added Tax / Explain Taxonomy (process ... Explain accounting treatment of VAT as suggested by ICAI. ... Q26

Value Added Tax 119

This method is preferable as the tax is charged at each stage of sales on the entire sales value and the tax

paid at the earlier stage is allowed as set off. This method ensures payment of tax at the earlier stage.

Question 7 (4 Marks)

Briefly state the contents of VAT invoice (any 8 items).

Answer:

Refer to Answer given in IPCC MAY – 2011Question No.7

IPCC NOV – 2010 Question 1 (5 Marks)

Mr. Rajesh is a registered dealer and gives the following information. You are required to compute the net

tax liability and total sales value under Value Added Tax:

Rajesh sells his products to dealers in his State and in other States.

The profit margin is 15% of cost of production and VAT rate is 12.5% of sales.

Intra State purchases of raw material `2,50,000/- (excluding VAT @ 4%)

(i) Purchases of raw material from an unregistered dealer `80,000

(ii) High seas (import) purchases of raw material are `1,85,000/- (excluding basic custom duty @ 10%

and education cess and secondary and higher education cess @ 3%)

(iii) Purchases of raw materials from other States (excluding CST @ 2%) `50,000

(iv) Transportation charges, wages and other manufacturing expenses excluding tax `1,45,000

(v) Interest paid on bank loan `70,000/-. (Modified)

Answer:

Computation of net VAT liability and total sales value

`

Intra-State purchases of raw material (excluding VAT ` 10,000) 2,50,000

Purchases of raw materials from unregistered dealer 80,000

High seas purchases of raw materials [Refer Note (i)] 2,04,055

Purchase of raw materials from other States [Refer Note (ii)] 51,000

Transportation charges, wages and manufacturing expenses 1,45,000

Cost of production 7,30,055

Add: Profit margin 15% 1,09,508

8,39,563

Add: VAT @ 12.5% 1,04,945

Total sales value 9,44,508

Computation of VAT liability:- ` VAT on above sales price @ 12.5% 1,04,945

Less: Set off of VAT on purchases:

From high seas Nil

From intra-State [Refer Note (iii)] 10,000

From inter-State Nil

From unregistered dealer Nil 10,000

Net VAT payable 94,945

Notes:

Page 120: sipoysatish@gmail · PDF file · 2013-08-07Explain the concept of Value Added Tax / Explain Taxonomy (process ... Explain accounting treatment of VAT as suggested by ICAI. ... Q26

Value Added Tax 120

(i) Duty paid on high seas purchases i.e., imports is not a State VAT, so the input tax credit is not

available in respect of the same and it is a part of cost of production.

(ii) Set-off of tax paid on inter-state purchases is not allowed.

(iii) Tax on intra-State purchases is `10,000. As credit of the same will be available, it is not included in the

cost of production.

(iv) Interest on loan has been excluded for calculating the cost of production on the presumption that the

loan is availed for purposes other than working capital.

(v) It has been assumed that the entire production is sold.

Question 2 (4 Marks)

What record should be maintained under VAT system by a registered dealer?

Answer:

The following records should be maintained under VAT system:

(i) Purchase records;

(ii) Sales records;

(iii) VAT account;

(iv) Separate record of any exempt sale.

Further, the following records should also be kept and produced to an Officer:

(i) Copies of invoices issued, in serial number;

(ii) Copies of all credit & debit notes issued, in chronological order;

(iii) All purchase invoices, copies of customs entries, receipt for payment of customs duty or tax and

credit and debit notes received to be filed chronologically either by date of receipt or under each

supplier’s name;

(iv) Details of amount of tax charged on each sale or purchase;

(v) Total of the output tax and the input tax in each period and a net total of the tax payable or the excess

carried forward, as the case may be, at the end of each month;

(vi) Details of goods manufactured and delivered from the factory of the taxable person;

(vii) Details of each supply of goods from the business premises, unless such details are available at the

time of supply in invoices issued at, or before, that time.

Question 3 (4 Marks)

State the Variants of VAT. Present them in schematic diagram and explain each one briefly.

Answer:

VAT has following three variants:

Page 121: sipoysatish@gmail · PDF file · 2013-08-07Explain the concept of Value Added Tax / Explain Taxonomy (process ... Explain accounting treatment of VAT as suggested by ICAI. ... Q26

Value Added Tax 121

(a) Gross product variant

(b) Income variant

(c) Consumption variant

These variants are presented in a schematic diagram given below:

Different variants of VAT

Gross product variant Income variant Consumption variant

(a) Gross product variant

The gross product variant allows deductions for taxes on all purchases of raw materials and components, but

no deduction is allowed for taxes on capital inputs. That is, taxes on capital goods such as plant and

machinery are not deductible from the tax base in the year of purchase and tax on the depreciated part of the

plant and machinery is not deductible in the subsequent years.

(b) Income variant

The income variant of VAT on the other hand allows for deductions on purchases of raw materials and

components as well as depreciation on capital goods. This method provides incentives to classify purchases

as current expenditure to claim set-off. In practice, however, there are many difficulties connected with the

specification of any method of measuring depreciation, which basically depends on the life of an asset as

well as on the rate of inflation.

(c) Consumption variant

Consumption variant of VAT allows for deduction on all business purchases including capital assets. Thus,

gross investment is deductible in calculating value added. It neither distinguishes between capital and

current expenditures nor specifies the life of assets or depreciation allowances for different assets.

Question 4 (4 marks)

State with reasons in brief whether the following statements are correct or incorrect with reference to the

provision of Value Added Tax.

(i) It is permitted to issue ‘tax invoice’ inclusive of VAT i.e. aggregate sales price & VAT.

(ii) A registered dealer is compulsorily required to get its books of accounts audited under VAT Laws of

different states irrespective of limit of turnover.

Answer:

(i) Incorrect

Reason: One of the requirements under the contents of the tax invoice is that rate and amount of tax charged

in respect of taxable goods should be distinctly shown in the ‘tax invoice’, in order to claim input credit.

(ii)Incorrect

Reason: Different States have determined different turnover limits above which a registered dealer will have

to get its books of accounts audited under VAT laws.

Question 5 (4 Marks)

Tax is levied on all sales

and deduction for tax

paid on inputs excluding

capital inputs is allowed.

Tax is levied on all sales

with set-off for tax paid on

inputs and only depreciation

on capital goods.

Tax is levied on all sales

with deduction for tax paid

on all business inputs

(including capital goods).

Page 122: sipoysatish@gmail · PDF file · 2013-08-07Explain the concept of Value Added Tax / Explain Taxonomy (process ... Explain accounting treatment of VAT as suggested by ICAI. ... Q26

Value Added Tax 122

What are the conditions to be fulfilled by the dealer accepting the composition scheme under the Value

Added Tax?

Answer: The dealer accepting the composition scheme should fulfill the following conditions:

(i) He should intimate to the Commissioner of VAT in writing that he is opting to such scheme for a year

or a part of the year in which he gets himself registered.

(ii) If he avails this scheme, he is not required to maintain any statutory records as prescribed under the

Act. Only the records for purchases, sales and inventory should be maintained.

(iii) The dealer should not have any stock of goods which were brought from outside the State on the date

he opts to pay tax under composition scheme and should not use such goods after such date.

(iv) The dealer should not claim input tax credit on the inventory available on the date on which he opts for

composition scheme.

Question 6 (4 Marks)

Mention the purchases which are not eligible for input tax credit (any eight items) under Value Added Tax.

Answer:

The following purchases are not eligible for input tax credit:

(i) purchases from unregistered dealer;

(ii) purchases from a registered dealer who opts for composition scheme;

(iii) purchases of goods as may be notified by the State Government;

(iv) purchases of goods where the purchase invoice is not available with the claimant;

(v) purchases of goods where invoice does not show the amount of tax separately;

(vi) purchases of goods which are being utilized in the manufacture of exempted goods;

(vii) purchases of goods used for personal use or provided free of charge as gifts;

(viii) imports from outside the territory of India;

(ix) imports from other States;

(x) goods in stock, which have suffered tax under an earlier Act, but under the VAT Act they are

covered under exempted items. (Note: Any eight points may be given.)

Question 7 (4 Marks)

Compute the VAT amount payable by Mr. Shyam, who purchased goods from a manufacturer on payment

of `4,16,000 (including VAT) and earned 20% profit on purchase price. VAT rate on both purchases and

sales is 4%.

Answer:

Computation of VAT payable by Mr. Shyam ` Payment made to manufacturer 4,16,000

Less: VAT paid [(4,16,000/104) x 4] 16,000

Purchase price 4,00,000

Page 123: sipoysatish@gmail · PDF file · 2013-08-07Explain the concept of Value Added Tax / Explain Taxonomy (process ... Explain accounting treatment of VAT as suggested by ICAI. ... Q26

Value Added Tax 123

Add: Profit margin @ 20% on purchase price 80,000

Sale price before VAT 4,80,000

Add: VAT @ 4% on `4,80,000 19,200

Less: Input credit 16,000

VAT payable by Mr. Shyam 3,200

PCC NOV – 2010 Question 1 (5 Marks) Compute net VAT liability of Rishi from the following information:

Particulars ` ` Raw materials from foreign market (Including Basic Custom duty @ 20%

plus EC)

- 1,23,600

Raw material purchased from local market

Cost of raw material 2,50,000

Add: Excise duty @ 16% 40,000

2,90,000

Add: VAT @ 4%

11,600 3,01,600

Raw material purchased from neighbouring State (Includes CST @ 2%) 51,000

Storage and transportation cost 9,000

Manufacturing expenses 30,000

Rishi sold goods to Madan and earned profit @ 12% on the cost of production. VAT rate on sale of such

goods is 4%. (Modified)

Answer.

Computation of VAT liability of Rishi:-

Particulars ` ` Raw materials purchased from foreign market (including basic custom duty 1,23,600

@ 20% plus EC)

Raw material purchased from local market:-

Cost of raw material 2,50,000

Add: Excise duty @ 16% 40,000 2,90,000

Raw material purchased from neighbouring State (including CST @ 2%) 51,000

Storage and transportation cost 9,000

Manufacturing expenses 30,000

Cost of production 5,03,600

Add: Profit @ 12% of cost of production 60,432

Sale Price 5,64,032

VAT @ 4% on `5,64,032 22,561

Net VAT liability of Rishi:- VAT on sale price 22,561

Less: Input tax credit

Basic custom duty paid on imports Nil

CST paid on inter-state purchases Nil

VAT paid on local purchases 11,600

Net VAT payable by Rishi 10,961

Question 7 (4 Marks)

List out the merits of VAT.

Page 124: sipoysatish@gmail · PDF file · 2013-08-07Explain the concept of Value Added Tax / Explain Taxonomy (process ... Explain accounting treatment of VAT as suggested by ICAI. ... Q26

Value Added Tax 124

Answer.

Merits of VAT

1. No tax evasion is possible as the credit of duty paid is allowed against the liability on the final product

manufactured or sold. Under VAT, unless proper records are kept in respect of various inputs, it is not

possible to claim credit. A perfect system of VAT is a perfect chain where tax evasion is difficult.

2. Neutrality is the greatest advantage of VAT. VAT does not interfere in the choice of decision for

purchases because it has anti-cascading effect. The system is neutral with regard to choice of production

technique, as well as business organisation. All other things remaining the same, the issue of tax liability

does not vary the decision about the source of purchase.

3. It has a certainty as it is based simply on transactions. There is no need to go through complicated

definitions like sales, sales price, turnover of purchases and turnover of sales. The tax is also broad-based

and applicable to all sales in business leaving little room for different interpretations.

4. Transparency is ensured as the buyer knows, out of the total amount paid for purchases of material, what

is the amount paid towards VAT. This transparency enables the State Governments to know as to what is the

exact amount of tax coming at each stage. Thus, it is a great aid to the Government while taking decisions

with regard to rate of tax etc.

5. For Government, better revenue collection and stability is achieved as the tax credit will be given only if

the proof of tax paid at an earlier stage is produced. This means that if the tax is evaded at one stage, full tax

will be recoverable from the person at the subsequent stage or from a person unable to produce proof of such

tax payment.

6. Since the tax paid on an earlier stage is to be received back, the system promotes better accounting

systems.

Note: Any four points may be given.

IPCC MAY – 2010 Question 7 (4 x 2 = 8 Marks)

(a) What are the items aggregated in the Addition method to calculate the VAT payable? When is this

method mainly used?

(b) Is any threshold exemption limit fixed for dealers to obtain VAT registration, as per the White Paper? If

yes, why is the same provided?

(c) Is the VAT chain continued when a purchasing dealer opts for VAT composition scheme? What is the

loss to the seller and buyer opting for the composition scheme, and the subsequent buyers?

(d) Can it be said that VAT brings about certainty to a great extent in the matter of interpretational issues? If

so, how?

Answer 7(a):

(a) In the addition method,

(i) All the factor payments (rent, interest, and wages), and

(ii) Profit, are added to arrive at the value addition on which VAT rate is applied to compute the VAT

payable.

Answer 7(b):

Page 125: sipoysatish@gmail · PDF file · 2013-08-07Explain the concept of Value Added Tax / Explain Taxonomy (process ... Explain accounting treatment of VAT as suggested by ICAI. ... Q26

Value Added Tax 125

The threshold limit for small traders, as per the White Paper is `5 lakh. The same was subsequently

increased to `10 lakh. The same is fixed to provide relief to small traders.

Answer 7(c):

As soon as the dealer opts for the composition scheme, the VAT chain is broken.

When a composition scheme is availed by a seller or buyer, he cannot claim input credit of the tax paid on

the purchases. This will add to the cost of the goods.

The benefit of tax paid earlier will not be passed on to subsequent buyers.

Answer 7(d):

VAT is a system, based simply on transactions; hence there is no need to go through complicated definitions

like sales, turnover, etc.

The tax is also broad based and is applicable to all sales of the commodity in question, leaving little room

for different interpretations.

Hence it can be said that VAT brings certainty to a great extent.

Question 8 (a) (8 Marks)

Mr. X, a dealer in Mumbai dealing in consumer goods, submits the following information pertaining (to

apply in a particular situation) to the Month of March, 2013:

(i) Exempt goods ‘A’ purchased for `2,00,000 and sold for `2,50,000.

(ii) Goods ‘B’ purchased for `2,25,000 (including VAT) and sold at a margin of 10% profit on purchase

(VAT rate 12.5%);

(iii) Goods ‘C’ purchased for `1,00,000 (excluding VAT) and sold for `1,50,000 (VAT rate 4%);

(iv) His unutilized balance in VAT input credit on 01.03.2013 was `1,500.

Compute the turnover, Input VAT, Output VAT and Net VAT payable by Mr. X. (Modified)

Answer 8(a):

Goods Purchases Input VAT credit Sales (Turnover) Output VAT

` ` ` ` A (Exempt) 2,00,000 - 2,50,000 -

B (2,25,000 × 100 / 112.5) 2,00,000 25,000 2,20,000 27,500

C 1,00,000 4,000 1,50,000 6,000

5,00,000 29,000 6,20,000 33,500

` Computation of Net VAT

Output VAT 33,500

Less: Opening balance of Input VAT credit (1,500)

Less: Input VAT credit for March, 2013 (29,000)

Net VAT payable 3,000

Page 126: sipoysatish@gmail · PDF file · 2013-08-07Explain the concept of Value Added Tax / Explain Taxonomy (process ... Explain accounting treatment of VAT as suggested by ICAI. ... Q26

Value Added Tax 126

Computation of purchase price and sale price of goods B

Goods B purchase value (including VAT) 2,25,000

Less: VAT included in above

2,25,000 × 12.5 / 112.5 25,000

Purchase price excluding VAT 2,00,000

Add: Profit on above @ 10% 20,000

Selling price before VAT 2,20,000

VAT @ 12.5% on selling price 27,500

Question 8(b) (3 x 3 = 9 Marks)

(i) What are the merits of VAT in the context of tax evasion, neutrality and transparency?

(ii) State the importance of VAT invoice/tax invoice in administering VAT.

(iii) Discuss the tax consequences of Stock transfer under the VAT scheme.

Answer 8(b)(i):

Refer to Answer given in PCC NOV – 2010 Question No.7

Answer 8(b)(ii):

Invoices are crucial documents for administering VAT. In the absence of invoices VAT paid by the dealer

earlier cannot be claimed as set off.

A VAT invoice:

(i) helps in determining the input tax credit and prevents cascading effect of taxes;

(ii) facilitates multi-point taxation on the value addition;

(iii) promotes assurance of invoices;

(iv) assists in performing audit and investigation activities effectively and checks evasion of tax.

Answer 8(b)(iii):

Inter-State stock transfers do not involve sale and, therefore they are not subjected to sales tax. The same

position is continued under VAT.

However, the tax paid on:

(i) inputs used in the manufacture of finished goods which are stock transferred; or

(ii) purchases of goods which are stock transferred

is available as input tax credit after retention of 2% of such tax by the State Governments.

PCC MAY – 2010 Question 6 (2 Marks)

Do you agree with the statement that tax cannot be evaded under VAT system?

Answer.

The statement that tax cannot be evaded under VAT system is correct. It is said that VAT is a logical beauty.

Under VAT, credit for duty paid is allowed against the liability on the final product manufactured or sold.

Therefore, unless proper records are kept in respect of various inputs, it is not possible to claim credit.

Hence, suppression of purchases or production will be difficult because it will lead to loss of revenue. A

perfect system of VAT is a perfect chain where tax evasion is difficult.

Page 127: sipoysatish@gmail · PDF file · 2013-08-07Explain the concept of Value Added Tax / Explain Taxonomy (process ... Explain accounting treatment of VAT as suggested by ICAI. ... Q26

Value Added Tax 127

Question 8 (3 Marks Each)

(a) Compute the VAT liability of Mr. P Kapoor for the month of October, 2012, using the ‘Invoice method’

of computation of VAT.

Purchases from the local market

(Includes VAT 4%) `65,000

Storage cost incurred ` 750

Transportation cost ` 1,750

Goods sold at a margin of 5% on the cost of such goods

VAT rate on sales 12.5%.

(b) What are the three variants of VAT? Which of these methods is most widely used and why?

Answer 8(a).

Computation of VAT Liability of Mr. P. Kapoor for the month of October 2012 using 'invoice

method' of computation of VAT:

Particulars ` Purchase price (including VAT @ 4%) 65,000

Less: VAT paid on purchases (65,000 × 4 / 104) 2,500

Add: Storage cost 750

Add: Transportation cost 1,750

Cost Price 65,000

Add: Profit @ 5% of cost price 3,250

Sale price before VAT 68,250

VAT @ 12.5% (` 68,250 × 12.5%) 8,531

Less: VAT paid on purchases 2,500

VAT Liability of Mr. P. Kapoor 6,031

Answer 8(b).

Refer to Answer given in PCC NOV – 2011Question No.5

IPCC NOV – 2009 Question 7 (2 x 4 = 8 Marks)

Answer the following:

(a) What are the different rates under VAT system?

(b) Under what circumstances registration can be cancelled under VAT?

(c) Briefly explain the income variant of VAT.

(d) State with reasons in brief whether the following statement is true or false with reference to the provision

of Value Added Tax.

The VAT rate on sale of Lottery Ticket is 4%.

Answer 7(a):

Page 128: sipoysatish@gmail · PDF file · 2013-08-07Explain the concept of Value Added Tax / Explain Taxonomy (process ... Explain accounting treatment of VAT as suggested by ICAI. ... Q26

Value Added Tax 128

To reduce the multiplicity of sales-tax rates between various States in India, it was recommended that VAT

will have broadly the following tax rates:

(a) Zero rate for tax free goods,

(b) 1% on precious or semi-precious metals i.e., bullion etc.

(c) 4% on items of basic necessities, agricultural and industrial inputs, capital goods and declared goods

(d) 20% on non VAT goods

(e) 12.5% on other goods.

Answer 7(b):

VAT registration can be cancelled on:

(i) discontinuance of business; or

(ii) disposal of business; or

(iii) transfer of business to new location; or

(iv) annual turnover falling below the specified limit.

Answer 7(c):

The income variant of VAT allows deduction of purchases of raw material and components as well as

depreciation of capital goods. This method provides incentive to classify purchases as current expenditure to

claim set off. In practice, however, there are many difficulties connected with the specification of any

method of measuring depreciation, which basically depends on the life of an asset as well as on the rate of

inflation.

Answer 7(d):

False. VAT rate for lottery tickets is different in different States e.g. in Delhi it is 20%

Question 8 (8 Marks)

Mr. X (Registered Dealer) is a manufacturer sells goods to Mr. B (Registered Dealer), a distributor for

`2,000 (excluding of VAT). Mr. B sells goods to Mr. K (Registered Dealer), a wholesale dealer for `2,400.

The wholesale dealer sells the goods to a retailer (Registered Dealer) for `3,000, who ultimately sells to the

consumers for `4,000.

Compute the Tax Liability, input credit availed and tax payable by the manufacturer, distributor, wholesale

dealer and retailer under Invoice method assuming VAT rate @ 12.5%.

Answer:

Computation of tax liability, input tax credit availed and tax payable under invoice method

Stage Particulars

Output Tax Less

VAT

Credit

Net Tax

1. X, the manufacturer, sells to B, the distributor, for

`2,000. Therefore his tax liability will be `250 (`2,000

@ 12.5%). He will not have any VAT credit.

250 - 250

2. B, the distributor, sells goods to K, the wholesale

dealer, for `2,400. B’s tax liability will be `300

(`2,400 @ 12.5%). He will get set off of tax paid at

earlier stage of `250. Thus, tax payable by him will

be `50.

300 250 50

Page 129: sipoysatish@gmail · PDF file · 2013-08-07Explain the concept of Value Added Tax / Explain Taxonomy (process ... Explain accounting treatment of VAT as suggested by ICAI. ... Q26

Value Added Tax 129

3. K, the wholesaler dealer, sells to retailer at `3,000.

K’s tax liability will be `375 (`3,000 @ 12.5%). He

will get set off of tax paid at earlier stage of `300.

Thus, tax payable by him will be `75.

375 300 75

4. Retailer sells goods to consumers at `4,000. His tax

liability will be `500 (`4,000 @ 12.5%). He will get

set off of tax paid at earlier stage of `375. Thus, tax

payable by him will be `125/-

500 375 125

Note: It has been assumed that sales made by the distributor, the wholesale dealer and the retailer are also

exclusive of VAT.

Question 8 (3 x 3 = 9 Marks)

(i) What are the different stages of VAT? Can it be said that entire burden falls on the final consumer?

(ii) Discuss filing of Return under VAT.

(iii) List six purchases which are not eligible for input tax credit.

Answer 8(i):

The Value Added Tax (VAT) is a multistage tax levied (to use official authority to demand and collect a

payment, tax) as a proportion of the value added (i.e. sale minus purchase) which is equivalent to wages plus

interest, other costs and profits. In an economy, apart from the manufacturers and final consumers, there

would be wholesalers and retailers also. VAT is collected at each stage of production and distribution

process, and in principle, its entire burden falls on the final consumer, who does not get any tax credit. Thus,

VAT is a broad based tax covering the value added to each commodity by parties during the various stages

of production and distribution.

Answer 8(ii):

VAT returns are to be filed monthly/quarterly/half-yearly/annually along with tax paid challans according to

the provisions of the State Acts. They should contain details of output tax liability, value of input tax credit

and payment of VAT and should be filed within the prescribed time schedule. In case of any mistakes,

revised returns may be filed.

The returns will be checked and any deficiency in payment of tax may have to be made good.

Filing of returns are designed with a view:

(i) to reduce cost of compliance

(ii) to encourage businesses to comply with their obligations; and

(iii) to ensure efficient processing of data.

Answer 8(iii):

Refer to Answer given in IPCC NOV – 2010 Question No.6

Page 130: sipoysatish@gmail · PDF file · 2013-08-07Explain the concept of Value Added Tax / Explain Taxonomy (process ... Explain accounting treatment of VAT as suggested by ICAI. ... Q26

Value Added Tax 130

PCC NOV – 2009 Question 7 (5 Marks)

Mr. Goenka, a Registered Dealer, is selling raw materials to a manufacturer of finished products. He imports

(purchases from other states is also called import) his stock in trade as well as purchases the same from the

local markets.

Following transactions took place during financial year 2012-13.

Calculate the VAT and invoice value charged by him to a manufacturer. Assume the rate of VAT @ 12.5%:

` (1) Cost of imported materials (from other State) excluding tax 1,00,000

(2) Cost of local materials including VAT 2,25,000

(3) Other expenditure includes storage, transport, interest and loading and unloading and

profit earned by him

87,500

(Modified)

Answer.

Sales Price of goods:-

Particulars ` Imported material cost 1,00,000.00

Add: CST @ 2% 2,000.00

Add: Cost of local materials 2,25,000

Less: VAT @12.5% (2,25,000 x 12.5 / 112.5) 25,000 2,00,000.00

[Since, credit of ` 25,000 would be available, it will not be included in cost of input]

Add: Other expenses and profit 87,500.00

Sales Price of goods 3,89,500.00

Add: Output VAT @12.5% 48,688.00

Invoice value charged by Mr. Goenka to a manufacturer 4,38,188.00

VAT Payable by Mr. Goenka (48,688 – 25,000) 23,688.00

Question 8 (3 Marks)

VAT would increase the working capital requirements and the interest burden. Discuss.

Answer.

One of the demerits of VAT is that it increases the working capital requirements and the interest burden. The

tax is imposed or paid at various stages and not on last stage only.

It increases the requirement of working capital and also the interest element as compared to single stage-last

point taxation system.

PCC JUNE – 2009 Question 6 (2 Marks)

Discuss the word “transparency” in the context of VAT system.

Answer.

Page 131: sipoysatish@gmail · PDF file · 2013-08-07Explain the concept of Value Added Tax / Explain Taxonomy (process ... Explain accounting treatment of VAT as suggested by ICAI. ... Q26

Value Added Tax 131

Out of total consideration paid for purchase of material, the buyer knows the tax component under a VAT

system. Thus, the system ensures transparency. This transparency enables the State Government to know as

to what is the exact amount of tax coming at each stage. Thus, it is a great aid to the Government while

taking decisions with regards to rate of tax etc.

Question 7 (3 Marks)

Compute the VAT amount payable by Mr. A (Registered Dealer) who purchases goods from a Manufacturer

(Registered Dealer) on payment of `2,25,000 (including VAT) and earns 10% profit on purchase.

The goods have been sold to retailers and VAT rate on purchase and sale is 12.5%. (Modified)

Answer.

Computation of VAT payable by Mr. A:-

Amount (`) Payment made to manufacturer 2,25,000

Less: VAT paid (2,25,000 x 12.5)/112.5 25,000

Purchase price 2,00,000

Add: Profit margin (10% of Cost Price) 20,000

Sale price before VAT 2,20,000

Add: VAT @ 12.5% on ` 2,20,000 27,500

Output tax 27,500

Less: Tax credit 25,000

VAT payable by Mr. A 2,500

Question 8 (3 Marks Each )

(a) How can an auditor play role to ensure that the tax payers discharge their tax liability properly under the

VAT system.

(b) Discuss the ‘subtraction method’ for computation of VAT.

Answer 8(a).

Refer to Answer given in IPCC NOV – 2011 Question No.2

Answer 8(b).

Under the subtraction method, the tax is charged only on the value added at each stage of the sale of the

goods. Since, the total value of goods sold is not taken into account, the question of grant of claim for set-off

or tax credit does not arise.

This method is normally applied where the tax is not charged separately. Under this method for imposing

tax, ‘value added’ is simply taken as the difference between sales and purchases.

PCC NOV – 2008 Question 6 (2 Marks)

Can we say that levy of VAT will have effect on retail price of goods?

Answer.

A persistent criticism of the VAT form has been that since the tax is payable on the final sale price, the VAT

usually increases the prices of the goods. However, VAT does not have any inflationary impact as it merely

replaces the existing equal sales tax. It may also be pointed out that with the introduction of VAT, the tax

impact on raw material is to be totally eliminated. Therefore, there may not be any increase in the prices.

Question 8 (3 Marks Each)

Page 132: sipoysatish@gmail · PDF file · 2013-08-07Explain the concept of Value Added Tax / Explain Taxonomy (process ... Explain accounting treatment of VAT as suggested by ICAI. ... Q26

Value Added Tax 132

(a) Explain “Input Tax Credit” in context of VAT.

(b) What are the exceptions to input tax credit?

Answer 8(a).

Refer to Answer given in IPCC MAY – 2011 Question No.4

Answer 8(b).

Input tax credit may not be allowed in the following circumstances:

(i) purchases from unregistered dealers;

(ii) purchases from registered dealer who opt for composition scheme under the provisions of the Act;

(iii) purchase of goods as may be notified by the State Government;

(iv) purchase of goods where the purchase invoice is not available with the claimant or there is evidence that

the same has not been issued by the selling registered dealer from whom the goods are purported (claimed)

to have been purchased;

(v) purchase of goods where invoice does not show the amount of tax separately;

(vi) purchase of goods, which are being utilized in the manufacture of, exempted goods;

(vii) goods in stock, which have suffered tax under an earlier Act but under VAT Act they are covered under

exempted items;

(viii) purchase of goods used for personal use/consumption or provided free of charge as gifts;

(ix) goods imported from outside the territory of India (commonly known as high seas purchases);

(x) goods imported from other States viz. (namely) inter-State purchases

.

Note: Any three points can be given in the above answer.

PCC MAY – 2008 Question 6 (2 x 2 = 4 Marks)

(a) Briefly explain the income variant of VAT.

(b) What is the demerit of VAT from the view point that it is a form of consumption tax?

Answer 6(a):

Refer to Answer given in IPCC NOV – 2009 Question No.7

Answer 6(b).

Demerit of VAT

VAT is a form of consumption tax. Since the proportion of income spent on consumption is larger for the

poor than for the rich, VAT tends to be regressive.

However, this weakness is inherent in all the forms of consumption tax. While it may be possible to

moderate the distribution impact of VAT by taxing necessities at a lower rate, it is always advisable to

moderate the distribution considerations through other programmes rather than concessions or exemptions,

which create complications for administration.

Page 133: sipoysatish@gmail · PDF file · 2013-08-07Explain the concept of Value Added Tax / Explain Taxonomy (process ... Explain accounting treatment of VAT as suggested by ICAI. ... Q26

Value Added Tax 133

Question 8 (3 Marks)

(a) What are the different stages of VAT? Can it be said that the entire burden falls on the final consumer?

(b) Briefly explain, how VAT helps in checking tax evasion and in achieving neutrality.

Answer 8(a).

Different Stages of VAT

The Value Added Tax (VAT) is a multistage tax levied as a proportion of the value added (i.e. Sale minus

purchase) which is equivalent to wages plus interest, other costs and profits.

In an economy, apart from the manufacturers and final consumers, there would be wholesalers and retailers

also. The wholesaler might supply to retailer, and each one of them could supply to the manufacturer and the

end consumer. VAT will be collected at each stage, and wherever applicable, the manufacturer or retailer

will claim input credit.

Thus, VAT is collected at each stage of production and distribution process, and in principle, its entire

burden falls on the final consumer, who does not get any tax credit. Thus VAT is a broad-based (based on a

wide variety of things) tax covering the value added to each commodity by parties during the various stages

of production and distribution.

Answer 8(b).

No Tax Evasion

It is said that VAT is a logical beauty. Under VAT, credit of duty paid is allowed against the liability on the

final product manufactured or sold. Therefore, unless proper records are kept in respect of various inputs, it

is not possible to claim credit. Hence, suppression of purchases or production will be difficult because it will

lead to loss of revenue. A perfect system of VAT will be a perfect chain where tax evasion is difficult.

Neutrality

The greatest advantage of the system is that it does not interfere in the choice of decision for purchases. This

is because the system has anti-cascading effect. How much value is added and at what stage it is added in

the system of production/distribution is of no consequence. The system is neutral with regard to choice of

production technique, as well as business organisation. All other things remaining the same, the issue of tax

liability does not vary the decision about the source of purchase. VAT facilitates precise identification and

rebate of the tax on purchases and thus ensures that there is no cascading effect of tax. In short, the

allocation of resources is left to be decided by the free play of market forces and competition.

PCC NOV – 2007 Question 6 (2 x 2 = 4 Marks)

(a) Does the VAT system bring certainty to a great extent?

(b) Can VAT be said to be non-beneficial as compared to single stage-last point system?

Answer 6(a).

The VAT is a system based simply on transactions. Thus there is no need to go through complicated

definitions like sales, sales price, turnover of purchases and turnover of sales. The tax is also broad-based

and applicable to all sales in business leaving little room for different interpretations. Thus, this system

brings certainty to a great extent.

Answer 6(b).

Page 134: sipoysatish@gmail · PDF file · 2013-08-07Explain the concept of Value Added Tax / Explain Taxonomy (process ... Explain accounting treatment of VAT as suggested by ICAI. ... Q26

Value Added Tax 134

VAT system has many advantages like no tax evasion, transparency, certainty, reduction in cascading effect

of taxes etc. However, since the VAT is imposed or paid at various stages and not at last stage, it increases

the working capital requirements and the interest burden on the same. In this way, it may be considered to be

non-beneficial as compared to the single stage-last point taxation system though to a certain extent, this

rigour (problem) can be brought down through input credits on purchases.

Question 7 (3 Marks)

Compute the invoice value to be charged and amount of tax payable under VAT by a Registered Dealer who

had purchased goods for ` 1,20,000 and after adding for expenses of ` 10,000 and profit of ` 15,000 had sold

out the same.

The rate of VAT on purchases and sales is 12.5%. (Modified)

Answer.

Computation of Invoice Value

Particulars ` ` Cost of goods purchased 1,20,000

Add: Expenses 10,000

Add: Profit margin 15,000 25,000

Product Sale Value 1,45,000

Add: VAT @ 12.5% 18,125

Invoice Value 1,63,125

Computation of VAT payable

VAT charged on sales 18,125

Less: Input credit of VAT paid on purchases @ 12.5% on 1,20,000 15,000

VAT Payable 3,125

Note: It has been assumed that the purchase price of ` 1,20,000 is exclusive of VAT.

Question 8 (3 Marks)

What are the different variants of VAT and how is deduction available for tax paid on inputs including

capital inputs?

Answer. Refer to Answer given in IPCC NOV – 2010 Question No.3

PCC MAY – 2007 Question 6 (2 x 2 = 4 Marks)

(a) Which is the most popular and common method for computing VAT liability and at what stage is the tax

imposed?

(b) Is it correct to state that VAT usually increases the retail price, as the tax is payable on the first sale

price?

Answer 6(a).

Invoice method is the most common and popular method for computing the tax liability under the VAT

system. Under this method, tax is imposed at each and every stage of sales on the entire sale value, and the

tax paid at the earlier stage is allowed as set-off.

Answer 6(b).

Page 135: sipoysatish@gmail · PDF file · 2013-08-07Explain the concept of Value Added Tax / Explain Taxonomy (process ... Explain accounting treatment of VAT as suggested by ICAI. ... Q26

Value Added Tax 135

The statement is not correct as VAT is a multi-point tax where tax is imposed at each and every stage of

sales and tax paid at the earlier stage is allowed as set-off.

Question 8 (3 Marks)

Briefly explain the invoice method of computing tax liability under the VAT system. What are its other

names?

Answer.

Invoice method is the most common and popular method for computing the tax liability under ‘VAT’

system. Under this method, tax is imposed at each stage of sales on the entire sale value and the tax paid at

the earlier stage is allowed as set-off. In other words, out of tax so calculated, tax paid at the earlier stage

i.e., at the stage of purchases is set-off, and at every stage the differential tax is being paid. The most

important aspect of this method is that at each stage, tax is to be charged separately in the invoice. This

method is very popular in western countries. In India also, under the VAT law as introduced in several Sates

and Central Excise Law, this method is followed.

This method is also called the ‘Tax Credit Method’ or ‘Voucher Method’.

Page 136: sipoysatish@gmail · PDF file · 2013-08-07Explain the concept of Value Added Tax / Explain Taxonomy (process ... Explain accounting treatment of VAT as suggested by ICAI. ... Q26

Value Added Tax 136

FORM DVAT 04 – COVER PAGE

(See Rule 12 of the Delhi Value Added Tax Rules, 2005)

Application for Registration under Delhi Value Added Tax Act, 2004

Checklist of Supporting Documents

Please tick as applicable

Mandatory Supporting Documents

Annexures of the Form duly filled in (in case any of the annexures is not applicable, please mention

the same )

Proof of incorporation of the applicant dealer i.e. Copy of deed of constitution (partnership deed (if

any), certificate of registration under the Societies Act, Trust deed, Memorandum and Articles of

Association etc) duly certified by the authorized signatory

Proof of identity of authorized signatory signing the Registration Application Form

Two self addressed envelopes (Without stamps)

In case of a dealer applying for registration and simultaneously opting for payment of tax under

composition scheme, please attach application in Form DVAT 01 along with this application

Proof of Security along with duly filled form DVAT-12

Optional Supporting Documents (For reduction in Security Amount)

Proof of ownership of principle place of business

Proof of ownership of residential property by proprietor/ managing partner

Copy of passport of proprietor/ managing partner

Copy of Permanent Account Number in the name of the business allotted by the Income Tax

Department

Copy of last electricity bill (The bill should be in the name of the business and for the address

specified as the main place of business in the registration form)

Copy of last telephone bill (The bill should be in the name of the business and for the address

specified as the main place of business in the registration form)

Reasons for Rejection (For Office Use Only)

Please tick as applicable

Not attached Mandatory Supporting Document(s) _________________________

Page 137: sipoysatish@gmail · PDF file · 2013-08-07Explain the concept of Value Added Tax / Explain Taxonomy (process ... Explain accounting treatment of VAT as suggested by ICAI. ... Q26

Value Added Tax 137

Other____________________________________________________________

FORM DVAT 04 (See Rule 12 the Delhi Value added Tax Rules, 2005)

Application for Registration under Delhi Value Added Tax Act, 2004

1. Full Name of Applicant Dealer

(For individual, provide in order of first

name, middle name, surname)

2. Trade Name (if any)

3. Nature of Business

(Tick [√] all applicable)

Manufactur

e

Trade

r

leasin

g

Work contractor Other(specify)

4. Constitution of

Business

(Tick [√] all applicable)

Proprietorship Private Ltd.

Company

Public Sector Undertaking

Partnership Government

Company

Government Corporation

HUF Public Ltd.

Company

Govt. Deptt/Society

/Club/Trust

Others, please

Specify

5. Type of Registration Tike [√ ] one Mediatory Voluntary

5A. Opting for composition scheme under section 16 of the Act? Tick [√] one Yes

No

6. Annual Turnover Category Tick** one less than ` 10 lacs ` 10 lacs or above

(a) Turnover in preceding financial year `

(b) Expected turnover in the current Financial year `

7 Date from which liable for registration under Delhi Value

Added Tax Act,2004

/

/

Day Month Year

8. Permanent Account Number of the applicant dealer (PAN)

9. Registration number under Central Excise Act (if applicable)

10. Principal Place of Business Building Name/

Number

Area/ Road

Locality/Market

Page 138: sipoysatish@gmail · PDF file · 2013-08-07Explain the concept of Value Added Tax / Explain Taxonomy (process ... Explain accounting treatment of VAT as suggested by ICAI. ... Q26

Value Added Tax 138

Pin Code

Email Id

Telephone Number

Fax Number

11. Address for Service of Notice

(if different from principal place of

business)

Building Name/

Number

Area/ Road

Locality/Market

Pin Code

Email Id

Telephone Number

Fax Number

12. Number of additional places of business within or

outside the State

(also please complete part C)

Godown /Warehouse

Factory

Shop

Other place of business

13. Details of main Bank Account Account Number

MICR Number

Name of Bank

Address of Bank

14. Details of investment in the

business

(details should be current as on

date of application)

Own Capital (`)

Loans from Banks (`)

Others loans and borrowing (`)

Plant & Machinery (`)

Land & Building (`)

Others assets & investments (`)

15. Description of top 5 items you deal or propose to deal in Description of items

(1-higest volume to 5-lowest volume) 1

2

3

4

5

16. Accounting Basis Tick [ √ ] One Accrual Cash

Page 139: sipoysatish@gmail · PDF file · 2013-08-07Explain the concept of Value Added Tax / Explain Taxonomy (process ... Explain accounting treatment of VAT as suggested by ICAI. ... Q26

Value Added Tax 139

17. Frequency of filing of returns (to be filled in by the

dealer whose turnover is less than ` 5 crores in the

preceding year) Tick One [ √ ] if applicable

Monthly Quarterly

18. Security (a) Amount of Security `

(b) Type of Security

(c) Date of expiry of Security / /

Day Month Year

19. Number of person having interest in business

(also please complete part B for each such person

20. Number of managers

21. Numbers of authorized Signatories

22. Name of manager

First Name Middle Name Surname

*If more than one manager, attach particulars for additional managers on a separate sheet.

23. Name of authorized

signatory *

First Name Middle Name Surname

*please complete Part D

24. Verification

I/We ________________________________ hereby solemnly affirm and declare that the information

given hereinabove is true and correct to the best of my/our knowledge and belief and nothing has been

concealed there from.

Signature of Authorized Signatory……………………

Name……………….

Designation/Status…………………

Place…………

Date:

Day Month Year

Page 140: sipoysatish@gmail · PDF file · 2013-08-07Explain the concept of Value Added Tax / Explain Taxonomy (process ... Explain accounting treatment of VAT as suggested by ICAI. ... Q26

Value Added Tax 140

FORM DVAT 04 : Annexure I Particulars of person [proprietor/karta/partners/directors in the business/members of executives

committee of societies, clubs etc.] having interest in the business

1. Full Name of Applicant Dealer

(For individual, provide in order of first

name, middle name, surname)

2. Registration No.*

*This field is applicable when applying for amendment of registration in Form DVAT 07

3. Full name of person

(Provide in order of first name, middle name

surname)

4.Date of birth / /

5. Gender (tick [√] one) Male Female

6. Father’s / Husband’s name

First Name Middle Name Surname

7. PAN :

8. Passport No.

9. E-mail address

10. Residential Address

(If different from principle place

of Business)

Building Name/ Number

Area/ Road

Locality/ Market

Pin Code

Telephone Number

Fax Number

11. Permanent Address

(If different from residential

address)

Building Name/ Number

Area/ Road

Locality/ Market

Pin Code

Telephone Number

Please affix a passport size

photograph of the person

whose particulars are being

given in this form

Page 141: sipoysatish@gmail · PDF file · 2013-08-07Explain the concept of Value Added Tax / Explain Taxonomy (process ... Explain accounting treatment of VAT as suggested by ICAI. ... Q26

Value Added Tax 141

Fax Number

12. Verification

I/We __________________________________________ hereby solemnly affirm and declare that the

information given hereinabove is true and correct to the best of my/our knowledge and belief and nothing

has been concealed there from.

Signature of Authorized Signatory ……………………………….

Full Name (first name, middle, surname) …………………………

Designation/Status ……………………………………………

Place: …………………

Date:

Day Month Year

Page 142: sipoysatish@gmail · PDF file · 2013-08-07Explain the concept of Value Added Tax / Explain Taxonomy (process ... Explain accounting treatment of VAT as suggested by ICAI. ... Q26

Value Added Tax 142

FORM DVAT 04: Annexure II Details of additional places of business

1. Full Name of Applicant Dealer

(For individual, provide in order of first name,

middle name, surname)

2. Registration No.*

*This field is applicable when applying for amendment of registration in Form DVAT 07

3. Details of Additional Places of Business (attach additional sheets if required)

Type Godown/Warehouse Factory Shop Other place of business

Address Building Name/ Number

Area/ Road

Locality/ Market

Pin Code

Email Id

Telephone Number

Fax Number

Date of establishment / /

State local sales tax/VAT/CST registration

number

(if place of business is situated outside

Delhi)

Day Month Year

Type Godown/Warehouse Factory Shop Other place of business

Address Building Name/Number

Area/ Road

Locality/ Market

Distt.

State

Pin Code

Email Id

Telephone Number

Fax Number

Date of establishment / /

State local sales tax/VAT/CST registration

number

(if place of business is situated outside

Delhi)

Date Month Year

Type Godown/Warehouse Factory Shop Other place of business

Address Building Name/Number

Area/ Road

Locality/ Market

Distt.

Page 143: sipoysatish@gmail · PDF file · 2013-08-07Explain the concept of Value Added Tax / Explain Taxonomy (process ... Explain accounting treatment of VAT as suggested by ICAI. ... Q26

Value Added Tax 143

State

Pin Code

Email Id

Telephone Number

Fax Number

Date of establishment / /

State local sales tax/VAT/CST registration

number

(if place of business is situated outside

Delhi)

Date Month Year

Type Godown/Warehouse Factory Shop Other place of business

Address Building Name/Number

Area/ Road

Locality/ Market

Distt.

State

Pin Code

Email Id

Telephone Number

Fax Number

Date of establishment / /

State local sales tax/VAT/CST registration

number (if place of business is situated

outside Delhi)

Date Month Year

4. Verification

I/We __________________________________________ hereby solemnly affirm and declare that the

information given hereinabove is true and correct to the best of my/our knowledge and belief and nothing

has been concealed there from.

Signature of Authorized Signatory ………………………………….

Full Name (first name, middle, surname) ………………………….

Designation/Status …………………………….

Place: …………………………..

Date:

Day Month Year

Page 144: sipoysatish@gmail · PDF file · 2013-08-07Explain the concept of Value Added Tax / Explain Taxonomy (process ... Explain accounting treatment of VAT as suggested by ICAI. ... Q26

Value Added Tax 144

FORM DVAT 04: Annexure III Particulars of the authorised signatory

1. Full Name of Applicant Dealer

(For individual, provide in order of first name,

middle name, surname)

2. Registration No.*

*This field is applicable when applying for amendment of registration in Form DVAT 07

3. Name of Authorised Signatory

(Provide in order of first name, middle name,

surname)

4. Date of birth / /

5. Gender (tick [√] one) Male Female

6. Father’s / Husband’s name

First Name Middle Name Surname

7. PAN:

8. Passport No.

9. E-mail address

10. Residential Address

(If different from principle place

of Business)

Building Name/ Number

Area/ Road

Locality/ Market

Distt.

State

Pin Code

Telephone Number

Fax Number

11. Permanent Address

(If different from residential

address)

Building Name/ Number

Area/ Road

Locality/ Market

Distt.

State

Pin Code

Telephone Number

Fax Number

Page 145: sipoysatish@gmail · PDF file · 2013-08-07Explain the concept of Value Added Tax / Explain Taxonomy (process ... Explain accounting treatment of VAT as suggested by ICAI. ... Q26

Value Added Tax 145

12. Declaration

I/We ________________________________________________ hereby solemnly affirm and declare that

the person named above is authorized to act as an authorized signatory for the above referred business for

which application for registration is being filed/ is registered under the Delhi VAT Act, 2004. All his actions

in relation to this business will be binding on us.

S. No. __________________

Full Name (First name, Middle Name, Surname) __________________________

Designation ______________________

Signature _______________________

13. Acceptance as an authorized signatory

I __________________________________________ hereby solemnly accord my acceptance to act as

authorized signatory for the above referred business and all my acts shall be binding on the business.

Signature of Authorised Signatory…………………………

Full Name (First name, middle, surname) …………………..

Designation/Status ……………………………

Place: ……………………………

Date:

Day Month Year

Page 146: sipoysatish@gmail · PDF file · 2013-08-07Explain the concept of Value Added Tax / Explain Taxonomy (process ... Explain accounting treatment of VAT as suggested by ICAI. ... Q26

Value Added Tax 146

Instructions for filling Registration Form (DVAT-04) (For details refer to Section 19 and Rule 12)

1. Please fill in all the details in CAPITAL letters.

2. Please note that you are mandatorily required to register if you:

(i) had turnover of more than Rupees 10 lakhs in the preceding financial year; or

(ii) exceed turnover of Rupees 10 lakhs in the current year; or

(iii) are liable to pay tax, or are registered or required to be registered under Central Sales Tax Act,

1956

3. Please note that irrespective of the quantum of turnover of the business, a dealer may apply for voluntary

registration under the Delhi Value Added Tax Act, 2004.

4. For field 3, an “importer” means -

(i) a person who brings his own goods into Delhi; or

(ii) a person on whose behalf another person brings goods into Delhi; or

(iii) in the case of a sale occurring in the circumstances referred to in sub-section 2 of section 6 of the

Central Sales Tax Act, 1956, the person in Delhi to whom the goods are delivered

5. The application for registration under this Act should be filed within 30 days from the date of person

becoming liable for payment of tax.

6. For field 8, if the business does not have a PAN, then please mark ‘Applied for’ or ‘N/A’ as applicable.

7. For field 15, please fill the description of top 5 items on the basis of value of goods sold.

8. In case any of these details change, the dealer is required to intimate the department of the amendments

within one month of the change.

9. The form has to be filled and signed by the authorised signatory of the business.

10. Businesses with a turnover of more than ` 5 crores are mandatoryily required to file returns every month.

Businesses with a turnover of less than ` 5 crores are required to file returns every quarter. They may

however, elect to file their returns every month.

11. Registration application should be verified and signed by the following:

(i) in the case of an individual, by the individual himself, and where the individual is absent from

India, either by the individual or by some person duly authorised by him in this behalf and where the

individual is mentally incapacitated from attending to his affairs, by his guardian or by any other

person competent to act on his behalf;

(ii) in the case of a Hindu Undivided Family, by a Karta and where the Karta is absent from India or

is mentally incapacitated from attending to his affairs, by any other adult member of such family;

(iii) in the case of a company or local authority, by the principle officer thereof;

(iv) in the case of a firm, by any partner thereof, not being a minor;

(v) in the case of any other association, by any member of the association or persons;

(vi) in the case of a trust, by the trustee or any trustee; and

(vii) in the case of any other person, by some person competent to act on his behalf.

Page 147: sipoysatish@gmail · PDF file · 2013-08-07Explain the concept of Value Added Tax / Explain Taxonomy (process ... Explain accounting treatment of VAT as suggested by ICAI. ... Q26

Value Added Tax 147

Instructions for filling Registration Form (Annexures I, II and III)

1. In case of partnerships, Annexure I to be filled and signed by the managing partner plus top 4 other

partners.

2. In case of companies, Annexure I to be filled and signed by the company secretary, the managing director

and 3 other directors.

3. If required, make additional copies of the Annexures and attach with application form for registration

(DVAT-04).

4. An amendment would be required each time a person changes (and not when the details of an existing

person change)

5. In case of minors, the specimen signature of guardian/ trustee should be furnished.

6. In case of Annexure III, it is to be filled and signed by the person whose details are given in the

Annexure.

7. Every sheet filled in the part has to be signed by the same person (authorized signatory) who has signed

the registration application.

8. In case any of the Parts are not applicable, please strike off the same and write ‘Not Applicable’ on the

said Part.

Method of Calculating Security Amount

Prescribed Security Amount (`) 1,00,000

Reduction sought (Maximum reduction available ` 50,000) Rebate (`)

1. Proof of ownership of principle place of business

30,000

2. Proof of ownership of residential property by proprietor/ managing partner

20,000

3. Copy of passport of proprietor/ managing partner 10,000

4. Copy of Permanent Account Number in the name of the business allotted by the

Income Tax Department

10,000

5. Copy of last electricity bill (The bill should be in the name of the business and for

the address specified as the main place of business in the registration form)

10,000

6. Copy of last telephone bill (The bill should be in the name of the business and for

the address specified as the main place of business in the registration form)

5,000

Page 148: sipoysatish@gmail · PDF file · 2013-08-07Explain the concept of Value Added Tax / Explain Taxonomy (process ... Explain accounting treatment of VAT as suggested by ICAI. ... Q26

Value Added Tax 148

FORM DVAT 16 (Refer Rules 28 and 29 of Delhi Value)

Added Tax Rules, 2005

DELHI VALUE ADDED

TAX RETURN

R1 Tax period Form / / To / /

mm dd yy mm dd yy

R2.1 Registration No/TIN

R2.2 Full Name of Dealer

R2.3 Address

R3 Description of top 3 items you deal in

(In order of volume of sale for the Tax period,

1-highlights volume to 3-lowest volume

1

2

3

R4 Turnover Total Turnover ( `) Output Tax (`)

R4.1 Goods Taxable At 1%

R4.2 Goods Taxable At 4%

R4.3 Goods Taxable At 12.5%

R4.4 Goods Taxable At 20%

R4.5 Works contract taxable @ 12.5%

R4.6 Exempt sales

R4.7 Output Tax before adjustments Sub Total ( A )

R4.8 Adjustments to output tax (Complete Schedule 1 and enter Total

S1.2 here ( B )

R4.9 Total Output Tax ( A

+ B )

R5 Computation of output tax Turnover (`) Output tax(`) R5.1 goods taxable at 1%

R5.2 good taxable at 4%

R5.2(1) Goods taxable at 5%

R5.3good taxable at 12.5%

R5.4 Good taxable at 20%

R5.5 work contract taxable at 4%

R5.5(1) works contract taxable @ 5%

R5.6 work contract taxable at 12.5%

Original/Revised

……………

If revised- 1. Date of original return-

2. Acknowledgement /

Receipt No.-

3. Date of discovery of

mistake or error…….

Attach a note explaining the

revisions

Refund Claimed?

Yes

No

Page 149: sipoysatish@gmail · PDF file · 2013-08-07Explain the concept of Value Added Tax / Explain Taxonomy (process ... Explain accounting treatment of VAT as suggested by ICAI. ... Q26

Value Added Tax 149

R5.7 Exempt sales

R5.8 Output Taxable adjustments Sub total (A)

R5.9 Adjustments to output tax

(Complete Annexure and enter Total 2 here)

Total A 2

From

ANNEXURE

(B)

R5.10 Total output Tax (A+B)

R6 Turnover of Purchases in Delhi (excluding

Tax) & Tax credit Purchases (`) Tax Credit (`)

R6.1 Capital goods

R6.2 Other goods

R6.2(1) Goods taxable at 1%

R6.2(2) Goods taxable at 4%

R6.2(3) Goods taxable at 5%

R6.2(4) Goods taxable at 12.5%

R6.2(5) Goods taxable at 20%

R6.2(6) Works Contract taxable at 4%

R6.2(6)(1) works contract taxable @ 5%

R6.2(7) Works Contract taxable at 12.5%

R6.2(8) Exempted purchase

R6.3 Tax Credit before adjustment sub total (A)

R6.4 Adjustment to tax credits

(Complete Annexure and enter Total A4 here) Total A4 from

Annexure

(B)

R6.5 Total Tax credits (A + B)

R7.1 Net Tax (R5.10)-(R6.5)

R7.2 Add: Interest, if payable

R7.3 Add: penalty, if payable

R7.4 Less: Tax deducted at source (attached No. of TDS certificates in original)

R7.5 Balance payable (R7.1+R7.2-7.4)

R7.6 Less: Amount deposited by the dealer (attach proof of payment)

S. No. Date of deposit Challan No. Name of Bank and

Branch

Amount

R8 Net Balance* (R7.5-R7.6)

*The net balance should not be positive as the amount due has to be deposited before filing the return:

IF THE BALANCE OF LINE R8 IS NEGATIVE, PROVIDE DETAILS IN THIS BOX

Balance brought forward from line R8

R9.1 Adjusted against liability under Central Sales Tax

R9.2 Refund Claimed

R9.3 Balance carried forward to next tax period

Page 150: sipoysatish@gmail · PDF file · 2013-08-07Explain the concept of Value Added Tax / Explain Taxonomy (process ... Explain accounting treatment of VAT as suggested by ICAI. ... Q26

Value Added Tax 150

IF REFUND IS CLAMIED,PROVIDE DETAILS IN THIS BOOK

R10 Details OF Bank Account

R10.1 Account No.

R10.2 Account Type (Saving/Current etc.)

R10.3 MICR No.

R.10.4 Name of Bank & Branch

R11 Inter-state trade and exports/imports Inter-state

Sales/Exports

Inter-state

Purchases/Imports

R11.1 Against C Forms

R11.2 Against C+E1/E2 Forms

R 11.3 Inward/outward stock Transfers against F

Forms

R11.4 Against H Forms

R 11.5 Against I Forms

R 11.6 Against J Forms

R 11.7 Export to/Imports from outside India

R11.7(1)Exempted sale / purchase including High Sea

Sale etc

R 11.8 Other (not supported by any Form)

R 11.9 Capital goods

R 11.10 Total

R12 Verification

I/We________________________________________ hereby solemnly affirm and declare that the

information given hereinabove is true that the best of my/our knowledge and belief and nothing has been

concealed there from.

Signature of Authorized Signatory ………………………

Full Name (first name, middle, surname) ……………….

Designation/State …………………

Place ……………………

Date:

Day Month Year

Instructions for filling return form

1. Please complete all the fields in the form.

2. Insert N/A in any filed which is not applicable to you.

3. Return has to be field within the time limit prescribe in rule 28 of the DVAT Rules.

4. Each page of the return form shall be signed by the authorized signatory.

5. For reporting adjustments, please use the following conversation:

(a) Any amount that decrease the output tax or tax credits should be entered as a negative

amount with a negative sign (-) before it.

(b) Any amount that increases the output tax or tax credit should be entered as a positive

amount.

Page 151: sipoysatish@gmail · PDF file · 2013-08-07Explain the concept of Value Added Tax / Explain Taxonomy (process ... Explain accounting treatment of VAT as suggested by ICAI. ... Q26

Value Added Tax 151

ANNEXURE

(To be attached with the return where adjustment is Output Tax or Tax Credit are made)

A.1 Adjustment to Output Tax

Nature of adjustment Increase in

Output Tax

(A)

Decrease in

Output Tax

(B)

A1.1 Sale cancelled [Section8(1)(a)]

A1.2 Nature of sale changed [Section 8(1)(b)]

A1.3 Change is agreed consideration [Section 8(1)(c)]

A1.4 Goods sold returned [Section 8(1)(d)]

A1.5 Bed debts written off [Section 8(1)(e) and Rule 7A]

A 1.6 Bed debts recovered [Rule 7 A(3)]

A 1.7 Tax payable on goods held on the date of cancellation of

registration (Section 23)

A 1.8 others adjustments, if any (specify)

Total

A.2 Total Net Increase/(decrease) in Output Tax (A-B)

A.3 Adjustments to Tax Credits

Nature of Adjustment Increase in Tax

Credit

(C)

Decrease in Tax

Credit

(D)

A3.1 Tax credit carried forward from previous tax period

A 3.2 Receipt of debit note from the seller [Section 10(1)]

A 3.3 Receipt of credit notes from seller [Section 10(1)]

A 3.4 Goods purchased returned or rejected [Section 10(1)]

A 3.5 Change in use of goods, for purposes other than for which

credit is allowed [ Section 10(2)(a)

A 3.6 Change is use of goods, for purposes for which credit is

allowed [Section 10(2)(b)]

A 3.7 Tax credit disallowed in respect of stock transfer out of

Delhi [Section 14)

A 3.8 Tax credit for transitional stock held on 1st April 2005

(Section 14)

A 3.9 Tax credit for purchases of second hand goods (Section 15)

A 3.10 Tax credits foe goods held on the date of withdrawal from

composition Scheme (Section 16)

A 3.11 The credit for trading stock and raw materials held at the

time of registration (Section 20)

A 3.12 Tax credit allowed for goods lost or destroyed (Rule 7)

A 3.13 Tax credit adjustment on sale or stock transfer of capital

goods [Section 9(9)(a)]

A 3.14 Other adjustments if any (specify)

Page 152: sipoysatish@gmail · PDF file · 2013-08-07Explain the concept of Value Added Tax / Explain Taxonomy (process ... Explain accounting treatment of VAT as suggested by ICAI. ... Q26

Value Added Tax 152

Total

A4. Total net Increase/(decrease) in Tax Credit (C-D)

Annexure 2A

(See instruction 6)

SUMMARY OF PURCHASE / INWARD BRANCH TRANSFER REGISTER (Month-wise)

(To be filled along with return)

Name of the Dealer:

Purchase for the tax period: From_____to_____

TIN :

Address

(Summary of Purchase (As per DVAT-30)

Sr.

No.

Month Seller’s

TIN

Seller’s

Name

Purchases not eligible for credit of input tax Purchases eligible for credit of input tax

Import

from

outside

India

Interstate

purchases

or stock

transfers

Purchases

from

exempted

units

Total

Purchases

excluding

tax,

if any

Local

purchases

Rate

of

tax

Input

Tax

paid

Total

purchase

including

tax

Capital

Goods

Others

1. 2. 3. 4. 5. 6. 7. 8. 9. 10. 11. 12. 13.

TOTAL

Note: Purchases made from un -registered dealers may be reported in one row for a month.

Signature of Dealer /

Authorised Signatory

Page 153: sipoysatish@gmail · PDF file · 2013-08-07Explain the concept of Value Added Tax / Explain Taxonomy (process ... Explain accounting treatment of VAT as suggested by ICAI. ... Q26

Value Added Tax 153

Annexure-2B

(See instruction 6)

SUMMARY OF SALE/OUTWARD BRANCH TRANSFER REGISTER (Month-wise)

(To be filled along with return)

Name of the Dealer:

Sale for the tax period : From____to_____

TIN

Address:

Summary of Sales (As per DVAT –31)

Sr.

No.

Month Buyer’s

TIN

Buyer’s

Name

Inter-state

Branch/

Consignment

transfer

Exports

Out of

India

Inter State Sales Local Sales

Sale Price

(excluding

CST)

Central

Sales

Tax

Total

(7 +

8)

Sales

Price

(excluding

Tax)

Output

Tax Total

(10

+11)

1. 2. 3. 4. 5. 6. 7. 8. 9. 10. 11. 12.

Total

Note : Sales Made to un-registered dealers may be reported in one row for a month.

Signature of Dealer / Authorised Signatory”

Page 154: sipoysatish@gmail · PDF file · 2013-08-07Explain the concept of Value Added Tax / Explain Taxonomy (process ... Explain accounting treatment of VAT as suggested by ICAI. ... Q26

Value Added Tax 154

SELF READING

A WHITE PAPER ON STATE-LEVEL VALUE ADDED TAX

By

The Empowered Committee of State Finance Ministers

(Constituted By the Ministry of Finance, Government of India on the Basis of

Resolution Adopted in the Conference of the Chief Ministers on November 16, 1999)

New Delhi January 17, 2005

PREFACE

This White Paper is a result of collective efforts of all the States in formulating the basic design of the State-

level Value Added Tax (VAT) through repeated and candid discussions in the Empowered Committee of

State Finance Ministers.

The State-level VAT, as elaborated in this White Paper, has certain distinct advantages over the existing

sales tax structure. The VAT will not only provide full set-off for input tax as well as tax on previous

purchases, but it will also abolish the burden of several of the existing taxes, such as turnover tax, surcharge

on sales tax, additional surcharge, special additional tax, etc. In addition, Central Sales Tax is also going to

be phased out. As a result, the overall tax burden will be rationalised, and prices, in general, will fall.

Moreover, VAT will replace the existing system of inspection by a system of built-in self-assessment by

traders and manufacturers. The tax structure will become simple and more transparent. This will

significantly improve tax compliance and will also help increase revenue growth.

While this State-level VAT has all these advantages, it is a State subject derived from Entry 54 of the State

List, for which the States are sovereign in taking decisions. On these decisions on VAT, the States, through

discussion in the Empowered Committee, have found it in their interests, to avoid unhealthy competition and

have certain features of VAT to be common for all the States. These features will constitute the basic design

of VAT. At the same time, the States will have freedom for appropriate variations consistent with this basic

design. This White Paper is a collective attempt of the States to strike a balance between this needed

commonality and the desired federal flexibility in the VAT structure.

The White Paper also strikes a balance between what is possible in the VAT design to begin with and what

can be improved upon in subsequent years as we gather more experience.

The White Paper further mentions how after working out a consensus on this VAT design, nearly all the

States either have finalised their VAT Bills by now and are in the process of obtaining Presidential Assent,

or will reach that stage very soon. Even for one major State where there are some ground-level problems, a

positive interaction with the Empowered Committee has recently opened up the possibility of resolving most

of these problems.

Page 155: sipoysatish@gmail · PDF file · 2013-08-07Explain the concept of Value Added Tax / Explain Taxonomy (process ... Explain accounting treatment of VAT as suggested by ICAI. ... Q26

Value Added Tax 155

These efforts of the States towards formulation of VAT design and its implementation have received full

cooperation of the Finance Ministry, Government of India. At the same time, the Finance Ministry has never

imposed their views on us. We, therefore, remain thankful to the former Union Finance Ministers––Dr.

Manmohan Singh, Shri Yashwant Sinha and Shri Jaswant Singh. We are specially grateful to Shri P.

Chidambaram, the present Union Finance Minister, for his active support over the last eight months, when

he not only helped formulate the modality of Central financial support to the States for possible loss of

revenue in the transitional years of implementation of VAT, but also took time off his busy schedule to

participate with us in the campaign for VAT in the States.

It has always been fruitful to have interaction with Dr. Parthasarathi Shome, Adviser to the Union Finance

Minister, for his insightful observations on the analytical structure of VAT as well as his reference to vast

experience in the implementation of VAT. The Secretary, Revenue, Additional Secretary, Revenue and all

the concerned officials of the Revenue Department of the Finance Ministry have helped us by participating

in the discussions whenever we requested them, and also by assisting in various procedural matters.

Interaction with Dr. Govinda Rao, the Chairman of Technical Experts Committee on VAT and other

members of the Committee has also been useful. We take this opportunity to thank all of them.

Discussions with the representatives of trade organizations and chambers of commerce and industry at the

national level as well as in the States have been relevant in assessing the ground level difficulties. Together

with them, we are determined to overcome these difficulties in implementing VAT in the States. We remain

thankful to them, and our mutual interaction will take place regularly.

Finally, this White Paper could be written only on the basis of lively support of the Finance Ministers of the

States, and with constant help from the Finance Secretaries and the Commissioners of Commercial Taxes of

the States. The Commissioners of Commercial Taxes have often burnt their midnight oil, and their

contribution should be particularly recorded. Shri Ramesh Chandra, Member-Secretary of the Empowered

Committee had to carry on the difficult administrative task in the functioning of the Empowered Committee.

We appreciate the efforts of Shri Chandra and the staff of the Empowered Committee.

Even after all these efforts, there may be some unavoidable shortcomings in this White Paper, which we will

try to overcome as we learn more from the actual experience of implementation of VAT. With this

background and the attitude, this White Paper is an expression of the genuine commitment of the States to

the implementation of VAT from April 1, 2005, which we are all looking forward to.

Asim Kumar Dasgupta

Convenor,

Empowered Committee of

State Finance Ministers,

and

New Delhi, Finance Minister,

January 17, 2005. Government of West Bengal.

Page 156: sipoysatish@gmail · PDF file · 2013-08-07Explain the concept of Value Added Tax / Explain Taxonomy (process ... Explain accounting treatment of VAT as suggested by ICAI. ... Q26

Value Added Tax 156

A White Paper On State-Level Value Added Tax

This White Paper on State-level Value Added Tax (VAT) is presented in three parts. To begin with, the

justification of VAT and its background have been mentioned (Part 1). In Part 2, the main design of VAT, as

evolved on the basis of a consensus among the States through repeated discussions in the Empowered

Committee, has been elaborated. While doing so, it is recognized that this VAT is a State subject and

therefore the States will have freedom for appropriate variations consistent with the basic design as agreed

upon at the Empowered Committee. Finally, in Part 3, the other related issues have been discussed for

effective implementation of VAT.

1. Justification of VAT and Background

1.1 In the existing sales tax structure, there are problems of double taxation of commodities and multiplicity

of taxes, resulting in a cascading tax burden. For instance, in the existing structure, before a commodity is

produced, inputs are first taxed, and then after the commodity is produced with input tax load, output is

taxed again. This causes an unfair double taxation with cascading effects. In the VAT, a set-off is given for

input tax as well as tax paid on previous purchases. In the prevailing sales tax structure, there is in several

States also a multiplicity of taxes, such as turnover tax, surcharge on sales tax, additional surcharge, etc.

With introduction of VAT, these other taxes will be abolished. In addition, Central sales tax is also going to

be phased out. As a result, overall tax burden will be rationalised, and prices in general will also fall.

Moreover, VAT will replace the existing system of inspection by a system of built-in self-assessment by the

dealers and auditing. The tax structure will become simple and more transparent. That will improve tax

compliance and also augment revenue growth. Thus, to repeat, with the introduction of VAT, benefits will

be as follows:

a set-off will be given for input tax as well as tax paid on previous purchases other taxes, such as turnover

tax, surcharge, additional surcharge, etc. will be abolished overall tax burden will be rationalized prices will

in general fall there will be self-assessment by dealers transparency will increase there will be higher

revenue growth

The VAT will therefore help common people, traders, industrialists and also the Government. It is indeed a

move towards more efficiency, equal competition and fairness in the taxation system.

1.2 For these beneficial effects, a full-fledged VAT was initiated first in Brazil in mid 1960’s, then in

European countries in 1970’s and subsequently introduced in about 130 countries, including several federal

countries. In Asia, it has been introduced by a large number of countries from China to Sri Lanka. Even in

India, there has been a VAT system introduced by the Government of India for about last ten years in

respect of Central excise duties. At the State-level, the VAT system as decided by the State Governments,

would now be introduced in terms of Entry 54 of the State List of the Constitution.

1.3 The first preliminary discussion on State-level VAT took place in a meeting of Chief Ministers convened

by Dr. Manmohan Singh, the then Union Finance Minister in 1995. In this meeting, the basic issues on VAT

were discussed in general terms and this was followed up by periodic interactions of State Finance

Ministers. Thereafter, in a significant meeting of all Chief Ministers, convened on November 16, 1999 by

Shri Yashwant Sinha, the then Union Finance Minister, three important decisions were taken. First, before

the introduction of State-level VAT, the unhealthy sales tax rate “war” among the States would have to end

and sales tax rates would need to be harmonised by implementing uniform floor rates of sales tax for

different categories of commodities with effect from January 1, 2000. Second, in the interest again of

harmonisation of incidence of sales tax, the sales-tax-related industrial incentive schemes would also have to

Page 157: sipoysatish@gmail · PDF file · 2013-08-07Explain the concept of Value Added Tax / Explain Taxonomy (process ... Explain accounting treatment of VAT as suggested by ICAI. ... Q26

Value Added Tax 157

be discontinued with effect from January 1, 2000. Third, on the basis of achievement of the first two

objectives, steps would be taken by the States for introduction of State-level VAT after adequate

preparation. For implementing these decisions, an Empowered Committee of State Finance Ministers was

set-up.

1.4 Thereafter, this Empowered Committee has met regularly, attended by the State Finance Ministers, and

also by the Finance Secretaries and the Commissioners of Commercial Taxes of the State Governments as

well as senior officials of the Revenue Department of the Ministry of Finance, Government of India.

Through repeated discussions and collective efforts in the Empowered Committee, it was possible within a

period of about a year and a half to achieve nearly 98 per cent success in the first two objectives on

harmonisation of sales tax structure through implementation of uniform floor rates of sales tax and

discontinuation of sales-tax- related incentive schemes. As a part of regular monitoring, whenever any

deviation is reported from the uniform floor rates of sales tax, or from decision on incentives, the

Empowered Committee takes up the matter with the concerned State and also the Government of India for

necessary rectification.

1.5 After reaching this stage, steps were initiated for systematic preparation for the introduction of State-

level VAT. In order again to avoid any unhealthy competition among the States which may lead to

distortions in manufacturing and trade, attempts have been made from the very beginning to harmonise the

VAT design in the States, keeping also in view the distinctive features of each State and the need for federal

flexibility. This has been done by the States collectively agreeing, through repeated discussions in the

Empowered Committee, to certain common points of convergence regarding VAT, and allowing at the same

time certain flexibility for the local characteristics of the States.

1.6 Along with these measures at ensuring convergence on the basic issues on VAT, steps have also been

taken for necessary training, computerisation and interaction with trade and industry, particularly at the State

levels. This interaction with trade and industry is being specially emphasised.

1.7 It may be noted that while such preparation was going on, the Chief Ministers of all the States in an

important meeting on State-level VAT convened by the Prime Minister on October 18, 2002, when Shri

Jaswant Singh, the then Union Finance Minister was present, clearly stated their intention of introducing

VAT from April 1, 2003. About 29 States and Union Territories had expeditiously sent their Bills to the

Ministry of Finance, Government of India for prior vetting. The Union Ministry of Finance had considered

these Bills of States and Union Territories, and sent their comments/ suggestions to the States and Union

Territories in line with the decisions of the Empowered Committee of the State Finance Ministers for

incorporating the same in VAT Bills to be placed in the State legislatures and subsequent transmission to the

Government of India for Presidential Assent. At this stage, there were certain developments which delayed

the introduction of VAT. Despite these developments, most of the States remained positively interested in

implementation of VAT. Madhya Pradesh VAT Bill had already been accorded Presidential Assent in

November 2002. One State, namely, Haryana, has already introduced VAT on its own with good results on

revenue growth. It is important to note that in the meeting of Empowered Committee on June 18, 2004 when

Shri P. Chidambaram, the Union Finance Minister, was invited and was kindly present, all the States,

excepting one, once again categorically renewed their commitment to the introduction of VAT from April 1,

2005. Even for this particular State with certain problems, a positive interaction has recently been organised

with that State to resolve certain genuine ground-level problems. Now nearly all the States have either

finalised their VAT Bills and are in the process of obtaining Presidential Assent, or will reach that stage very

soon.

2. Design of State-Level VAT

2.1 As already mentioned, the design of State-level VAT has been worked out by the Empowered

Committee through several rounds of discussion and striking a federal balance between the common points

of convergence regarding VAT and flexibility for the local characteristics of the States. Since the State-level

Page 158: sipoysatish@gmail · PDF file · 2013-08-07Explain the concept of Value Added Tax / Explain Taxonomy (process ... Explain accounting treatment of VAT as suggested by ICAI. ... Q26

Value Added Tax 158

VAT is centred around the basic concept of “set-off” for the tax paid earlier, the needed common points of

convergence also relate to this concept of set-off/input tax credit, its coverage and related issues as

elaborated below.

Concept of VAT and Set-off / Input Tax Credit

2.2 The essence of VAT is in providing set-off for the tax paid earlier, and this is given effect through the

concept of input tax credit/rebate. This input tax credit in relation to any period means setting off the amount

of input tax by a registered dealer against the amount of his output tax. The Value Added Tax (VAT) is

based on the value addition to the goods, and the related VAT liability of the dealer is calculated by

deducting input tax credit from tax collected on sales during the payment period (say, a month).

If, for example, input worth `1,00,000/- is purchased and sales are worth `2,00,000/- in a month, and input

tax rate and output tax rate are 4% and 10% respectively, then input tax credit/set-off and calculation of

VAT will be as shown below:

(a) Input purchased within the month: ` 1,00,000/-

(b) Output sold in the month: ` 2,00,000/-

(c) Input tax paid: ` 4,000/-

(d) Output tax payable: ` 20,000/-

(e) VAT payable during the month: ` 16,000/-

after set-off/input tax credit

[(d) – (c)]

Coverage of Set-Off / Input Tax Credit

2.3 This input tax credit will be given for both manufacturers and traders for purchase of inputs/supplies

meant for both sale within the State as well as to other States, irrespective of when these will be

utilised/sold. This also reduces immediate tax liability.

Even for stock transfer/consignment sale of goods out of the State, input tax paid in excess of 4% will be

eligible for tax credit.

Carrying Over of Tax Credit

2.4 If the tax credit exceeds the tax payable on sales in a month, the excess credit will be carried over to the

end of next financial year. If there is any excess unadjusted input tax credit at the end of second year, then

the same will be eligible for refund.

Input tax credit on capital goods will also be available for traders and manufacturers. Tax credit on capital

goods may be adjusted over a maximum of 36 equal monthly instalments. The States may at their option

reduce this number of instalments.

There will be a negative list for capital goods (on the basis of principles already decided by the Empowered

Committee) not eligible for input tax credit.

Treatment of Exports, etc.

2.5 For all exports made out of the country, tax paid within the State will be refunded in full, and this refund

will be made within three months. Units located in SEZ and EOU will be granted either exemption from

payment of input tax or refund of the input tax paid within three months.

Inputs Procured from Other States

Page 159: sipoysatish@gmail · PDF file · 2013-08-07Explain the concept of Value Added Tax / Explain Taxonomy (process ... Explain accounting treatment of VAT as suggested by ICAI. ... Q26

Value Added Tax 159

2.6 Tax paid on inputs procured from other States through inter-State sale and stock transfer will not be

eligible for credit. However, a decision has been taken for duly phasing out of inter-State sales tax or Central

sales tax. As a preparation for that, a comprehensive inter-State tax information exchange system is

also being set up.

Treatment of Opening Stock

2.7 All tax-paid goods purchased on or after April 1, 2004 and still in stock as on April 1, 2005 will be

eligible to receive input tax credit, subject to submission of requisite documents. Resellers holding tax-paid

goods on April 1, 2005 will also be eligible. VAT will be levied on the goods when sold on and after April

1, 2005 and input tax credit will be given for the sales tax already paid in the previous year. This tax credit

will be available over a period of 6 months after an interval of 3 months needed for verification.

Compulsory Issue of Tax Invoice, Cash Memo or Bill

2.8 This entire design of VAT with input tax credit is crucially based on documentation of tax invoice, cash

memo or bill. Every registered dealer, having turnover of sales above an amount specified, shall issue to the

purchaser serially numbered tax invoice with the prescribed particulars. This tax invoice will be signed and

dated by the dealer or his regular employee, showing the required particulars. The dealer shall keep a

counterfoil or duplicate of such tax invoice duly signed and dated. Failure to comply with the above will

attract penalty.

Registration, Small Dealers and Composition Scheme

2.9 Registration of dealers with gross annual turnover above `5 lakh will be compulsory. There will be

provision for voluntary registration. All existing dealers will be automatically registered under the VAT Act.

A new dealer will be allowed 30 days time from the date of liability to get registered.

Small dealers with gross annual turnover not exceeding `5 lakh will not be liable to pay VAT. States will

have flexibility to fix threshold limit within `5 lakh.

Small dealers with annual gross turnover not exceeding ` 50 lakh who are otherwise liable to pay VAT, shall

however have the option for a composition scheme with payment of tax at a small percentage of gross

turnover. The dealers opting for this composition scheme will not be entitled to input tax credit.

Tax Payer’s Identification Number (TIN)

2.10 The Tax Payer’s Identification Number will consist of 11 digit numerals throughout the country. First

two characters will represent the State Code as used by the Union Ministry of Home Affairs. The set-up of

the next nine characters may, however, be different in different States.

Return

2.11 Under VAT, simplified form of returns will be notified. Returns are to be filed monthly/quarterly as

specified in the State Acts/Rules, and will be accompanied with payment challans. Every return furnished by

dealers will be scrutinized expeditiously within prescribed time limit from the date of filing the return. If any

technical mistake is detected on scrutiny, the dealer will be required to pay the deficit appropriately.

Procedure of Self-Assessment of VAT Liability

2.12 The basic simplification in VAT is that VAT liability will be self-assessed by the dealers themselves in

terms of submission of returns upon setting off the tax credit. Return forms as well as other procedures will

be simple in all States. There will no longer be compulsory assessment at the end of each year as is existing

now. If no specific notice is issued proposing departmental audit of the books of accounts of the dealer

within the time limit specified in the Act, the dealer will be deemed to have been self-assessed on the basis

of returns submitted by him.

Page 160: sipoysatish@gmail · PDF file · 2013-08-07Explain the concept of Value Added Tax / Explain Taxonomy (process ... Explain accounting treatment of VAT as suggested by ICAI. ... Q26

Value Added Tax 160

Because of the importance of the concept of self-assessment in VAT, provision for “self-assessment” will be

stated in the VAT Bills of the States.

Audit

2.13 Correctness of self-assessment will be checked through a system of Departmental Audit. A certain

percentage of the dealers will be taken up for audit every year on a scientific basis. If, however, evasion is

detected on audit, the concerned dealer may be taken up for audit for previous periods. This Audit Wing will

remain delinked from tax collection wing to remove any bias. The audit team will conduct its work in a time

bound manner and audit will be completed within six months. The audit report will be transparently sent to

the dealer also.

Simultaneously, a cross-checking, computerised system is being worked out on the basis of coordination

between the tax authorities of the State Governments and the authorities of Central Excise and Income Tax

to compare constantly the tax returns and set-off documents of VAT system of the States and those of

Central Excise and Income Tax. This comprehensive cross-checking system will help reduce tax evasion

and also lead to significant growth of tax revenue. At the same time, by protecting transparently the interests

of tax-complying dealers against the unfair practices of tax-evaders, the system will also bring in more equal

competition in the sphere of trade and industry.

Declaration Form

2.14 There will be no need for any provision for concessional sale under the VAT Act since the provision

for setoff makes the input zero-rated. Hence, there will be no need for declaration form, which will be a

further relief for dealers.

Incentives

2.15 Under the VAT system, the existing incentive schemes may be continued in the manner deemed

appropriate by the States after ensuring that VAT chain is not affected.

Other Taxes

2.16 As mentioned earlier, all other existing taxes such as turnover tax, surcharge, additional surcharge and

Special Additional Tax (SAT) would be abolished. There will not be any reference to these taxes in the VAT

Bills. The States that have already introduced entry tax and intend to continue with this tax should make it

vatable. If not made vatable, entry tax will need to be abolished. However, this will not apply to entry tax

that may be levied in lieu of octroi.

Penal Provisions

2.17 Penal provisions in the VAT Bills should not be more stringent than in the existing Sales Tax Act.

Coverage of Goods under VAT

2.18 In general, all the goods, including declared goods will be covered under VAT and will get the benefit

of input tax credit.

The only few goods which will be outside VAT will be liquor, lottery tickets, petrol, diesel, aviation turbine

fuel and other motor spirit since their prices are not fully market determined. These will continue to be taxed

under the Sales Tax Act or any other State Act or even by making special provisions in the VAT Act itself,

and with uniform floor rates decided by the Empowered Committee.

VAT Rates and Classification of Commodities

2.19 Under the VAT system covering about 550 goods, there will be only two basic VAT rates of 4% and

12.5%, plus a specific category of tax-exempted goods and a special VAT rate of 1% only for gold and

silver ornaments, etc. Thus the multiplicity of rates in the existing structure will be done away with under

the VAT system.

Page 161: sipoysatish@gmail · PDF file · 2013-08-07Explain the concept of Value Added Tax / Explain Taxonomy (process ... Explain accounting treatment of VAT as suggested by ICAI. ... Q26

Value Added Tax 161

Under exempted category, there will be about 46 commodities comprising of natural and unprocessed

products in unorganized sector, items which are legally barred from taxation and items which have social

implications. Included in this exempted category is a set of maximum of 10 commodities flexibly chosen by

individual States from a list of goods (finalised by the Empowered Committee) which are of local social

importance for the individual States without having any inter-state implication. The rest of the commodities

in the list will be common for all the States. Under 4% VAT rate category, there will be the largest number

of goods (about 270), common for all the States, comprising of items of basic necessities such as medicines

and drugs, all agricultural and industrial inputs, capital goods and declared goods. The schedule of

commodities will be attached to the VAT Bill of every State. The remaining commodities, common for all

the States, will fall under the general VAT rate of 12.5%.

In terms of decision of the Empowered Committee, VAT on AED items relating to sugar, textile and

tobacco, because of initial organisational difficulties, will not be imposed for one year after the introduction

of VAT, and till then the existing arrangement will continue. The position will be reviewed after one year.

Effects of the VAT System

2.20 This design of the State-level VAT has been carefully worked out by the Empowered Committee after

repeated interactions with the States and others concerned and striking a balance between the needed

convergence and federal flexibility as well as ground-level reality. If now all the components of the VAT

design are taken together, then it will be seen that the total effect of this VAT system will be to rationalise

the tax burden and bring down, in general, the price level. This will also stop unhealthy tax-rate “war” and

trade diversion among the States, which had adversely affected interests of all the States in the past.

Moreover, this VAT design will also significantly bring in simplicity and transparency in the tax structure,

thereby improving tax-compliance and eventually also the revenue growth, as mentioned in the beginning.

3. Steps Taken by the States

3.1 It is now of significance to note that most of the States, after collective interaction in the Empowered

Committee, have either already modified or agreed to modify their VAT Bills by incorporating these

common points of convergence including flexibility as mentioned in the VAT design above, and are also

taking other preparatory steps towards introduction of VAT from April 1, 2005.

3.2 As a part of the preparatory steps, the States have started the process of preparing the draft of VAT

Rules, including Books of Accounts to be maintained. The objective will be to keep these as simple as

possible so that it becomes easy for a small trader to comply with the requirements.

3.3 Moreover, the States have initiated, and in many cases also completed, steps for computerisation upto

the levels of assessing officers and also at the check posts. This process will continue since this is extremely

important for document-based verification and integration with Taxation Information Exchange System as

well as with information of the Central excise and income tax systems as indicated earlier.

3.4 It may be mentioned here that appropriate Central funds for VAT-related computerisation in the North-

Eastern States are also being released by the Government of India.

4. Related Issues

4.1 While the States have thus taken several steps towards introduction of VAT, certain supporting decisions

were critically needed at the national level for more effective implementation of VAT from April 1, 2005.

4.2 It needs to be carefully noted that although introduction of VAT may, after a few years, lead to revenue

growth, there may be a loss of revenue in some States in the initial years of transition. It is with this in view

that the Government of India had agreed to compensate for 100 per cent of the loss in the first year, 75 per

cent of the loss in the second year and 50 per cent of the loss in the third year of introduction of VAT, and

the loss would be computed on the basis of an agreed formula. This position has not only been reaffirmed by

the Union Finance Minister in his Budget Speech of 2004-05, but a concrete formula for this compensation

Page 162: sipoysatish@gmail · PDF file · 2013-08-07Explain the concept of Value Added Tax / Explain Taxonomy (process ... Explain accounting treatment of VAT as suggested by ICAI. ... Q26

Value Added Tax 162

has also now been worked out after interaction between the Union Finance Minister and the Empowered

Committee.

4.3 As mentioned earlier, there is also a need, after introduction of VAT, for phasing out of Central Sales

Tax (CST). However, the States are now collecting nearly ` 15 thousand crore every year from CST. There

is accordingly a need of compensation from the Government of India for this loss of revenue as CST is

phased out. Moreover, while CST is phased out, there is also a critical need for putting in place a regulatory

frame-work in terms of Taxation Information Exchange System to give a comprehensive picture of inter-

State trade of all commodities. As already mentioned, this process of setting up of Taxation Information

Exchange System has already been started by the Empowered Committee, and is expected to be completed

within one year. The position regarding CST will be reviewed by the Empowered Committee during 2005-

06, and suitable decision on the phasing out of CST will be taken.

4.4 It is also essential to bring imports into the VAT chain. Because of the set-off, this will not result in any

tax cascading effect, but will only improve tax compliance. A proposal for VAT on imports, including the

collection mechanism with adequate safeguards for the protection of interest of land-locked States, is being

discussed with the Government of India.

4.5 Similarly, discussion between the Empowered Committee and the Government of India is going on for

an early decision on the question of collection and appropriation of service tax by the Centre and the States.

If decisions on VAT on imports and service tax are taken expeditiously at the national level, then these two

important spheres of taxation can be integrated, along with the AED items as mentioned earlier, into the

VAT system of the States from the second year of introduction of VAT.

4.6 It may be noted that this VAT design has been worked out carefully by the Empowered Committee to

strike a balance not only between the common points of convergence and federal flexibility, but also a

balance between what can be done to begin with and what should be incorporated subsequently for further

perfection of the VAT system.

4.7 For successful implementation of State-level VAT, close interaction with trade and industry is specially

important. The Empowered Committee has therefore also set up a Consultative Committee with one

representative from each of the national level trade organisations and national level chambers of commerce

and industry. This Committee has already started interacting with the Empowered Committee. This process

of interaction will continue regularly to discuss issues and sort out problems of implementation of VAT.

Such Consultative Committees will also be set up at the level of each State, and interaction with the State

Government will take place in a similarly regular manner.

4.8 In course of discussion with representatives of trade and industry, reference has often been made to the

earlier VAT Bills of some of the States. It should be clearly noted, as already mentioned before, that all the

States have agreed to amend their earlier VAT Bills so as to conform broadly to the common design as

elaborated in this White Paper. This process of amendment has also already started. The point of reference

on VAT should therefore be this design of VAT as explained in this White Paper. It should also be

mentioned that there are some important points on the ground-level implementation of VAT which have

been raised by the representatives of trade and industry. Many of the points will be taken care of in the VAT

rules of the States, with changes where necessary.

4.9 Finally, a comprehensive campaign on State-level will be launched to communicate in simple and

transparent manner the benefit of VAT for common people, traders, industrialists and also the State

Governments. This campaign will then be launched first at the national level on the basis of necessary

coordination between the States and the Centre. This will then be simultaneously followed up at the level of

every State and also in districts of the States. This campaign will be based on written materials as well as

Page 163: sipoysatish@gmail · PDF file · 2013-08-07Explain the concept of Value Added Tax / Explain Taxonomy (process ... Explain accounting treatment of VAT as suggested by ICAI. ... Q26

Value Added Tax 163

publicity through all media. The purpose of this campaign will be a two-way interaction between the

Government and the trade and industry as well as the common people.

There is now only looking forward to the introduction of State-level VAT by all the States and Union

Territories from April 1, 2005. We seek cooperation of all sections of people in the country.

——————

Cost Sheet Format

Particulars Amount Amount

Opening Stock Of Raw Material -

Add: Purchase Of Raw Material -

Add: Purchase Expenses -

Less: Closing Stock of Raw Material -

Raw Material Consumed -

Direct Wages (Labour) -

Direct Charges -

Prime cost (1) -

Add:- Factory Over Heads:

Factory Rent -

Factory Power -

Indirect Material -

Indirect Wages -

Supervisor Salary -

Drawing Officer Salary -

Factory Insurance -

Factory Asset Depreciation -

Works cost Incurred -

Add: Opening Stock of WIP -

Less: Closing Stock of WIP -

Works cost (2) -

Add:-Administration Over Heads:

Office Rent -

Asset Depreciation -

General Charges -

Audit Fees -

Bank Charges -

Page 164: sipoysatish@gmail · PDF file · 2013-08-07Explain the concept of Value Added Tax / Explain Taxonomy (process ... Explain accounting treatment of VAT as suggested by ICAI. ... Q26

Value Added Tax 164

Counting House Salary -

Other Office Expenses -

Cost of Production(3) -

Add: Opening Stock of Finished Goods -

Less: Closing Stock of Finished Goods -

Cost of Goods Sold -

Add:- Selling and Distribution Over Head:-

Sales man Commission -

Sales man Salary -

Travelling Exps -

Advertisement -

Delivery man Exps -

Sales Tax -

Bad Debts -

Cost of Sales (5) -

Profit (balancing figure)

Sales